Vous êtes sur la page 1sur 212

New collection of MCQ other thanthe 1000 of ALQASEM:

1-initial regulation of BP in vascular system occur at :


a- Arterioles
b-Aorta and its branches
c-Heart
d-Capillaries
e-Vein and venules
________________________________________________________________________
2-hematological disease occurs in children treated with heparin and fresh frozen
plasma what is the disease :
A)hemophilia A
B)hemophilia B
C )VON WILL brand disease
d)DIC thrombosis
3-Patient came with low iron and high AST and high MCV no megaloblasts in the
blood wt is the Dx?
Alcohol
Vitamin B12defeciency
Folic acid deficiency
Due to drugs
Patient with macrocytic anemia without megaloblast. Whats the most likely
diagnosis:
a. Folic acid
b. Vitamin B12 deficiency
c. Alcoholism
4- pregnant lady with hepatits, how to confirm dx :
a- ALP
B-SGOT
C-WBC
D-ESR
E-
________________________________________________________________
__
5-which of the following antidepressant drugs causes agitation, anxiety :
triAD
tetraAD
SSRI
1

6-PT WITH ASBESTOSISLEAD TO


Plural calcification
Plural effusion
Or bilateral fibrosis
7-which of the folowing is the most radiosenstive testicular tumor
a- semenoma
b- yolc sac tumor
c- germ cell tuor
d- choriocarcinoma
e- immature teratoma
8-ttt of pyoderma gangrenosum
a- sys steroids
b- methotrexate
________________________________________________________________
_
9-ttt of foliculitis after shaving the bread
a- oral steroid
b- topical steroid
c -oral antibiotics
may be there was topical antifungal but no topical antibiotic choice
10- Baby in NICU has a heart rate of 300, good blood pressure level. What
should u do:
DC shock
IV amiodrane
Digoxin
Carotid massage??
Newborn with 300 bpm , with normal BP , normal RR , what do you will do
for newborn : (atrial flutter) imp.
Cardiac Cardiversion
Verpamil
Digoxin
Diltzam iv
if you suspect atrial flutter : Consider digoxin if not already in use
because it frequently increases the conduction ratio and decreases
the ventricular rate. , Avoid adrenergic and atropinic agents during
sedation or anesthesia for cardioversion. Ketamine is relatively
contraindicated
__________________________________________________________________________
the cause of premature ventricular contraction?
decrease O2 supply to the muscle
decrease blood supply to the muscle
the heart trying to beat fast
2

________________________________________________________________
__
11-yr old sustained trauma to the chest present with severe short of breath with
cyanosis, his rt lung is silent with hyperresonance. The FIRST step to treat this pt:
a. O2 mask
b. Tube thoracostomy
c. CXR
________________________________________________________________
_
12- Patient with hypertension , DM, somking, which the following are most
important to be deal with :
obesity and HTN ???
smoking and obesity
smoking and HTN
13-baby with streptococcal pharyngitis:
- Ttt after 9 days carries no risk of GN
- Ttt effective in prevention of GN
- Clindamycin effective against gram ve organisms
all choices are wrong
14- Hx of trauma in DIP(finger hyperextention)with palm pain: (incomplateQ)
- Extraarticular fracture in DIP
- Intraarticular fracture in PID
- Superficial tendon tears
- Tendon profundus tear??
15-pt with pict of bilateral pneumonia , high grade fever , normal WBC . organism
causing that:
- Chlamydia pneumonae
- Leigonella sp. (cause neutropenia)
- Staph pneumonae
________________________________________________________________
_
16-ttt of cold induced urticaria:
- Cemitidine
-diphenhydramine.
________________________________________________________________
__
17-methyl-progesteron used for PPH what is contrindication :
Pregnant with asthma
Pregnant with hypertension
Pregnant with DM
__________________________________________________________________
18-what is the most common cause of death in patients with Ludwig's angina?
sepsis
3

Sudden asphyxiation
rupture of the wall
____________________________________________________________________
19- 4 days post c-section pt with profound hypotension
a-normal saline 5ooml IV with to big lines
b-dopamine
________________________________________________________________
__
20-Adult male during excercise he suddenly felt pain in the middle of his rt. Thigh
posteriorly. On exam. He has discoloration in the same site and mass in the
hamstring ms. No bone tenderness or palpable defect. Mx:
Surgery.
Splint.
Bandage.
Ice, elevation and bandage
Cast.
21-patient has complete ptosis in hih rt eye. pupil is out and down, fixed dilated.
restricted ocular movements. dx
a. 3rd n palsy.
b. 4th n palsy.
c. 3rd and 4th.
d. 6th n palsy
22-target lesion are found in erythema:
a-annular
b-marginatom
c-multiform
d-nodosum
_________________________________________________________________
23-patient came with cervical carcinoma next investigation :
-cone biopsy
- Direct biopsy
-pap smear
__________________________________________________________________
24- the best to give as DVT prophylaxis post surgery which is cost effective, safe
with high efficacy:
a) LMWH
b) Unfractioned Heparin
c) asprin
d) Warfarin
25-case of right hypocnodrial abdominal pain for 2 hours after eating for months
no fever no jaundice, no radiation and nothing suggistive of cholycytitis
what is the investigation
4

a- US
b- oral cholystogram
c- iv cholystogram
26-72- pt intubated ,the most reliable method to make sure for tube proper
position:
- 5 point auscultation bilaterally breathing heard
- CXR
27-23 years old female with regular menses. On US, she has a 7cm ovarian cyst.
otherwise everything is normal. dx:
a. corpus luteum cyst
b.follicular cyst
c.teratoma
d.another cancer
30- regarding paracetamol toxicity:
- Not toxic if dose exceed 150-180 mg
- Cause vomiting and neuropathy
- Therapeutic effect after 4 hours
- Use Deferoxamine
-the liver enzyme reach the max. Level 4-6 hours after ingestion
31- 22 yr, low HGB low PLT and high WBC , peripheral smear shows blast cell
with large nucleus and scant cytoplasm and some nucleoli -- positive
meyloperoxidase test and negative esterase , DDx:
- Acute lymphocytic
- Acute myelocytic
- Acute monocytic
________________________________________________________________
__
32- child with hyperemia and pulging of tym mem had previous history of
treated impetigo so ttt is:
- Cefuroxime
- Amoxicillin (not sure)
- Erythromycin
- Ceftriaxone
- Cephalexine
_________________________________________________________________
33-child , urine odor like burned sugar:
- Phenylketonuria
- Maple syrup urine disease
_________________________________________________________________
34-40 yo presented by hx of syncope when he do excurses and there was hix of
same complain when he on rest . and there is chest pain on ex. There was
ejection systolic murmur 2-4 degree most side is lower lf sternum not radiating to
5

other site increase when he lying down and there is non specific s and t changing
and there is lf atrium enlargement
Aortic stenosis
p.s
hypertrophic cardiomyopathy
constrictive cardiomyopathy
35-Pt. had chest pain and fainting , ECG shows st- elevation and significant Q
wave in -v4 and st-depression in inferior leads :
Ant. MI
Inf. MI
Pericarditis
Post. MI
________________________________________________________________
36-A patient with normal kidney function post MI. The troponin level will last for :
A. 48 h
B. 73 h
C. 24 h
D. 12 h
E. 8 h
37-In lung diseases like pneumonia, it usually affects certain site based on the
anatomy, this is more obvious in which of the following:
Right upper lobe
Right middle lobe
Right lower lobe
Left upper lobe
Left lower lobe
38-in lung diseases like pneumonia, it usually affects certain site based on the
anatomy, this is less obvious in which of the following:
Right upper lobe
Right middle lobe
Right lower lobe
Left upper lobe
Left lower lobe
39-Pt. above 40 , pregnant and ask you for down syndrome screening :
Triple test
US
Amniocentisis
Chorionicvillus
40-You received a call from a father how has a son diagnosed recently with DM-I
for six months, he said that he found his son lying down unconscious in his
bedroom, What you will tell him if he is seeking for advise:
a. Bring him as soon as possible to ER
b. Call the ambulance
6

c. Give him his usual dose of insulin


d. Give him IM Glucagone
e. Give him Sugar in Fluid per oral
_____________________________________________________________________
41- Pt. with headache and vertebral lesion (Moth-eaten),Investigation?
- Bone scan
-
-
42-uveitis is treated by
a-chloramphinicol drops
b-tetracine drops
and other choises i dont remember
uveitis is treated with cycloplegic and steroid
43-Which drug can use in acute back pain
Diazepam
Alprozam
Metoxelen
44-patint C/o menomentogia 6 month and pelvic pain .all investigatin and
examinatin normal what is ttt
Mafnemic
Compined pill
45- pt was PDD ve , know become + ve , there is no symptoms , normal x ray,
the management :
-Reassure
-Rifambicin and INH for 6 month
-Streptomycine for 7 month
_ rifambicin for 6 months
________________________________________________________________
__
46_ pt was PDD ve , know become + ve , there is no symptoms , normal x ray,
the management :
Isonized and rifampcin for 6 month
Isonized for 6 month
Isonized and rifampcin and streptomycin for 12 month
the treatment of latent TB:
1-INH for 6-9 months
2-alternative is rifampicin 4-6 month
47_During blood transfusion , the pt develop fever and pain at infusion site your
action:
-slow infusion+antibiotic
7

-slow infusion + acetaminophen


-stop infusion + crystalloid fluid
- stop infusion+ mannitol+acetaminophen
________________________________________________________________
__
48-To deffrentiate between sinua arythmia and atrial firbllation
a- carotid massage
b- Temporal artery massage
c- Amidrone
d- digoxin
________________________________________________________________
__
49-A 35 yr old pt she is on phenytoin since she was 29 due to partial epilipsy she
didnt have any attack since. She want to stop taking the drug due to facial hair
growth:
a. It is reasonable to stop it now
b. Stop it after 6 months
c. Stop after 10 years
d. Dont stop it
50-Patient with seasonal watery nasal discharge, sneezing and nasal block. What
should you give him as a treatment:
Topical steroid
Decongestants
Antihistamines
Systemic Steriods
51-child presented with decreased hearing for 1 year, on exam. there is fluid
behind the ear drum and adenoid hypertrophy. In addition to adenoidectomy what
will you do:
myringotomy.
gromet tube insertion.
antibiotics.
________________________________________________________________
__
52-Pt. with perforated tympanic membrane ttt:
Topical ABX
Topical steroid
Systemic ABX
Systemic steroid
53_Pregnant lady presented with bleeding from gums. On exam, spleen in
palpable 4 cm below the costal margin. Ix; platelets 50,000. Dx:
HELLP.
ITP
Gestational thrombocytopenia
Thromboembolic disorder.
8

54-40 ys female, multigravida, no sexual intercourse for 1 year bcz her husband
going abroad, C/O was intermenistrual bleeding with menorrhgia, provotional Dx:
Endometriosis
Endometrial CA ??
ch endometrites
this is a common age of DUB not sure of the answer but according to the choices
it is B
55-patient with bed sore involve skin and extend to fascia and muscle what a
grade
Grade1
Grade 2
Grade 3
Grade 4
56-patient with rhumatic heart disease and had mitral valve stenosis
Mitral vave diameter less than 1 mm
In order to maintain COP what will happen :
Left atrial hypertrophy and decrease pulmonary prusser
Left atrial hypertrophy and champer dilatation
RV hypertrophy and decrease pulmonary prusser
RV hypertrophy and champer dilatation
choices in other words
a. Dilatation in the atrium with chamber hypertrophy
b. Dilatation in the ventricle with chamber hypertrophy
c- atrium dilatation with decrease pressure of contraction
d- ventricle dilatation with decrease pressure of contraction .
57-55 years male with bleeding on examination have external hemorrhoid what to
do
advise him to remove it
do rigid sigmoidscopy
go home and visit after 6 months
do barrium enema
58-infant with high grade fever .. Irritable .. Look sick .. Complain of anuria 4 hour
with multiple petechiea and purpura on body .. He was tachycardic and
hypotensive DX
Renal fauiler
Septic shock
59-Verrryy long scenario of old age pt with DM, HTN, hx of multiple cardiac
attack, CVA, came for routine check up in PHC, u found bilateral opacification in
9

both lenses, with decreasing of visual acuity, u will:


Refer to lazer therapist
refer to cataract surgeon
refer to ophthalmologist
follow up
60-patient on glaucoma medication for weeks came with SOB, cough the cause
a- timolol
b- betoxolol
c- pilocarpin
61-Considerd positive mantux test in
Erythema more than 5 mm in HIV patient
Induration more than 10 mm in diabetic patient
Induratin more than 5 mm in iv drug abuse
Induration more than 10 mm in philpine man
________________________________________________________________
_
62-old with bilateral hydronephrosis:
- Stricture of uretheral meatus
- Prostate enlargement
- Bladder tumor
________________________________________________________________
_______
An 80 year old male presented with dull aching loin pain & interrupted
voiding of urine. BUN and creatinine were increased. US revealed a
bilateral hydronephrosis. What is the most probable Dx?
a) Stricture of the urethra
b) Urinary bladder tumor
c) BPH
d) Pelvic CA
e) Renal stone
________________________________________________________________
__
63-old man healthy felt in collapse before he collapsed there was epigastric
discomfort , came with pain n the back, pulse 114, bp 140Ldx:
Perforated peptic ulcer
Leakage aortic aneurysm
64- in pt with RA to preserve joint function and movement:
-disease modifying medication is sufficient
- fish oil gives subjective improvement
- cold (ice) compression and decrease joint movement
_________________________________________________________________
65-patient with breast cancer and metastasis came complain of tachycardia
hypotension , engorged neck vein and SOB what is most next action
D-dimer
10

Ventlation prefusion scan


Give fursamide and refer to do echo
66- 19 year old athlete, his weight increase 45 pound in last 4 months . in
examination , he is muscular , BP 138/89 . what is the cause
1.alcohol
2. cocaine abuse
3. anabolic steroid use
67-Which antiviral drug causes fever and muscle pain:
Acyclovir
Oseltamivir
Interferon
Gancyclovir
68-5 yr-old baby presented with his parents with pallor his HB is 9, he has microcytic
hypochromic anemia, no other complain .. what u'll do for him ??
iron therapy and close observation
daily multivitamins with iron
________________________________________________________________
_
69- patent with vaginal discharege ,suprabubic pain for 3 days ,fever and bilateral
fornieces tenderness what is the Dx
1-apendicytis
2-acute salpyngitis
3-chronic salpingytis
70- patent is presented with SOB. On Xray he has cardiomegaly and rt. pleural
effusion. pleural aspiration reveals: protein is <30, LDH <200 IU. dx:
CHF.
pneumonia.
TB
hyperproteinemia.
71-pt have mutiple risk factor , obese , HTN not on medication BP 130/90 ,
unhealthy diet, hyperlipediemia, lack exercise which factors control improve
survival:
- Cholesterol, HTN, obesity ???
- Cholesterol, sedentary lifestyle , diet
- Triglyceride, obesity, HTN
- Low HDL, ??, ??
________________________________________________________________
_
72- 44- male old patient has S&S of facial palsy ( LMNL) ; which of the following
correct about it;
A- almost most of the cases start to improve in 2ed weeks
11

b- it need ttt by antibiotic and anti inflammatory


c- contraindicated to give corticosteroid
d- usually about 25 % of the cases has permanent affection
73-pateints with appendicits what is most helpful to make DX
Age
Fever
High WBC
High sedmentation rate
74-Child with high fever and after 2 day develop sorethorate on examination there is
congested thorat and pharynx and white to yellowish papule on erthymatus
base in mouth and lip what is most likly DX
Coxsacki virus
Herps simplix virus

75- Which drug can not be use in acute cholysystits


Naproxen
Morphine
Mepriden
Acetamenophin
Perdoxyphen
_________________________________________________________________
76-pt na 123, k 3 what to do
a-normal saline with 20 meq kcl 80 cc/h
b-normal saline with 5 meq kcl 20cc/h
c-half normal saline with 20 meq kcl 80 cc/h
d-half normal saline with 5 meq kcl 20cc/h
77-female after placement of IUD , she develop abdominal pain and watery brown
discharge ,
1, uterine rupture
2, bacterial vaginosis
3, pelvic inflammatory disease
78- Female with recent hx of IUCD insertion, coming withwatery brownish vaginal
discharge & abdominsl pain what is th most likely dx:
a. Uterine rupture
B. Ovarian torsion
C. Bacterial vaginosis
D. Ectopic pregnancy
NO PID here !!!!!! in alqaseem questions diagnosed as ectopic pregnancy not sure
79- Rx. Of scabies in pregnant women:
permethrin
12

80- Profeational player came with history of truma on the


lateral side of left knee , on examination there is swelling in the
medial aspect of left knee , the diagnosis is :
a- Medial collateral ligament spasm .
b- Lateral collateral ligament spasm .
c- Medial meniscus tear .
d- Lateral meniscus tear
81- Patient he had multiple problem in his chest and he
lives in crowded area what your action:
a- Immunoglobulin
b- H.influnza
c-meningococal
____________________________________________________________________
82- Pt came with cough , wheezing , his chest ascultation revealed monophonic
sound , on xray ther is patchy shadows in the upper lobe+ low volum wirh fibrosis
,, he lives in a crowded place .. What is the injection shuold be given to the
pateint's contacts :
hemophe.influanza type b
Immunoglobuline
Menngioc. Conjugated C
Basil calament .... !!?
83- young male complains of generalized skin lesions and redness, before that
there is a hx of mouth and lips swelling for couple of days
the its denied any hx of traveling or unusual exposure
the is the Dx ?
?? urticarai
coxsackievirus infection
cold urticaria
hot urticaria
84- paient suspected to have connective tissue disease what is most favurable to
SLE
Cystoid body in fundoscopy??
Cavitaion in lung
+ve anti RNP
Sever Ryundoe phenomena
________________________________________________________________
__
85- pt with pycosis on medication developed rigidity and uprolling eyes , afebrile :
- Tradive dyskinesia
- Malignant neuroleptic
- Hypotonic
13

86-What you will find in patient with idiopathic autonomic insuffeciency?


Orthostatic hypotension
Horner syndrome
Anhydrosis
Palpitation
Diaphoresis
87- What is true about hormonal contraception?
Decrease breast Ca
Decrease ovarian Ca
Contraindecated in diabetic women
Increase risk of ectopic pregnancy
88- Patient with colon cancer stage 3 and chemotherapy was prefered so when
do you start it?
As soon as possible
When the lab results normalized
After psychological preperation
No need to start
89-young female complains of 6 weeks amenorrhea and history of VP bleeding
for many days and by laparoscopy the is free fluid in douglas of pouch(I don't
remember the exact NO.) what is the most probable cause??
rupture ectopic pregnancy
90-pregnant woman with UTI which is the best antibiotics to be given if she has
no allergy?
nitrofurantoin
ampicillin
sulfatrimethoprim
tetracyclin
aminoglycoside
91-Pt has a scaly hypopigmented macules on the chest and arms
They seem even lighter under the sunlight,,, what is the ttt? (diagnosis Pityriasis
alba or pityriasis versicolor)
Topical steroid
Na selinum
Topical antibiotics
Oral antibiotics
25 years old male complaining from scaly lesion in his chest , then
become hypopigmented , last 2 months in winter he spend his time neat
to sea, by examination showed hypopigmented lesion over chest & arms
Dx :
14

Vitiligo
taenia versicolor
92- attributable risk factor is :::???
measurement of exposed and not have the disease mius those exposed
and have the disease
93-29 years old male diagnosis as case of gastric ulcer , culture -ve h.pylori
pathology ve of cancer ttt:
proton pumb inhibitor
antihistamine
after 6-8 wk do endoscope after therapy
referral to surgery
94-32 years old with cystic mobile breast mass, no LN enlargement. What would
you do:
Aspiration with cytological evaluation
Reassurance
Fluroscopic biopsy
Mammography then discuss the options according to the new information
________________________________________________________________
__
95-Pt with hix of URTI for 3d then stat to develop rt. Ear pain rinnen test was
negative and wiber test (loud sound in affected side)
Mastoditis
O.M
O.E
96-3 months old baby brought by his parents complaining of abd. distention
bilious vomiting, constipation, the parents informed that the constipation has been
an issue since his birth
what is the single diagnostic investigation to do ??
barium enema
pain xray
??metery
rectal examination
97- A patient presents with long time history of knee pain suggestive of
osteoarthritis. Now he complains of unilateral lower limb swelling and on
examination there is +ve pedal & tibial pitting edema. What is the next
appropriate investigation?
a. CXR
15

b. ECG
c. Echocardiography
d. Duplex ultrasound of lower limb
98- patient came with MI 2 day after addmission develop. Sever
abdominal pain and bloody diarrhea DX
Ischemic colitis
- diffuse abdominal pain , bleeding per rectum and fever 38.3 c ,
preceded by urinary infection 3 weeks back treated with AB ,
diagnosis :
Ischemic colitis
Amoebic colitis
Pseudomembranous colitis

99- looong scenario about old male came with typical history of MI
all of the following can be used in the mX of this its except:
atenolol
heparin
sorry I couldn't remember the rest of the choices :(
100- Pt. with long hix of hyperthyroidism to screen about the complication of
hyperthyroidism do:
liver us
Ct brain
Bone scan (for osteoporosis)
101-to increase absorption of oral iron , give with:
- Vit C
- Vit E
- Zinc
- Ca
- Antacid
102- old, black macule on his back with irregular border and color variation :
- Sq cell carcinoma
- Basal cell carcinoma
- Melanoma
- Acanthic keratosis
103--cord prolapse at level L4-L5 the patient will presented by
a-painful calf muscle
b-absent ankle jerk
c-parethesis of knee joint
d-weak dorsiflextion
104-Old male with acute pancreatitis, (high glucose, low Ca)the appropriate
nutrition:
TPN
16

Regular diet with low sugar


High protein ,high ca , low sugar
Naso-jujenal tube
105- stroke with loss of smell, which lobe is affected :
a- frontal
b-parital
c-occipetal
d-temporal
________________________________________________________________
__
106-Adult P.t recived a vaccine ( i don't remeber the name )
After that he complain of itching , tachycardia and SOB
What is the ttt?
IV hydrocortizone 500 mg
SC epinephrine
107- polycythemia vera ttt:
-Myelosupression
-Plephotomy
-X ray therapy
________________________________________________________________
__
108-CHILD with eczema on 1% hydrocortisone what other medication u can add
Dexamethazon
Cyclosporine
Tacrolimus
109-Pt with heartburn use antiacid for long time but not improve what is
appropriate drug he can use
H2 reseptor antagonist
PPI
..ANTIACID
110-OSTIOMYLITIS start in
Metaphysic
Epiphysis
Dyaphysis
111-Most benign cause of postmenopausal bleeding
Cervical polyp
Atrophic vaginitis
112-Pt came by hx os sudden eye pain burning vision photophobia and by ex.
Small pupil and keretic cell on cornea and cell in humorus
Ttt
Cyclospoine + corticosteroid
113- which of the following is not a feature of normal ECG:
-P wave is the repolarization of the atria

17

114- n normal puerperium..


-lekoria lasts for up to 4 weeks
-the uterus can't be felt after the 1st week in abdomen
epidural analgesia can cause urinary retention
115- best test to detect age of gestation is
-LMP
-U.S.
116-ibuprofen is contraindicated in
-htn
-dm
-peptic ulcer
117-a pt with AF came with black stool (and i think hypotenstion)..dx is:
-ischemic mesntry
118- Female with greenish vaginal discharge, red cervix(srawberry appearance).
under the microscope it was a protozoa..Dx:
a. Trchimoniosis
________________________________________________________________
119- Old man with left lower abdominal pain with fever and constipatin, imaging
showed decreased the fatty shadows around distal colon, your next step:
Double contrast
IV antibiotic
Control diet
120- Child with SCD, about pneumococcal vaccine
- give 23 valent in high risk only
- give heptavalent after 2 yr
- child with high risk give the vaccine along with antibiotics when exposed to
infected ppl )
121- Which of the following increases the quality of the randomized controlled
study & make it stronger:
a. Systemic Assignment predictability by participants
b. Open Allocation
c. Including only the participants who received the full intervention
d. Following at least 50 % of the participants
e. Giving similar intervention to similar groups
________________________________________________________________
__
122- self breast examination:
a. 3-5 day after period
b. 7-10 day ''''''''''''''''''
c. 2weeks ''''''''''''''''
123-right lung anatomy
1. one fissure
2. 7 pulmonary segment
18

3. no relation with azygus vein


4. 2 pulmonary veins
5. no sibson's fascia
124-antidepressant action starts within
- 1 day
- 1 wk
- 2wk
- 3-4 wk
125-child came with generalized body swelling, fever , dark urine with decrease
urine output ,,, what is the most useful investigation for diagnosis:
CBC
Renal function test
Abd. US
Urine sedmintation test
126-a man with 2nd and 1st degree burn over his face and neck
a) wash, cover all burns with Silver sulfadiazine, cover with sterile gauze, give
IVfluid, antibiotic and tetanus toxoid and discharge home with daily dressing
b) cover burn with Silver sulfadiazine, sterile gauze, oral fluid, and discharge
home
c) doesnt make sense
d) doesnt make sense
e) Silver sulfadiazine, sterile gauze, IV fluid and admit to hospital
127-pateint with decrease lipido and weak erection ( or ejaculation)
In investigation prolactin high , LH and FSH normal what is next step
Brain MRI
Abdomen and pelvic CT

Patient with Premature ejaculation + libido + Erectile dysfunction he is


thin and looks sad, he is married for 26years obese and annoying wife, he
came for treatment:
A. Testosterone Injection every one week
B. Sublingual Nitroglycerin 6h before intercourse
C. SSRI
D. ????
__________________________________________________________________________
__________________________
128-4 or 5 ( not sure ) brought by his parents with weight > 95th percentile ,
height < 5th percentile & bowing of both legs what is the appropriate
management :
a- Liver & thyroid function tests
b- Lower limb X-ray
19

c- Pelvis X-ray
d- Thyroid or ( liver not sure ) function test
129-18month old boy came with bite by her brother what you will do ?
A) augmentin
B) titanus toxoid
C) suture
130-old pt with 2 years bone pain , lethargy , fatigue, wedding gait ,
came with table show high calcium and high phosphorus ;
A_ osteoporosis
B_ osteomalacia
C_ paget disease of bone
D_ metastases prostate cancer
E_ paraneoplastic syndrome
131-child with inferior and pain but with normal
movment of knee , no effusion on knee what the important thing to
do ;
A_blood culture
b-ESR
c_ASO titer
d-aspirate from knee joint
d-plain film on thigh
132-miliary TB caractarized by
a- spare lung apical
(b- septal line
c- multiple lung nudules
133-a man who has had MI you will follow the next enzyme
a) CPK
b) ALP
c) AST
d) Amylase
134-a child of parents who have TB, PPD test done for him and revealed 10 cm
induration, this is
strong +ve.
intermediate +ve.
weak +ve.
-ve
135-in aspirin overdose:
a) liver enzyme will peak within 3-4 hr
b) first signs include peripheral neuropathy and loss of reflexes
c) 150 mg/kg of aspirin will not result in aspirin toxicity
136-female pregnant has HIV +ve , what is the most accurate information to tell
her about risk of transmition to baby ;
A-likely transmtion through placenta
20

b-through blood cord


c-hand contamion of mother
d-by breast feeding
137-Female with dysurea, urgency and small amount of urine passed .. she
received several courses of AB over the last months but no improvement .. all
investigations done urine analysis and culture with cbc are normal .. you should
consider:
a) interstitial cystitis
b) DM
c) Cervical erosion
d) Candida albicans
138-patient is presented with acute chlangitis, what you will do to alleviate the
symptoms:
IV antibiotics + gastric lavage.
IV antibiotics + drainage of bile.
hydration + chlolecystectomy.
139-Which of the following drugs prolongs QT interval as side effect?
a. Respridone
b. Clozaopine
c. Amisulpride
d. Aripiprazole
e. Ziprasidone
140-Which of the following is a 3-hydroxy-3-methyl-glutaryl-CoA reductase (HMG
CoA)?
a. Statin
b. Fenofibrate
c. Niacin
141-picture of bulls in food ... In biosy there is epidermal lysis and on
immunoflurescen: deposition of IgG DX
Bulls pemphigoid
Pemphigoid valgarius
________________________________________________________________________
____________________
142-perthes disease all except
Can be presented with painless limp
It always unilateral
-how to calculate -143relative risk
144-- paient complain of infirtility 6 year ago and sever pain with cycle
( dysmeanorhea) DX
Endometriosis
Pelvic congestion
Endometritis
21

________________________________________________________________________
_________________
145-you want to give varcilla vaccine in one no have vaccin before

Two dose and 6 weak between


146-- pregnant not vaccinated against measls and mumps and rublla ..
She exposed to rublla 3 day ago what you do
No treatment
Immunoglobin
Tell her no affected on her pregnancy if she take the vaccine
147- Pateint complian of diplopia , weakness , and frequant aspiration
pnumonia in last 2 month ... In examination there is spascity and
fasciculation DX
Mythenia gravis
Mythenia syndrome
Motor neuron disease
__________________________________________________________________
148-child with low grade fever , sore thorat in examination there is
lymph node enlarment but not tender and no exudate on phrynx DX
It is most likly streptococcal than viral
It is viral more than bactrial
Most likly EBV
149-what is the. symptom Most likly occure with hiatus hernia
Skin pigmentation
The symptom increase with pregnancy
150- patient with bilateral eye redness . Discharge and tearing on
examination cornea , lens all normal Nd tere is conactival follicle DX
Acute conjunctivitis
151- child C/O fever , sore thorat all examination was normal What is
the ttt :
Cefruxime
Ceftriaxone
Give paracetamol and take pharynx swab
_____________________________________________________________________
152-- pateint C/o ictrus in skin and eye on investigation WBC 2500
plt 70,000 HG 7 lekocytosis 17% total bilirubin 51 and direct bilrubin 12
what is the test most likly positve
+ve coomb's test
In US obestructive billiary duct
22

antiparietal antibodies
____________________________________________________________________
153-
...
Because the organsim develop resistant
Develop new antigenic drift
154- patient c/o low self steam and fatigue .. Lack of intersted and
concentration loss of sleaping , depressed mood for last 2 years what
DX
Dysthymic
155--lacteting mother complain of fever and breast tenderness and
redness diagnosed as bactrial mastitis what is ttt :
Continoue breastfeeding and hot compresser and antibiotic
Discontinue breast feeding and give antibiotic to mother and baby
156-the most common cause of nipple discharge in non lactating
women is ;
a-prolactenoma
b-hypothyroidism
c- breast CA
d-fibrocystic disease with ductal ectesia .
e ductal papiloma
157-with patient has fear ,SOB ,sweating when he is in automobile
DX
a-specific phobia
b-panic disorder
c-generalize anxiety disorder
________________________________________________________________________
_________________
158-which of the following causes the highest maternal mortality in pregnancy
a. toxoplasma
b. hyperbilirubenia
c. pheochromocytoma
d.rubella
159-Old pt presented with abdominal pain, back pain, pulsatile abdomen what's
the step to confirm dx:
a. Abdominal US
b. Abdominal CT
c. Abdominal MRI
23

________________________________________________________________
_
160-The most common cause for chronic irregular rectal bleeding is:
Diverticulitis
Hemorrohids
Colon cancer
UC
161-the most common cause of excessive day time sleepiness is
1- circadian rhythm
2nacrolepsy
3-sleep apnea
162-the most common malignant tumor of parotid in childern
1-acinic cell ca
2- mucoepidermoid ca
3- adenocarcinoma
163- The useful exercise for osteoarthritis in old age to maintain muscle
strength and bone density
Low resistance, high repetition muscle training
Conditioning, low repetition muscle training ??
Walking and endurance muscle training
Low resistance and conditioning muscle training
164-pregnant with uterine fibroid , has no symptoms only abd. Pain ,
US showed live fetus ,,,,, What is the appropriate action to do:
Myomectomy
Hysteroectomy
Pain management
Pregnancy termination
165-Pt came with eye pain, watery discharge and light sinsitivity
Eye examination showed corneal ulceration. Her symptoms are
frequently repeated . Which of the folowing is triggring for recurrence
of her symptoms:
Dusts
Hypertension and hyperglycemia
Dark and driving at night
Ultraviolet light and stres
166-p.t taking a medication , came to the ER suspecting she has
overdose of her medication, her symptoms ( convulsion, dilated pupil,
hyperreflexia and strabismus) the medication is:
TCA
SSRI
Hypervitaminosis
___________________________________________________________
167-Pt complain of hearing voices from the microwave and refrigerator
24

Visual hallucination
Auditory hallucination
168- Old retired man having ansomnia only . Has no symptoms related
to anxity or depression .. U will give him :
Diazepam
If zolpidem is in choices it is more accurete
169-pt take cephalexin after tooth extraction for days
After that he develop profusre , green foul smilling diarrhea with low
grad fever . He has tachycardia and mild abdominal dist. Sigmoioscopy
showed white mucosal patches , what is the most ttt for this condition?
Clarythromycine
Vancomycine
Cephalosporine
Lineozides
170-pt with COPD came with couph , wheezing and greenish sputum
The causative organism:
H.influanza
Strep.pneumonia
Chlaymedia
Mycoplasma pneum.
171-what is the most effective measure to limting the complications in
COPD:
Pnumococcal vaccination
Smoking cescation
172-25 years old female came complaining of difficult hearing , she
mentioned that their a family history of early oncet hearing loss ( her
grandmother)
Oto. Exam was normal .. Weber and rinne tests result in ( bone
conduction is greater than air conduction ) ... Next action is :
Refer her for aid hearing
Tell her there is no avalible ttt
Refer her to otolaryngologist
________________________________________________________
173-Old man came complaing of progressive hearing loss , it is mostly
profounded when he listining to the radio, he does not has any
symptoms like that before
Weber and rinne tests result in bilateral sensorineural hearig loss..
Diagnosis:
meniere's disease
Otoscelerosis
Noise induced deffnese
Hereditary hearing loss
25

174-25 y wear glass 10 y and diagnosis DM type 2 when u do eye


screen 4 her:
6m
12m
2y
5y
175-Pregnant women has fibroid with of the following is True:
Presented with severe anemia
Likely to regress after delivery
Surgery immediately
d. Presented with Antepartum Hemorrhage
176-70y male with osteoporosis the T score of bone density would be:
-3.5
-2
1
2
3.5
177-Pt G3 P3 all her deliveries were normal except after the second
one she did D&C for retained placental parts, presented with
amenorrhea after a period of irregular cycle, labs all normal except :
high FSH, high LH, low estrogen DX:
a- Asherman syndrome
b- Ovarian failure
c- Sheehan syndrome
d- Turner $
178-The most dangerous red eye that need urgent referral to
ophthalmologist:
1.associated with itching
2. presence of mucopurulant discharge
3.bilateral
4.associated with photophobia
179-Neonate with mucopurulant eye discharge lid swelling and culture
positive for gm ve diplococcic , treatment (neonatal gonococcal
conjunctivitis)
1.intravenous cephalosporin
2.topical sulfonide
3.oral floroquinolol
26

4. IM aminoglycoside
180-a baby with blood in the stool and bought of crying and x ray
shows obstructive pattern.. looks like intussusception you will do:
a) surgery
b) Barium enema
c) observation
d) giv e IV fluids and let obstruction solve itself
_________________________________________________
181-pt with nasal congestion, watery nasal discharge and
conjunctivitis, ttt:
a- oral antihistamine
b- Na cromoglycate
c- Topical steroid
d- ??
182-initial treatment of OA in adult who has knee pain bilaterally:
a- Opoid
b- Intraarticular steroid
c- Quadriceps strengthening exercise
d- ??
183-IV drug user has macular rash on palms ,splinter he, and
ophthalmoscope shows macules with clear center in retina DDx:
Syphilis
Infective endocarditis

184-patient with red eyes for one day with watery discharge
No itching or pain or trauma (nothing indicate allergy or bacterial infection)there is
conjuctival injection
visual acuity 20/20
what is next management
antihistamines
topical AB
No further management is needed
refer to ophthalmologist
topical steroids
if allergic rhinitis :topical steroid
second line:antihistamine
185-newborn apgar score 3 (cyanotic, limp, decrease breathing, HR
less than 60) your action:
- Volume expansion
- Chest expansion
27

- Ventilation
- Bicarbonate
186- pt presented with sweating, myosis, and garlic breath odor:
- Organophosphorus toxicity
- Cyanid toxicity
- Alchol
- DKA
- Cocaine toxicity
187-in rheumatic fever:
- Bacteria in blood
- Bacteria lodge in myometrium
- Skin invasion
- ???
188-86- female G3P0 , c/o infertility , have regular non heavy cycle,
trichomonus infection treated at age of 17 , previous 3 elective D/C in
first month gestation ,DDx:
- Asherman $
- Sheehan $
- Endometritis
- ???
189-40 yr heavy and intercyclical bleeding , not pregnant , does not on
OCP:
- Anovulatory cycle
- ????
190- smoker , CXR shows lung mass, hyponatremia and diluted urine:
- Heart failure
- SIADH
- Renal failure
- Conn's disease
191-common cause of AOM in all age groups:
- H influenza
- St. pneumonae
- ???
192-old, which fracture caused by trauma on outstreatched hand:
colle's Fx
193- female, malodor vaginal discharge, dysuria, normal urinalysis,
leukocyte and gram -ve diplococci :
- N gonerrhea
194- old, black macule on his back with irregular border and color
variation :
- Sq cell carcinoma
- Basal cell carcinoma
- Melanoma
28

- Acanthic keratosis
195osteoporosis risk

65 75 80
According to above graph:
- 18 % develop osteoporosis after age of 80
- 80 % of elderly have osteoporosis
- Age directly related to risk of osteoporosis
- Pt after 80 at high risk of osteoporosis
196- upper limb HTN , decrease lower extremities pulsation:
- Coarcutation of aorta
197- pain and swelling at first metatarsophalyngeal joint:
- Na urate crystals
- Ca phosphate crystals
- ???
198- pain and swelling at first metatarsophalyngeal joint:
- Na urate crystals
- Ca phosphate crystals
- ???
199- old, smoker , rectal bleeding , wt loss: >>>>Colorectal cancer
200- 45 years old female came to ER with acutely swollen knee +
ballotment patella .. The most important to do is:
MRI of the knee
Aspiration
Complete blood count
Rhumatoid factor
201- Pt came with a history of about 12 dayes duration severly red ,
swollen painful first metatarsophalangeal joint.. He is hypertensive with
inverted T wave on ECG ... The most appropriate meaure for diagnosis:
CBC
Uric acid level
Troponin level
C-reactive protein
202-Pregnant on iron supplementation throughout her pregnancy for
her anemia , now she come complaining of weakness and easy
fatigability
Her Hemoglubin 7 , MCV 60 .... What is the diagnosis?
Iron def. Anemia
Hypothyrodism
Vit B12 def.
Beta thalassemia
29

203-baby who can name 4 colors .... His Age :


48 months ( 4 years )

204-Pt came after fight ( gunshot ) there is a pice of the omentum
coming out from the wound . Vital signs ( HR 98 , BP 130/80, RR 18 ) ..
What is the best action to do ?
CT
DPL
Fast us
Wound exploration
Scheduled laprotomy
205- about which breast mass present with bloody discharge ?
intraductal papilloma
______________________________________________________
206- Most Dangerouse sign during pregnancy?
Vaginal bleeding
207- the most common cause of epistaxis in children is:
Nasal polyps
Self induced
208-one of the folowing manifest. As croup:
Forigne body
Pneumonia
Common cold
Asthma
209- clear scenario of varicocele ( bag of worms scrotum )
210- scenario of glucoma in old pt ,, what is the best ttt?
Acetazolamide + pilocarpine ( sure 100% )
211- clear scenario of keratitis .. on examination there is dendritic
ulcer:
Herpes simplex keratitis
_______________________________________________________
212-Mass in the upper back .. with punctum and releasing white frothy
material
a- It's likely to be infected and Antibiotic must be given before anything
b- Steroid will decrease its size
c- It can be treated with cryotherapy
d- It must be removed as a whole to keep the dermis intact
213-Drug use in CHF with systolic dysfunction?
Nifidepine*
deltiazm*
and two drugs from ACEI I forget their names *
the 5th choice is one of B blocker*
30

214-A patient with severe headache, behind the eye, 4 times in one week ? with
other symptoms
(i don't remember the whole scenario, it seems a cluster headache case, not
sure)
which drug is not useful in prophylaxis:
CCB (may be verapamil)
methysergide
valium
lithium
prednisolone
read about the prophylaxis of Migraine and Cluster
and if any one remember the scenario please write it here
215-Pt diabetic he has wound in his leg with poor healing , Exudate ,no sign of
inflammation the hyperglycemia cause poor wound healing by :
a- inhibit phagocytosis
B-stimulate bacterial growth
c-decrease immunity

216-wt is non hormonal drug use to decrease hot flush in postmenuposal


women:
paroxitine
_________________________________________________________________
217- old pt complain of dull hip pain increase after walking and activity and it
make pt wake up from sleep many time and(several hours) morning stifness
whate is the mos DX
a-osteoarthritis
b-osteomlitis
c-osteoprosis
e-depression
_________________________________________________________________
218- csf examination show high igG and anbnormal band cell on agarose gel
electrophoresis whate is the DX
a-muscular dystrophy
b- multiple sclerosis
________________________________________________________
219-senario of pt e GER since 10 years endoscopy done reveal low-grade
dysplasia of lower oesophagus whate is the next step
1-esophageal resection
2-fundoplication
3-rescreening in (i forget the time)
the other choises I dont remember please read about barrot esophegous
31

220-perinatal mortality
A-include all stillbirth after the 20th wk of pregnancy
B- include all neonatal deaths in the firist 8wk of life
c-in clude all stillbirth and firist wk of neonatal deaths
d-is usually death per 10,000 live birth

221-pregnant 41 wk on complete biophysical profile oligohydraminous is found


whate is next step
a-induction of labor
222-female came wuth vulval irritation the doctor told her to stop using bubble
bath she stopped it but the irritation continues 0n examination the vulva skin was
waxy like and specked appearance the dx is
1- psoriasis
2- atopic dermitites
3- contact dermitits
4- lichen planus
223-the mechanism of action of propylthiouracil is
inhibits the enzyme thyroperoxidase
224- pt e hx of erythema and vesicle in the forehead but not affect the vision
whate is the best managment
1-oral acyclovire and F/U
2-oral acyclovire and opthalmologist refere
225- (Picture of a huge ulcer in the leg, the ulcer is red with raised edges)). Best
option of management:
-Topical steroids
- Biopsy
- Radiotherapy
- Topical antibiotics
226-- pateint felt fatigue, SOB , angina like pain after doing excercise
there is no thrombus done by cath. , he had aortic valve stenosis with
less than ,7 cm what would u do :
a- avoid exersion
b- aortic valve replacement
c- ttt with medication only
227-22y o female there is no breast development , amenorrhea , deep
voice O-E mass over the ovary Dxx:
a-thecoma
b- germ cell tumor
c- lyding cell tumor
d- ovarri storma
_________________________________________________________________
32

228- pt with CHF and atrial fibrillation u add digoxin what is the effect
of it in this case :
a- decrease ventricular effecacy
b- unchanged COP
C- decrease HF
229-infant swallow coeeosive material came within half an hour to ER
drooling, crying what is the initial thing to do
activated charcoal
endoscopy
secure airway
2 cups of milk
230-svt ttt:
Digoxin
adenosine
231- malaria in a child:
a- crescent shape gametocyte of vivex is diagnostic in the stool
b- the immediate ttt primquine for 3 d
c- 72h tt t of malaria is suffeceint
d- the most common cause is falciparum
232-scaly purpule lesions in the face of a child the cause
a- staf. Aureus
b- beta haemolytic srept.coci
c- H. influenza
233- child >90% of the normal . < persentile hight with sever bowing of
legs what help u for diagnosis: (same question mentioned but different
choices)
a- lower extremeties x-ray
b- pelvic x-ray
c- cbc
d- alkaline phosphatase
234- ttt PE:
a- iv heparin
b- iv tpa
c- streptokinase
235- The most common side effect of long use of systemic
corticosteroids:
a.Asthma
b.Weakness in pelvic muscles
c osteoporosis
236-pt taking digitalis he developed sudden disturbance in vision yellow
discoloration and light flashes (thats what I remember from the question)
a.digitalis toxicity
b.retinal detachment
33

237- Pt has carotid bruit with occlusion 60% of the left carotid artery what
well u advice the pt
a.Asprin daily
b.Angiograpy
c. endartectomy
238-) What is the most specific test for syphilis:
a.TPI
b.FAAT
treponema antibody absorption test
239-) pt had history of hypertension and no medication taken he eats a
lot of meat with no fruit and vegetables on examination he was obese
BP:130/98 investigations she high cholesterol ,high trigelcride, low HDL
in which category u well put the pt for risk of IHD:
A
B
C
d
I forgot

I forgot

High
cholesterol

High
cholesterol

Sedentary life obese


High BP

240- pt came with PND and orthopnea an examination he has bilateral


basal crepitation and pulmonary edema what is the diagnosis:
a.left heart failure
b.right heart failure
241-Likelihood ratio of a disease incidence is 0.3, mean:
1.large increase
2. small increase
3. no change
4. small decrease
5.large decrease
242- I study done on 10,000 people for about 3 years in the beginning of
the study 3,000 developed the disease and 1,000 on the end of the study
what is the incidence:
a. 10.3
b. 12.5
c. 30
243- the most useful test to detect early pregnancy:
a.urine pregnancy test (my answer) there was no serum BHCG in the
chooses
b.ultrasound
34

244- Pregnant lady which is hypertensive regarding methyldopa what well


u tell her
Methyl dopa better then lisnopril
(I couldnt remember the other chooses)
245-treatment of gonrehea:
Ceftriaxone
246-44 lady has previous history of DVT her husband doesnt want to use
condom what well u advice her:
a.OCP doesnt increase the risk.
b.IUD is preferred in this case.
c.she is unlikely to become pregnant
247-Pt covers the tv because he says that they see hem and well split on
his face diagnosis:
a.SCZ
248-A man has excessive worry form germs on his hand
a. Specific phobia
b.Agrophopia
c.OCD
249-Scenario about premenstrual dysphoric disorder. (straight forward
and they asked
about the diagnosis
250-regarding group Astrept. Infection have lead to rhematic fever :
a.blood dissemination
b.by causing pharyngitis, tonsillitis.
c.joint invasion.
d.affect skin.
e-reach endocardium
251- Child with positive gower sign which is the most diagnostic test :
a.Muscle biopsy
252- child has allergy to dust what well u advice the family
a.keep humidity of the house about..
b.cover his pillow with.
c.clean his clothe with warm water??
253-child with congenital; heart disease his parents doesnt know the
name of the disease he has peripheral and central cyanosis:
a.PDA
b.tetrolgy of fallot.
c.VSD
D.left ventricular hypoplasia.
25435

Treatment of papillary thyroid cancer: (read about it)


a. radioactive iodine uptake scan
a. surgery
255- picture of herpes zoster (the same picture)

256-15y boy appear patch in rt lower leg these patch is clear center , red
in peripheral, no fever no other complain so diagnosis (there was a
picture with lesion in the groin area)
a-contact dermatitis
b-tinea corpora
c- lyme disease
d-psiorosis

_________________________________________________________________
257- Man is complaining that he doesnt see the traffic signs well what is
the best way to measure the distance vision:
Snellin chart
__________________________________________________________________________
___
258- question about pt had pterygium what well you tell the pt:
-it is malignant
- needs surgery
in another word :regarding ptergium :
It indicate systemic disease
Will cause loss of vision
Treatment is surgery
I forgot the other choices read about the topic
259- Compelete loss of vision Lt eye, in pt with recent infarction?
a) Frontal
b) Parital
c) Cortex
d) Occipital
36

260- the commonest initial manifestation of increased ICP in patient after head
trauma is
1. Change in level of consciousness
2. ipsilateral pupilary dilatation
3. contralateral pupilary dilatation
4. hemiparesis
5. hypertension
261- Most accurate test for CTS ? carpal tennel syndrome
a) Tinels test
b) Phalens test
c) Nerve tapping numbness
There was no nerve conduction velocity.
262- Hx of Child has itching in his RT hand which increase in the night
described as linear fissures at their top there is blacknish
Scabias
263- Which of the following method is rapid and best for complete
gastric evacuation ? !!!! ...
a) G lavage
b) Manual induce V
c) Syrupe
d) Active charcoal
264- old male pt with hx of IHD, DM , HTN , dyslipidemia ,
family hx of heart disease ,,,,
lab showed :- LDL : 199 ,,,, HDL : 37
so , in this pt what is most dangerous Risk factor ?
a) Increased LDL
b) Decreased HDL
265- Old man psych pt , has halosination , aggressive bebaviour ,loss
of memory ,Living without care , urinate on him self , what is next step
to do for him ?
a) Give antipsychotic
B) Admit him at care center for elderly .
266-60 y/o male known to have ( BPH) digital rectal examination shows
soft prostate with multiple nodularity & no hard masses , the pt
request for ( PSA) for screening for prostatic ca what will you do ?
a)Sit with the pt to discuss the cons & rods in PSA test
b)Do trans-rectal US because it is better than PSA in detection
c) Do multiple biopsies for different sites to detect prostatic ca
267- Female com with lump in breast, which one of the following make
you leave him without appointment ?
a) Cystic lesion with serous fluid that not refill again
b) Blood on aspiration
c) Solid
37

d) Fibrocystic change on histological examination


268- Infant newly giving cow milk in 9 months old , closed posterior
fontanel, open anterior fontanel with recurrent wheezing and cough ,
sputum examination reveal hemoptesis , x-ray show lung infiltration ,
what is your action ?
a) diet free milk
b)corticosteroid
c) antibiotics
heiner syndrome (milk induced pulmonary disease in infants) is a food
hypersensitivity pulmonary disease that affects primarily infants.
269-In a certine study they are selecting the 10th family in each
group,ahat is the type of study: imp.
systemic study
non randomized study
stratified study
In statistics, stratified sampling is a method of sampling from a
population.
When populations vary, it is advantageous to sample each
subpopulation (stratum) independently. Stratification is the
process of members of the population into homogeneous
subgroups before sampling.
__________________________________________________________________________
__________________________
270-delusions definition:
271-giardiasis treatment:
metronidazole
272- TTT of refractory hiccups?
Chlorpromazine
273- TTT of miagrine?
Sumatriptan
If BB is
274-most common psychiatric condition come with mania ?
paranoid
grandiosity
275-patient with fever and fatigue prior to develop maculopapular then
vesicle and pustule ?
HSV1
HSV2
Varicella
38

276- postmenopausal women at high risk of:


osteoporosis
277- why SSRI best TTT?
Effective and tolerable
278-compliance of prophylactive antiasthmatic drugs important to
reduce
airway inflammation
reduce esinophil
279-2 months amnorrhea refuse examination because she is tense and
anxious what will do for her :
FSH and LH
US pelvis
280-60 years old patient has only HTN best drug to start with:
ACEI
ARB
diuretics
beta blocker
alpha blocker
__________________________________________________________________________
__________________
281- structure normally not palpable?
LN
282-COPD pt not responding to bronchiodilator what well u add for hem :
a.aminophiyline
b.methylpredinselone
283-female complain of colourless itching vagina ,her partener complain
of uretheral discharge ,cervical examination shows strawberry spots treat
with:
mecanozole cream
estrogen cream
progesterone cream
douch
284-four year old child complain of bleeding from 4 months ,intermittent
painless bowel movement good appetite after examination the examiner
found blood on his digits:
uc
mickels diverticulitis
juvenile polyps
39

__________________________________________________________________________
_______________
285-high grade fever, rigors ,painful hepatosplenomegaly:
malaria
visceral leshmania
toxoplasmosis
286- appropriate way to prevent spread of disease :
change human behavior
screening tests
287- first few words:
12 months
18 months
36 months
288-which of the following true about headache :"
-increase ICP at last of day
-normal CT may exclude subarachnoid hemorrhage
-amnursus fugax never come with temporal arteritis .
- neurological sign may exclude migraine
289- patient with typical sign of infections mononucleosis come with
abdominal pain and hypotension next step :
-abdominal CT and IV fluid
-antibiotic and IV fluid and and observation
290- patient with hypersensitivity skin at back take paracetamol and
develop vesicle at back extend to abdomen Dx :
Herpes zoster
291- picture of viral warts
292-pt in burn wll die due to :
-smoke inhalation
- trauma
293- elderly patient bedridden for long time what will you do :
-include family support
-IV valum
294- pt with dysphagia , weakness ,fasciculation ..:
-motor neuron disease
-polyneuropathy
295-Young male c/o pleurisy pain at rt side On EX there is only decrease
breath sound
tachypnia other wise normal and there is CXR I dont know if it is normal
or not But it seems to me normal what will you do?
a-discharge pt bez it is only viral plurzy
b-discharge him on Augmentine
C- I think refer him to pulmonologist
40

296- aseptic meningitis early will found:


a-lymphocytosis
I can't remember the other choices sorry but you must read about the
CSF analysis in aseptic meningitis and which cells present
297-Kernig's sign:Definition:
298- Diagnosting peritoneal lavage positive when
1000 RBC
50WBC

299- attributable Risk definition:
300- female with irregular cycle month and absent for two month with
heavy bleeding:
a-metroohaia
b-menorraghe
c-menometrogia
d-polymenorrhagia
301-8month complaining of gastroenteritis loss of skin truger, sunken eye
depressed anterior fontanel his dehydration is:
10%
20%
5%
302-Patient admitted as a case of emphysema, according to the vaccine what you
will do
a)give pneumococcal vaccine now
b)give flu vaccine now
c)give all vaccine 2week after discharge
d)give flu vaccine now and pneumococcal vaccine 4week after discharge
303-17 years male while play football felt in his knee (turn over ) what injury
medial meniscus lig
lsteral meniscus lig
medial collateral lig
lat collateral lig
anterior crussate lig
304-pt. complain of joint. Stiffness, and high ESR, CRP:
- Inflammatory condition of the soft tissue.
- Immune complex deposite.
305-patient has tangential thought, circumstantial, what is the type of this condition:

41

- Form
- content
306-pigmentation of OCP called:
melasma
307-old man did femoral popliteal bypass has 2 days of forgetting :
alzhiemer
vascular alzhiemer
308-clear scenario of turner syndrome (read about features of turner )
309- cause of death in inflamed burns :
-injuries
-inhalation of smoking
In flame burn , the most common cause of immediate death
1.hypovolemic shoke
2. septic shoke
3. anemia and hypoalbumin
4. smoke inhalation
5. associated injury
310-Atrial fibrillation +narrow complex+unstable pt (hypotensive):
cardioversion
311-epdiomology definition:
312-epdimology curve:
graphic registration of disease through a period of time
other choises :
a- Plotting number of cases on time line
b- Geographical places
c- Case with similar diagnosis
313-question in digoxin toxicity (read about):
314-child was playing and felt in the toy, his leg rapped and twisted he
dont want to walk since yesterday:
- ankle tissue swelling
- spiral tibial fracture 100%
- chip tibial fracture
- femur neck of the tibia freacture
42

315-which medication increase survival in COPD pt. :


a-b agonist inhaler
b-corticosteroid inh.
c-oral corticosteroid
D- continuous oxygen
__________________________________________________________________________
__________________________
316- How much Na in 0.9 normal saline ?
a- 30
b- 75
c- 90
d- 155
__________________________________________________________________________
__________________________
317-- Female with yeast vaginal discharge the treatment is:
a. Meconazole cream for 7 days
b. Fluconazole orally for one day
c. Metronisazole orally for 7 days
318-- Secondary dysmenorrhea is:
a) rare due to anovulation.
b) due to gonadal agenesis
c) always pathological
d) part of sheahan syndrome my answer
319- for with aggressive patient with rheumatoid arthritis:
Methotaxtrate my answer
320- Greatest thing to prevent disease
Genetic consulting
Immunization
Prevent environments ..
Personal behavior??
321- PT WITH POLYCYTHMIA VERA COMPLIN OF GENRALIZED PRURITTUS
AFTER BATHING THE CAUSE IS :
due to abnormal histamine release
322-- unwanted effect of antichlnrgic drugs :
a. Diarrhea
b. Urine incontince
c. Decrease intraocular pressure
d. Blurred vision
323-question about stratified simple study:
324- Rubella Incubation Period
14 to 21 days sure
325-De Quervain Thyroiditis histopathology :
43

the classic changes of granulomatous thyroiditis develop. This is


characterized by aggregations of lymphocytes, large histiocytes, and
plasma cells among damaged thyroid follicles. Multinucleated giant cells
enclose pools or fragments of colloid, from which stems the designation
giant cell thyroiditis.
__________________________________________________________________________
__________________________
326- Nodular sclerosis Hodgkin disease
In nodular sclerosis Hodgkin disease (NSHD), which constitutes 60-80% of
all cases of Hodgkin lymphoma, the morphology shows a nodular pattern.
Broad bands of fibrosis divide the node into nodules. The capsule is
thickened. The characteristic cell is the lacunar-type Reed-Sternberg cell,
which has a monolobated or multilobated nucleus, a small nucleolus, and
abundant pale cytoplasm.
327- Thyroid cancer associated with:
Euothyroid
Hyper
Hypo
graves
328- patient with recurrent pneumonia and productive cough , foul
smelling sputum increase with lying down + clubbing:
bronchectasis
BA
Pneumonia
329-) flu like sx since to days and now has red eye ( pic ) Dx:

Viralconjunctivitis / bacterial conjunctivitis

/ uvitis / glaucoma

330- young pt came to ER with dyspnia and productive tinged blood


frothy sputum , he is known case of rheumatic heart dz , AF and his
cheeks has dusky rash dx :
Mitral stenosis
CHF
endocarditis
331- You r supposed to keep a child NPO he's 25 kgs, how much you will
give for maintenance >> 1600 ml .
44

First 10 kg X 100ml >> 1000 ml


Second 10 kg X 50ml >> 500ml
Third 5 kg X 20 ml >> 100 ml
Total = 1600 ml
332-) old pt take hypercalcemic drugs and developed gout what is
responsible drugs >> frosamide
thiazide
333- In pt with moderately sever acne valgarus best ttt
Oral isotretinoin
topical Retinoids
Topical clindamycin
oral antibiotics
334- which of the following TTT contraindication in asthmatic pt :
Non-selective B blocker
335- case with 60 years old male with RT upper quadrant pain after
dinner , most likely DX gallstone ;
What is most appropriate inx to DX gall stone ?
US
Xray
Barium
336- human bite to hand , most common hand position that proposed to
infection ?
Clenched hand
dependent
extended thump
extended fingers
337- In chlamedia infection ttt is ? ( not mention pregnant or not )
Doxcycline
Azithromycine
Metroniadizole
( also, doxcy used )
338- case cord like cheesy white adherent odour less vagina after use of
antibiotic DX >>Candidiasis
339-)- malaria case , beside antiobtic how to prevent ?
Kill the vector
In another way :
What is the most important prevention measures to be taken in the
outbreak of malaria:
a. Clothing disinfected & inspect for insect bite
45

b. Clothing disinfected & prevent insect bite


c. Eradicate the vector & inspect for insect bite
d-Eradicate the vector & prevent the insect bite
340- Positive predicitive value : Definition ?
" pt who has high Risk factor & +ev test "
_________________________________________________________________________
341- Most difficult method to prevented in transmission:
Person to person / Vector / Droplet /Air flow
342- old pt, bedridden , with bactermia , organism is enterococcus
fecalis , what the source of infection:
UTI
GIT
pneumonia
bed sores
343-) 4y girl, decrease head growth, decrease social intraction, decrease
in language etc:
Rett's syndrome
344- case of Raynaud's phenomenon it was direct >> pallor then cyanotic
then red finger without other clinical features .
__________________________________________________________________________
345- read about rebound hyperglycemia in DM ?? somogi and down
phenomenon
346- During heart contraction,heart receive more blood by:
_ coronary artery dilatation??
_ IVC dilatation
_ pulmonary vein constriction
347- Pt. with 1st and 2nd degree burn involving face and neck:
All choices with no hospital admission except one which I choosed as the
burn involves the face
348- In patient with rheumatoid arthritis:
_ cold app. Over joint is good
_ bed rest is the best
_ exercise will decrease postinflammatory contractures
349- Pt after swimming pool(clear Dx of otaitis externa) Rx:
_ nothing
_ amphotericin B
_ steroid
_ ciprofloxacin drops
46

350- Patient with continous seizures for 35 min. despite taking 20 mg Iv


diazepam..what to do??
_ give 40 mg IV diazepam
_ give IV phenytoin
_ give IV Phenobarbital
351-16 wk pregnant not known to have illness before has high BP..DX:
_ preeclampsia
_ chronic HTN
_ gestational HTN
35 years prime 16 wk gestation PMH coming for her 1st cheek up she is
excited about her pregnancy no hx of any previous disease.
Her B/P after since rest 160/100 after one wk her B/P is 154/96
Most likely diagnosis :
a- Pre eclempsia
b- Chronic HTN
c- Lable HTN
d- Chronic HPT with superimposed pre eclampsia
e- Transit HPT
352- y/o with mild epigastric pain and nausea for 6
months..endoscopy>lossof rugeal folds, biopsy> infiltration of B
lymphocytes..treated with abx..cause:
_ salmonella
_ H.pylori??
353- Young suddenly develops ear pain, facial dropping..what to do:
_ mostly will resolve spontaneously
_ 25% will have permenant paralysis
_ no role of steroids
in another word: male old patient has S&S of facial palsy ( LMNL) ; which of the
following correct about it ;
A- almost most of the cases start to improve in 2ed weeks
b- it need ttt by antibiotic and anti viral
c- contraindicated to give corticosteroid
d- usually about 25 % of the cases has permanent affection
354-2 month infant with white plenched papules in the face what to do:
_ reassurance
_ topical steroids
_ abx
355- Recurrent watery discharge of eye, pain, sensitivity to light..on
exam.> inflammation,ulceration of eye..cause:
47

_ dust&pollens
_u/v light
_ stress
_ night accommodation
356- Patient with ARDS on ventilation developed pnemothorax..cause:
_ -ve pressure ventilation
_ central line
_ 100% O2
357- Lactational mastitis..Rx:
_ doxycycline
_ ciprofloxacin
_ ceftriaxon
_ gentamyecin
_ cephalexin
358- OCP that causes hyperkalemia:
cant remember the choices.
drospirenone
359- All are 1ry prevention of anemia except:
_ health education about food rish in iron
_ iron fortified food in childhood
_ limitation of cow milk before 12 month of age
_ genetic screening for hereditary anemia??
360-+ve leichman test:
ACL injury
361- Waking up from sleep..cant talk, no fever, can cough, normal vocal
cordsDx:
Functional aphonia
362- Patient with CML taking imatinib mesylate and odansetron for
nausea and vomiting presented with tachycardia,fever
Diphoresis and hyperreflexia Dx:
_ neuroleptic malignant syndrome
_ imatinib toxicity
_ odansetron toxicity
363- What is the most effective method to prevent the brucellosis
infection:
a. Treat the infected people
b. Immunize the farmers & those who deal with the animals
c. Get rid of all the infected animals
d. Pastralization of the diary products

364- PTS with history of infertility the first line of investigation for this
couple is >>>
semen analysis
48

365- PTS with depression manifestations , what is the mechanism of the


drug that you will prescribe >>>>
increase availability of serotonin
366- women 52 year complaint of hot flush , dry vagina, loss of libido ,
loss of concentration , wt gain since hot flush , affect marital state >>>>
estrogen
Progesterone
fluxatine
__________________________________________________________________________
__________________________
367- Old pts with history of bilateral pain and crepitation of both knee for
years now come with acute RT knee swelling , on examination you find
that there is edema over dorsum and tibia of RT leg ,what is the best
investigation for this condition >>>>
Rt limb venogram
368- Standard deviations >>
Measure central tendency or variability
Measure extreme value
Measure validity
369- PTS 18 yrs , you prescribe for him retinoid gel will counsel him for
>>>>>
make your skin sensitive for sun light
370- What is the name of questionnaire that differentiate b/w primary
and sleep apnea.???????
371- Post partum female with recurrent attack of hearing loss , which
diagnosed as conductive hearing loss , on CT the is dehesion in the of
semi circular canal diagnosis >>>>
otosclerosis
miner's
Tuberus sclerosis
372-most important investigation to diagnose maxillary sinusitis:
CT
Xray
373-child with bleeding from nose ,history no evidence of clotting
abnormalty ,vitally stable .on eaxamination slow bleeding from posterior
septum whats your action:
do clotting study
spray vasoconstrictive agents.
374- Child with posing head , bowing tibia ,,,, rickets ,,, what is the
deficiency >>>
49

vit D deficiency.
375-80 year old man complain of sever itching mainly in the wrist and b/w
fingers , with excoriation mark linear and superimposed by secondary
infection disturbing sleep, the pts newly finish 10 days course of
Antibiotics >>>>>>
Monilia
eczema
icythiosis
__________________________________________________________________________
__________________________
376- Live guard come to annual examination , no compliant , macular
dicloration, painless over the face , thers is history for exposure
unproductive to sun rays >>>>>
Sqamous cell carcinoma
377- Community problem of multiple chlymedia infection in the eye ,
best prevention method is >>>>
good water and good sanitation supply
378- Which IS considered abnormal & indicate fetal distress<<<
late deceleration
379- What is the vector for leshmania disease <<<<
sand fly
380- Child with URTI then complained from ear pain on examination there
is hyperemia of TM &+ve insufflations test he tri 2 drug no benefit what
is the bestTTT>>>
agmentine
azythromycin
ciprofloxacin
steroid
381- Infant in respiratory distress ,hypercapnia , acidosis & have rhinitis ,
persistent cough +ve aglutenation test & the doctor treat him by
ribavirin DX>>>
pertusus
RSV
382- Aluminum salt & salt will decrease absorption of <<<<
tetracycline
penicillin
383-40 yrs old male com with HX of smoking & alcohol intake for long
time complain of painless ulcer ,role out border on the lateral border of
the tongue DX<<<<<
SCC
lukoplakia
50

384- Old PTNs with osteoporosis TTT for HTN with diuretic that prevent
Ca loss complain of severe pain in big toe DX>>>>>
thiazide
__________________________________________________________________________
__________________________
385-Old male come with CHF & pulmonary edema what is the best initial
therapy>>>>
digoxin
frosamide
debutamine
386- Patient with HX of URTI & flash of light when he sneeze the cause
is>>>>
chemical irritation
mechanical irritation of retina
387- Adult with HTN what is the most common cause >>>>>
renal
essential
388- All of the following exaggerate the gastric ulcer except<<<<
decrease gastric empty time
use of tricyclic antideppresent
__________________________________________________________________________
__________________________
389- Sever pain in anatomical snaph box >>>>>
scavoid fracture
__________________________________________________________________________
__________________________
390-Child come to ER after ingestion of multiple iron tablet of his relative
&iron conc. In blood 700ml???? what is the best intervention>>>
gastric lavage
charcoal oil
iv defrroxamin
391-Patient with lacremation ,salivation, diarrhea, what is antidote<<<<
atropine
pralodexam
392- Child with recurrent UTI how to counsel him>>>
increase fluid intake
393- group of patient with lung cancer divided In tow group disease
&control>>>
case control study
394- Gouty arthritis -ve pirfringes crystal what is the mechanism>>>>
deposition of uric acid crystal in synovial fluid due to over saturation
51

395- Patient have of urethritis now com with lt knee urethral swap +ve
puss cell but ve for N.M, chlymedia>>>>
RA
riters disease
gonococcal
396-Pulled Elbow scenario the TTT>>>
immediate reduction supination
397- Which of the following indicate benign thyroid lesion>>>>
lymphadenitis
__________________________________________________________________________
__________________________
398- Young female, k/c of vitligo, came to the clinic with hx of hair loss, on
exam you found an area of4 cm devoid from hair, but the skin is normal
no redness or any abnormality in this area except hair loss, what does she
has:
a. Alopecia totalis
b. Alopecia areata
399- Young female came to your clinic complaining of skin rash involving
the whole body including the palms & soles after unprotected sexual hx,
what is your DX:
Secondary syphilis
400- Old man came to you with hx of anal pain, spastic in nature
associated with diaphoresis, tachycardia, which last for a few min.
bothering him more in the night, what does he has:
a. Thrombosis of external hemorrhoid
b. Goy syndrome
c. Proctalgia fugax
__________________________________________________________________________
__________________________
401- Female came with hx of sever abdominal pain, vaginal bleeding for 6
hours, amenorrhea for 8 wk, , O/E tachycardiac, hypertensive, tense
abdomen. what is the most likely site of the ectopic pregnancy:
a. Fallopian tube
b. Ovary
c. Peritoneum
d. Fimbria
402- Young pt was operated 1 year ago for small bowel perforation,
presented with hx of vomiting, constipation, abdominal distention, colicky
pain, what will be the appropriate investigation you will choose:
a. Barium swallow
b. Barium enema
c. Barium follow through
d. Double contrast study
52

403- Male came with hx of acute onset of knee pain with swelling, what
will be the most important investigation to do:
a. Arthrocentasis
b. Full CBC
c. Joint US
d. ????
404- Long scenario about child has sore throat & the culture showed
group A strepto, then he develop coca-colored urine, periorbital edema,
and headache what is the single most specific test to diagnose him with
post-strept acute GN:
a. BP more than 95% of normal
b. +ve strepto enzyme
c. Low C3
d. High creatine & urea
(the question is not new but here are the full choices)
405- Regarding case-control study which is true:
a. The number of control should be equal to the number of
diseased
b. It is forward study to the risk factor
c. It is backward study to the risk factor
406- Young pt came with hx of acute painful swelling of the first
metatarsophalangeal joint, redness, tenderness, fever 38c, what is the
etiology:
a. Staph aureus
b. Sodium urate deposition
c. Pyro phosphate calcium deposition
__________________________________________________________________________
__________________________
407- Child with barking cough dx to have croup, what is the causative
organism :
a. Parainflunza
408- Long scenario about 5 year old child otherwise healthy has cough,
fever, chest x ray shows infiltration of the middle & lower Rt lob, after 24
hour of receiving cefotaxime he develop complete Rt lung opasifcation,
what is the most likely causative organism:
a. Staph aureus
b. Strepto, pneumonea
c. Hib
d. Pseudomonas argenosa
In another word :
child pt. came with scenario of chest infection , first day of admission he
treated with cefotaxime , next day , pt state became bad with decrease
53

perfusion and x-ray show complete rt. Side opcifaction + hydrothorax ,


causative organism :
Strepto. Pnem
Staph. Aureus true if pnumothorax
Hemophilus influenza type b
Pseudomonas
-child presented to ER with SOB on x-ray there is filtration
on mid & lower zone on RT side after
24h of antibiotic pts become cyanosis the x-ray total lung
collapse with medastinal shift what cause?
H-influenza
-pneummocystic carnia
-streptoccouse pneumona
409- Regarding face suture, when should you remove it:
a. After 3-5 days
b. After 7-10 days (my answer)
c. Only absorbable suture should be used
410- Pt with dysmenorria, infertility, not responding to naproxen, what
cauld be the cause:
a. Endometritis
b. Endometriosis
411- Pt complain of central lower back pain when he wake up at the
morning, stay for 30 min without medication, with slight improvement
brought by NSAID, investigation shows lumber spinal stenosis, and O/E we
just found Para spinal muscle spasm, otherwise normal, what will you do:
a. Physical therapy
b. Steroid injection
c. Surgery
412- Pt had hx of trauma to his nose 2 days ago, he came with hx of nasal
obstruction & pain, O/E he has bilateral swelling (septal hematoma), what
will you do:
a. I &D
b- Antibiotic
413- pregnant woman with past hx of DVT, what will you do for her:
a. warfarin
b. heparin
c. enoxaparin
d. no anticoagulant need
e. aspirin
414- pt post cardiac arrest, poor tissue perfusion, inadequate ventilation,
ABG shows low PH, normal pCO2, low HCO3, what is the explanation:
a. Resp. acidosis
b. Resp. alkalosis
54

c. Metabolic acidosis
d. Met. Alkalosis
_____________________________________________________________
______________________
415-Long scenario about Pt with hyperkalemia 7.5, what will be your
FIRST action:
a. IV calcium gluconate
b-IV HCO3
416- Treatment of recurrent otitis media after multiple Abx course, with
bulging tympanic membrane:
a. Amoxicillin
b. Amoxicillin & clavulanic acid
c. Tazocin
d. IV ????
417- What is the serious sign in pregnancy:
a. Abdominal pain
b. Back pain
c. Leg swelling
d-Hyperacidity
418- Pt came to ER with his hand vist in the mid of his chest, what is the
most likely Dx:
a. Ischemic heart disease
b. Esophageal spasm
c. Pneumonia
419- After head trauma, the pt start to have disinhiption, agitation, and
restlessness what is the affected lob:
a. Prefrontal area
b. Temporal area
c. Occipital area
d. Parietal area
420- Which of the following suggestive of ovarian cyst rather than ascites
in percussion:

a. Dullness centrally & tympanic peripherally


b. Dullness peripherally &tympanic centrally
c. Dullness all over
d- Fluid wave
e-Decrease bowel motion
i) Shifting dullness
421- Q about sub dermal implantable contraception:
a. Is has low compliance compared to OCP
b. More side effect
55

c. No local reaction

422-Pt
presented with multiple attacks of palpitation, chest pain,
numbness & tingling of the upper limbs & fearing from dying, no
abnormality detected by physical examination. What is the Dx:
Thyrotoxicosis
Panic attacks
Agoraphobia
Peochromocytoma
a 28 yrs. old lady , C/O: chest pain, breathlessness and feeling that she'll die
soon .. O/E : just slight tachycardia .. otherwise unremarkable .. the most likely
diagnosis is:
a- panic disorder
423- which of the following medication is safe during lactation:
Tetracycline
Chloramphenicol
Erythromycin
424- small child presented to ER with hx of sudden onset of groin mass,
painful, associated with vomiting, nausea, O/E groin mass tender, scrotal
swelling with multiple bleeding dots, cremastric reflex is preserve, what is
your dx:
a-Testicular torsion
b- torsion of appendix testis
c-scrotal hematoma
e-incarcerated hernia.
425- what is true about marasmus disease:
a. In contract to kwashorcoir, it affect the low socioeconomic stat
b. It is due to late weaning
c. It leads to growth retardation & wt loss
________________________________________________________________________
_________________________
426- picture of pt with htperpigmentation of the axilla, under the
flourcene wood's light it become pink what is the dx:
a. Fungal infection
b. Erthrasma
c. Acanthosis nigricans
d. ??????
(picture in kumar)
427- the most common cause of 2dry amenorrhea with high FSH & LH is:
56

a. Menopause
b. Pituitary adenoma
c. Pregnancy
d. ???????
a. 428-?????
428- female G3P3 post-partum 4wk treated for 3days with antibiotic ( I
forget it) for PID but no response & she still febrial , O/E by PV you found
a 10cm mass in the sac between the vagina & the rectum, tender,
fluctuating what will be your action:
a. Colpotomy
b. Laparotomy
c. Laparoscopy
d. D/C antibiotic & start another one
429- pt with hyperthyroidism ask you regarding the long term
complication of her disease, what will you do:
a. Bone density scan
b. Brain CT scan
c. ECG
d-Echo
430- pt with hypothyroidism, you start her on the levothyroxine, then she
came for follow up you found her TSH is high, what will you do:
a. Reduce the thyroxin and follow her after 6 months
b. Reduce the thyroxin and follow her after 2 months
c. Continue the same dose & follow up after 2 months
d. Stop the dose till the TSH become normal and follow up after 2
months
431- picture of large cervical lymph node, O/E multiple enlarged lymph
node, matted, non tendr, no skin changes, what is the dx:
a. TB lymphadenitis
b. Metastatic tumor
c. Lymphoma
d. Infectious mononucleosis
432- pt fall from 10 stairs, he develop nasal swelling, tenderness, skull Xray shows non displaced fracture of the nasal bone, what will you do:
a. Refer to the surgeon immediately.
b. Do brain CT scan
c. Reassure him
433- long scenario about pt with GERD responding partially to PPI,
endoscopy biopsy shows barret esophagus with mild severity, what will
be your action:
a. Esophagus resection.
b. Fundoplication
c. Increase the dose of PPI
57

434- child came with hx of one attack of tonic-clonic convulsion, & his
mother said that he has multiple attacks of febrile convulsion, what will
give her if her child develop another attack at home:
Phenytoin
Diazepam
Clonazepam
Phenol barb.
435--- 35 year old smoker , on examination shown white patch on the
tongue, management: leucoplakia
a. Antibiotics
b. No ttt
c.
Close observation
d- excision biopsy ( may answer ) the choice is not in alqasem
question
436-male singer with colon cancer stage B2 ; which of the following
correct ?
a- no lymph node metastases
b-one lymph node metastasis
c-2 ===
d-lymph node metastasis + distant metastasis
437--child swallowing battery in the esophagus management : (the
choices are complete)
-bronchoscope
- insert fly catheter
- observation 12hrs
Remove by endoscope
438-young male patient present to ER due to RTA with poly trauma ; the
beast way to maintains airway in responsive poly trauma patient is ;
A-orophargenial airway
b-nasophargenial airway
c-trachastomy
d-endotracheacheal intubations
439- Young patient with congested nose, sinus pressure, tenderness and
green nasal
discharge, has been treated three times with broad spectrum antibiotics
previously, what is your action? (chronic sinusitis)
a) Give antibiotic
b) Nasal corticosteroid
c) Give anti histamine
d) Decongestant
e)observation
440-long case patient with RTA with Blount trauma to abdomen . patient
undergo remove of distal small intestine and proximal colon , patient
58

come after 6 month with chronic diarrhea , SOB , sign of anemia , CBC
show megaloblastic anemia
What the cause of anemia :
A-folic acid deficiency
b-vit B12 deficency
c-alcohol
441- case infant has genital rash ( the rash spares genital fold ) not
response to antibiotics , most likely Dx;
A-candida albicans
b-napkin dermitis
c-contact dermatitis
d- atobic dermatitis
e- sebborich dermatitis
442--long senior patient came with chest pain , burning in character ,
retrsternal , increase when lying down , increase after eating hot food ,
clinical examination normal DX GERD
a-MI
b-peptic ulcer
c-GERD
d443- about fetal alcohol syndrome (read about)
444- the beast way to ttt pinged induce nervosa ( bullima
nervosa )
a-interpersonal psychotherapy
b-cognitive behavior therapy
c-pharmacotherapy
d445- old female came with scales aroud the areola ,she took steroid but
no benefit on examination normal and no masses what is your next step?
A-Antibiotics
B-anti-fungal
C- Mammography
446- patient come with diarrhea , confusion , muscle weakness he suffer
from which ? (hyponatremia)??
A-hypokalemia
B-hyperkalemia
c-hypercalcemia
447- Patient come with jundice , three days after the color of jundice
change to greenish what is the cause?
oxidation of bilirubin

59

448- Patient with Rhumatoid Arthritis he did an X-Ray for his fingers and
show permanant lesion that may lead to premnant dysfunction , what is
the underlying process?
substance the secreted by synovial
449- prevention of lyme disease , what is best advice to parents
insect repellents
prevention of Lyme disease :
Treat early disease with doxycycline , Prevent with tick bite avoidance

Light-colored
clothing makes the tick more easily visible before it attaches itself.
People should use special care in handling and allowing outdoor
pets inside homes because they can bring ticks into the house.

A more
effective, communitywide method of preventing Lyme disease is to
reduce the numbers of primary hosts on which the deer tick
depends, such as rodents, other small mammals, and deer.
Reduction of the deer population may over time help break the
reproductive cycle of the deer ticks and their ability to flourish in
suburban and rural areas.

Backyard patios,
decks, and grassy areas that are mowed regularly are unlikely to
have ticks present. This may be because of the lack of cover for
mice from owls and other raptors that prey on mice. The ticks also
need moisture, which these areas do not provide.

-The areas
around ornamental plantings and gardens are more hospitable for
mice and ticks. The highest concentration of ticks is found in
wooded areas.

Individuals
should try to prevent ticks from getting onto skin and crawling to
preferred areas.

Long hair should


be worn under a hat.

Wearing longsleeved shirts and tucking long pants into socks is recommended.

Guineafowl
450--30-40 year male suffer from tenitus , vertigo , sensorconductal hear
loss diagnosis is
Miner's disease

60

451--patient 20 year old come with palptations ECG show narrow QRS
complexes and pluse is 300 bpm what is the true
Amidarone
452-40 years female complaining of thinking a lot in his children future,
she is alert, anxious, cant sleep properly, poor appetite, she always make
sure that doors in her home are closed, in spite of doors already closed,
provotional Dx:
OCD,
GAD
Schizo
453-Long scenario of restless leg syndrome(he didnt mention Dx in
scenario), 85 old male many times awake from his sleep bcz leg pain, this
pain relieved by just if he move his foot, but it recure, at rest,best
management:
Colazpin
Haloperidol
lorazepam,
one drug from dopamine agonist group forgot its name, its the right
answer.
Bromocriptin,opomorphine,cobegoline
454-best drug for von willbrand disease is:
fresh frozen plasma
cryoprecipitate
steroids
(he didnt mention vasopressin in choices)
455- Best fast management of acute hyperCalcemia is:
Iv fluid
frusamide
dialysis
456- Null hypothesis definition
457- Long scenario of a pt with melanoma in back, he is afraid of malignat
change, which one of the following indicate malignant melanoma:
>6mm,
irregular and invade the skin,
458- Best inv to visualize the cystic breast masses is:
MRI
CT
61

Mammogram,
US
459-Female pt new diagnosed as HTN pt, BMI 28, U sld advise her by:
Body wight reduction ALONE doesnt benefit her HTN
wt reduction and exercise may benefit him,
salt restriction well help to reduce the BP
460- Long scenario for pt smokes for 35 y with 2 packets daily, before 3
days develop cough with yellow sputum, since 3 hours became blood
tinged sputum, X ray show opacification and filtration of rt hemithorax,
DX:
Bronchogenic CA
acute bronchitis
lobar pneumonia
461-Pt with hypercholestrelemia, he should avoid:
Organ meat
Avocado
Chicken
white egg
462-5 y child diagnosed as UTI, best inv to exclude UTI comp:
Kidney US
CT
MCUG
IVU
463-6 y old boy, eat the paper and soil, best initial ttt is:
Fluoxetine
behavioral therapy,.
464-Baby said baba mama, pincer grasp, creeping well, sitting wth
support, estimate age:
6m
7m
8m
10m
465- Drug of choice for a schistosomaisis is:
Praziquanetilo
xaminiquine,
artemether
466-50 YEARS OLD FEMAL HAS HYPERTENSION ,COMPLAINING OF RIGHT
EYE PAIN ,HEADACH,FLUSHES LIGHT ,FLOTERS,SHE DID NOT TAKE HER
MEDICATIN TWO WEEKS BACK THE BLOOD PRESSURE IS 140\90
,ON EXAMINATION NON DAILATED PUPLE REACTIVE ,NO DECREASE EYE
VISION ,THERE IS CUPPING AND SLIGHT ARTERIOVENOUS NIPPING WHAT IS
YUOER MANAGEMENT:
62

A)REFERAL TO OPTHALMO
B)REASSURANCE AND TELL HER YOUR PROBLEM BENINGN
C)GIVE BEXOLO EYE DROPS
D)TREAT HER AS MIGREN HEADACH
___________________________________________________________________
_______
467-55 ys old male pt, presented with just mild hoarsness, on exam, there
was a mid cervical mass, best inv is:
Indirect laryngioscopy??
CT brain
CT neck
Biopsy
aspiration
__________________________________________________________________________
___
468-4 years old child, was diagnosed as SCD,so many times came to
hospitals with, dyspnia, dactylites , ( he put sign of acute crises ), the best
strategy for prolonged therapy is:
IV hydration fluids with analgesia
follow in Out pt clinic
refer to tertiary haem center.
469- Long scenario for a pt came to ER after RTA, splenic rupture was
clear, accurate sentences describe long term management:
We give pneumococcal vaccine for high risky people just,
we sld give ABs prophylaxis if there Hx of contact even with
vaccination against pneumococcal,
pneumoccal vaccine should not be given at same time with MMR
470- Female pregnant, 32weeks of gestational age, diabetic, and she has
a Hx of full term fetal demise, but her DM now well controlled, and BPP
show no fetal distress,best management:
Wait for SVD
report a CS in 36 weeks
Teminate pregnancy
471- Triad of heart block, uveites and sacroileatis,Dx:
Ankylosing spondylites,
lumbar stenosis,
multiple myeloma
472-72 years old Man with loss of vision in one eye , jaw claudication :
Temporal arteritis .
63

473-GDM with diagnostic GGT what will u do:


-repeat GTT
-diet
-start with monotherapy
-insulin then change to monotherapy
-insulin test
____________________________________________________________________
474- Lt sternal border murmur,ejection systolic, but not radiation to
carotid:
>AS
> PS .
475-commenst cause of failure to thrive:
psychosocial??
allergy to milk and protein
476-child with umblical hernia:
-sponteneous recovery before school age(: 90% disappear
spontaneously during the first 5 years of life
____________________________________________________________________
477-commenst cause of hearing loss in children:
-chronic otitis media
-dysfunction eschian tube
-antenatal causes.
478-child with meningitis symptoms and no nuchal rigidty ,whats the next
diagnostic investigation:
CSF
479-patient given 3 liter 10% dextrose then started to develop confusion
and leg cramps:
hyperkalemia
hypokalemia
hypernatremia
480-table shows only hyponatremia and low plasma osmolality with
normal urine osmolality :
_inappropriate secretion of ADH
-cushing syndrome
-addison
481-german measles cause what:
482-man had gun shot to left lung with decrease breath sound, what well
u do :
-2nd midclavicular needle.
64

-5th midaxillary needle


-5th midaxillary tube.
-ab.
483-> How did we differentiate between snoring and sleep apnea ? I think
!!!!! Read about Sleep study chart !
> Michegan chart . My answer
> Different names of chart -_-"
____________________________________________________________________
484-> Pt K/C of crohns , present with hip and Back pain , normal Ab Ex
and hip ROM , what to do next :
> Hip CT
> Ab US
> IVP
> Abdominal CT
> Renal US
____________________________________________________________________
485- Patient with around Rt eye pain with tearing and headache , lid
swelling , Dx :
> Migrane with aura
> Tension headache
> Cluster headache .
> Glucoma
----------------------------------------------------------------------------486- Recurrent swelling in the natal cleft with skin tract and recurrence ,
Dx :
> Hydrandinitis suuporitiva .
> Frunclosis .
65

> F..dermatic
487-> Recently diagnosed with DM type II , 32 years old , exercise for 8
weeks and BMI changed from 32 to 31 ..
Labs shown on table ? But no table !!!!!
> Continue exercise . My answer
> Start medication
488-74-year old female patient of Cushings syndrome, had hip fracture
falling off stool, what will you screen for while also treating her fracture:
Hyperparathyroidism
Osteomyelitis
Osteoporosis
Osteomalacia
489-Very long scenario about middle age man (50 years) with family
history of heart disease, active lifestyle, on self induced diet with 50% fat,
35% protein and 15 % carbohydrates, table showing labs, elevated LDL,
low HDL, elevated triglycerides and cholesterol, normal RFTs and all other
labs.
No risk of heart disease
Heart disease risk can be avoided by taking statins
Heart disease can be prevented by decreasing calorie intake
490-18 month old patient, parents were treating baby for flu-like illness
with fever with increased water intake at home, patient developed
generalized tonic-clonic convulsions, presented to you after 1 hour with
(description of post-ictal stage). Now baby becomes fully conscious.
What is the next most appropriate step?
rapid sponging to reduce fever
Give acetaminophen and antipyretics
Treat fever, Get MRI done and treat after result
491-9. Middle aged female patient with history of Stage 2 breast cancer
treated successfully, now presents with moderate to severe pain in left
leg, not relieved by lying down, pain on extension of leg and walking, O/E
Tender region in L3-L4 lower back. No Physical sign of cancer recurrence.
Last saw oncologist 2 years back. What is most appropriate scenario:
Refer to oncologist
66

Do DEXA Scan
Do MRI
Hospitalize and do neurology and oncology consultations
492-11. Baby present with weeping, shiny and crusting lesions around
mouth :
Impetigo
493-Male patient was advised to undergo Arterial Graft Bypass surgery at
other clinic after having episode of pain in leg, now is asymptomatic.
Came to you, Non-smoker, elevated cholesterol and early atherosclerotic
plaques on some descending aortal branches. What will you advise:
Undergo Bypass Grafting
Take medication to prevent formation of Arterial plaques
To undergo frequent arterial scans to see extent of disease.
494-1st line class of drugs against Post partum hemorrhage:
Uterine Contractile
Uterine Relaxant
495-Patient recovering from Viral Gastroenteritis, vomiting and diarrhea
abated but still having Anorexia. What will you advise:
Bananas
Rice cereal and apple juice Chopped pears
yougurt and .
Granola, .
496-Patient with idiopathic anovulation. What drug to give:
Clomiphene
Progesteron
LH
FSH
497-Description of PCOS. Mechanism of PCOS:
Androgen Excess
498-Prostitute with multiple sex partners presents with history of painless
vaginal sore which healed and did not leave scar. O/E has generalized
lymphadenopathy. What is your diagnosis:
Syphilis
__________________________________________________________________________
499-Female patient with wide-open eyes, tremors in hands that do not
diminish with intention, What investigation will you do:
Pituitary Scan
67

T4 Levels
500-Middle age patient alcoholic with H/O fullness in epigastric region and
mild pain, History of nausea and vomiting. Labs: Increased Serum
Amylase, Diagnosis:
Pancreatic Pseudocyst
Pancreatic Cystadenoma
Choledochal Cyst
Liver Cirrhosis
501-Which one of these patients with pneumonia will you treat as outdoor
patient:
a.
80 Year old with 104 F temperature, BR 24/min PR 126/min, BP
180/110
b.
60 year old with 102 F temperature BR 22/min PR 124/min, BP
160/110
c.
50 year old with 98 F temperature, BR 20/min. HR 110/min, BP
180/110
d.
80 year old with 96 F temperature, BR 18/min, HR 70/min, BP 110/80
http://pda.ahrq.gov/clinic/psi/psicalc.asp
according to pneumonia severty index calculator (class IV and V need
hospitalization class III depend on clinical judgment) the high blood
pressure is not involved in calculation.
a-classIII
B-classII
c-class I
D-class III
502-Long scenario of 28 year old male patient with symptoms of
Ulcerative Colitis+ anemia related to UC. Sigmoidoscopy revelaed
multiple polyps, Biopsy of polyps Carcinoma in situ. What is the most
definitive therapy that will be effective in the long-term:
1.
Correct Anemia
2.
Left hemicolectomy and Colostomy
3.
Total Colectomy and Ilectomy
4.
Removal of all polyps by Colonoscopy
503-Female patient came with hypertension, azootemia and GFR of 44.
What is her condition due to:
a. Pheochromocytoma
b.Renal artery stenosis
c.Renal Parenchymal Disease
(pheochromocytoma is excluded from the scenario
renal artery stenosis :1- Significant functional impairment of autoregulation,
leading to a decrease in the GFR, is not likely to be observed until arterial luminal
narrowing exceeds 50%
68

2-Patients with documented or possible renovascular hypertension may experience


progressive azotemia as a consequence of the renal ischemia and/or the persistence of
significant hypertension.
3-Refractory hypertension (ie, poor control of blood pressure despite treatment with 3 or more
antihypertensive agents) may occur. (emedicine)

renal parenchymal disease: 1- present with HTN.


2-the GFR OF 44>>>means chronic kidney disease stage 3

0) Normal kidney function GFR above 90mL/min/1.73m 2 and no proteinuria


1) CKD1 GFR above 90mL/min/1.73m2 with evidence of kidney damage
2) CKD2 (Mild) GFR of 60 to 89 mL/min/1.73m2 with evidence of kidney damage
3) CKD3 (Moderate) GFR of 30 to 59 mL/min/1.73m 2
4) CKD4 (Severe) GFR of 15 to 29 mL/min/1.73m 2
5) CKD5 Kidney failure - GFR less than 15 mL/min/1.73m 2 Some people add CKD5D for
those stage 5 patients requiring dialysis; many patients in CKD5 are not yet on dialysis

504- Patient comes with attack of Strep Throat, had history of previous
attack(RF), what is his chance of getting RHD now?
Nothing, he is immune due to previous infection.
100%
Needs Immunoglobulin to prevent re-infection.
50% chance of re-infection.
In the United States, rheumatic fever rarely develops before age 3 or after age 40 and is
much less common than in developing countries, probably because antibiotics are widely
used to treat streptococcal infections at an early stage. However, the incidence of rheumatic
fever sometimes rises and falls in a particular area for unknown reasons. Overcrowded
living conditions seem to increase the risk of rheumatic fever, and heredity seems to play a
part. In the United States, a child who has a streptococcal throat infection but is not treated
has only a 0.4 to 3% chance of developing rheumatic fever. About half of the children who
have had rheumatic fever develop it again after another streptococcal throat infection if it is
not treated. Rheumatic fever follows streptococcal infections of the throat but not those of
the skin (impetigo) or other areas of the body. The reasons are not known.

http://www.merckmanuals.com/home/childrens_health_issues/bacteri
al_infections_in_infants_and_children/rheumatic_fever.html
505-pediatric patient from developing country presented with muscle
wasting, weight loss and absent edema. What is the diagnosis:
Marasmus
Kwashiorkor
Muscle wasting syndrome
Marasmus:1-present of muscle wasting
2-body weight less than 80%of average weight.
3-absence of edema
4-increase prior to age 1
kwashiorkor: 1-presence of edema
2-increase in >18month
69

muscle wasting syndrome:1-loss of weight.


2-muscle atrophy.
3-in older pt with chronic disease.
(ref. wikepidia)
The most suitable answer is :marasmus
506-Patient with decreased vision, also peripheral vision decreased, using
tonometer pressure in right eye 24 mm and left eye 22 m. What is the
mechanism:
a.Obstruction in trabecular meshwork and ciliary muscle leads to
pupillary blockage and drainage of aqueous humor.
b.Obstruction at ciliary muscle leads to blockage in drainage of
Aqueous Humor.
In cases where POAG is associated with increased IOP, the cause for the elevated IOP generally
is accepted to be decreased facility of aqueous outflow through the trabecular meshwork.
Occurrence of this increase in resistance to flow has been suggested by multiple theories

http://emedicine.medscape.com/article/1206147-overview#a0104
507-Picture of optic disc. (Looks like this:
http://www.revophth.com/content/d/cover_focus/i/1315/c/25316/)
What does it show?
a. Normal disc
b. Cupping of optic disc
508- Picture of Patients legs (calves) showing maculopapular rash. H/O
red rah appearing on extensor surfaces. Rash is tender to palpate but
does not blanch on pressure. What is the diagnosis:
Henoch-Schnolein Purpura
Polyarteritis nodusa

Henoch-Schoenlein purpura begins with a symmetrical erythematous macular rash on the


lower extremities that quickly evolves into purpura. The rash may initially be confined to
malleolar skin but usually extends to the dorsal surface of the legs, the buttocks, and the ulnar
side of the arms. Within 12-24 hours, the macules evolve into purpuric lesions that are dusky
red and have a diameter of 0.5-2 cm. The lesions may coalesce into larger plaques that
resemble ecchymoses. Several cases of Henoch-Schoenlein purpura have been observed
after varicella infections.

(emedicine)
______________________________________________________________________
509- Female patient comes with history of periorbital swelling, itching all
over body, O/E there is lymphadenopathy. Liver and spleen are enlarged.
What is the diagnosis?
a.Urticarial
b.Angioedema
c.Lymphoma??
510- Picture of base of mouth showing a white patch with sharplydemarcated edges. Patient is male, long- term smoker and chews
70

tobacco, presents with painless lesion in mouth. What is the next most
important step:
Topical Fluconazole
Biopsy
Wide surgical excision
Diagnosis is :leukoplakia:
1-painless white plaque
2-associated with smoking
3- on the mucous membranes of the oral cavity, including the tongue, but also other areas of
the gastro-intestinal tract, urinary tract and the genitals.
4-Tobacco, either smoked or chewed, is considered to be the main culprit in its development
5-5% to 25% of leukoplakias are premalignant lesions; therefore, all leukoplakias should be
treated as premalignant lesions by dentists and physicians - they require histologic evaluation or
biopsy (ref. wikepidia)

511- What drug is likely to cause heat-stroke as it inhibits sweating :


Orphanedrine
Hyoscamine Sulfate

Hyoscamine sulfate Warnings:


In the presence of high environmental temperature, heat prostration can occur with drug use (fever
and heat stroke due to decreased sweating)
http://www.drugs.com/pro/hyoscyamine-sulfate-elixir.html
other drugs:
Anticholinergics
CogentinandArtaneareexamplesofanticholinergicdrugs.Theyarebothusedinthetreatmentof
Parkison'sdisease.Medscape.comsuggeststhatanticholinergicmedicationsareinvolvedinthedevelopment
ofheatstroke.Thesedrugsinhibitthebody'ssweatingmechanism,leadingtoinadequateheatelimination.
Thermal Analyzer Leaders In Analytical Instruments Contact Us For Quality
Products. www.Scinco.com
Sponsored Links
Neuroleptics
Neurolepticdrugsarealsoknownasantipsychoticmedications.Theyareusedtotreatmentalhealth
problemslikeschizophreniaandbipolardisorder.Somedrugsinthisclassofpsychiatricmedicationsare
Haldol,ProlixinandThorazine.Accordingto"Goldfrank'sToxicologicEmergencies,"antipsychoticdrugs
inhibitthebody'snormalresponsetoheat.Theyreducethebody'sabilitytoincreasebloodsupplytothe
skinforthepurposeofheatelimination.Duetothis,individualstakingmedicationsfromthisdrugclassmay
beatriskforheatstroke.
Diuretics
RobertWoodJohnsonUniversityHospitalreportsthatdehydratedindividualswhocannotcannotsweat
enoughtocooltheirbodymayexperienceheatstrokebecausetheirinternaltemperaturemayriseto
dangerouslyhighlevels.Diureticsaredrugsthatputindividualsatriskforheatstrokebecausetheypromote
71

dehydration,accordingto"IrwinandRippe'sIntensiveCareMedicine."Theyworkbymakingthebody
eliminatefluidsthroughurine.Furosemideandhydrochlorothiazidearediureticmedications.
Sympathomimetics
Sympathomimeticdrugs,asstatedin"SynthesisofEssentialDrugs,"mimictheactionsofthebody's
sympatheticnervoussystem.Theyincreaseheartrate,openuptheairwaysandconstrictthebloodvessels.
Sympathomimeticdrugssuchasamphetamines,cocaineandephedrinecanleadtoheatstroke,becausethey
constrictthebloodvesselsanddonotpermitheatlossthroughbloodvesseldilation.Thisclassofdrugsare
alsoriskfactorsforheatstrokebecausetheyincreasetheamountofheatproducedwithinthebody
Antihypertensives
Antihypertensivemedicationsareusedtotreathighbloodpressureandalsoputindividualsatriskforheat
stroke.Examplesofdrugsinthisclassarebetablockerssuchaspropanololandcalciumchannelblockers.
Thesedrugsreduceheartrate,thestrengthwithwhichtheheartcontractsandtheamountofbloodpumped
outtothebody.Thisleadstodecreasedbloodflowtotheskinandareductioninthebody'sabilityto
eliminateheat.
Read more: http://www.livestrong.com/article/118891-medications-predispose-heatstroke/#ixzz1jMAxCHRd

http://www.livestrong.com/article/118891-medications-predispose-heatstroke/
512- CT of Brain picture. Scenario: Patient with sudden severe occipital
headache came to emergency.
Subarachnoid Hemorrhage
Intracerberal Hemorrhage
Meningitis
The classic symptom of subarachnoid hemorrhage is thunderclap headache (a headache
described as "like being kicked in the head", [4] or the "worst ever", developing over seconds to
minutes). This headache often pulsates towards the occiput (the back of the head).[

(wikepidia)

intracerberal hem.:Patients with intraparenchymal bleeds have symptoms that correspond to the
functions controlled by the area of the brain that is damaged by the bleed. [3] Other symptoms
include those that indicate a rise in intracranial pressure due to a large mass putting pressure
on the brain.[3] Intracerebral hemorrhages are often misdiagnosed as subarachnoid
hemorrhages due to the similarity in symptoms and signs. A severe headache followed by
vomiting is one of the more common symptoms of intracerebral hemorrhage. Some patients may
also go into a coma before the bleed is noticed.

513- Patient with severe pain in forehead, over nose and sides of face,
also present are shiny blisters on surface of forehead, face and nose.
What is the diagnosis:
Post-herpetic neuralgia
Varicella
Herpes Simplex
Herpes Zoster
72

Herpes zoster is a viraldisease characterized by a painful skin rash with blisters in a limited area on
one side of the body, often in a stripe. The initial infection with varicellazostervirus (VZV) causes
the acute (short-lived) illness chickenpox which generally occurs in children and young people.
Once an episode of chickenpox has resolved, the virus is not eliminated from the body but can go
on to cause shinglesan illness with very different symptomsoften many years after the initial
infection. Herpes zoster is not the same disease as herpessimplex despite the name similarity (both
the varicella zoster virus andherpessimplexvirus belong to the same viral
subfamily Alphaherpesvirinae).
Varicella zoster virus can become latent in the nervecellbodies and less frequently in nonneuronal
satellitecells of dorsalroot, cranialnerveor autonomic ganglion,[1] without causing any symptoms.
[2]
Years or decades after a chickenpox infection, the virus may break out of nerve cell bodies and
travel down nerve axons to cause viral infection of the skin in the region of the nerve. The virus may
spread from one or more ganglia along nerves of an affected segment and infect the
corresponding dermatome (an area of skin supplied by one spinal nerve) causing a painful rash. [3]
[4]
Although the rash usually heals within two to four weeks, some sufferers experience residual
nerve pain for months or years, a condition called postherpeticneuralgia.
Herpes simplex:1-blisters containing infectious particles.
2-common infection may affect:1-face mouth(orofacial) 2-genitalia(genital herpes) 3hands(heraptic whitlow)
3-can cause herpes keratitis.
Varicella: is a highly contagious illness caused by primary infection with varicella zoster
virus (VZV).[1] It usually starts withvesicular skin rash mainly on the body and head rather than
at the periphery and becomes itchy, raw pockmarks, which mostly heal without scarring.
Post-herpetic neuralgia:
s a neuralgia caused by the varicella zoster virus. Typically, the neuralgia is confined to
a dermatomic area of the skin and follows an outbreak of herpes zoster (HZ, commonly known
as shingles) in that same dermatomic area. The neuralgia typically begins when the
HZ vesicles have crusted over and begun to heal, but it can begin in the absence of HZ, in which
case zoster sine herpete is presumed (see Herpes zoster).
The most likely answer is herpes zoster.

_____________________________________________________________________
514- Patient had fly in his eye. On removal of the foreign object what will
he need:
Topical corticosteroids
Topical Antibiotics
Oral corticosteroids
Oral Antibiotics
Answered based on the sle studing group couldnt find a ref.
515- Patient complains of discomfort in the eye. There is no discharge.
73

O/E with dye, a dendritic shaped ulcer is seen on the surface of the
cornea. What is the diagnosis:
Keratitis
Uveitis
A corneal ulcer, or ulcerative keratitis, or eyesore is an inflammatory or more seriously, infective
condition of the cornea involving disruption of its epithelial layer with involvement of the
corneal stroma.

(Wikepidia)
________________________________________________________________
516- A patient complains of 2 day history of stuck together lashes on
waking up. There is muco- purulent discharge. Anterior Chamber, uvea
and iris are clear. What is the diagnosis?
a.
Bacterial Infection
b.
Viral Infection
c.
Allergy
Bacterial conjunctivitis is usually a benign self-limiting illness, 1 although it can sometimes be
serious or signify a severe underlying systemic disease. Occasionally, significant ocular and
systemic morbidity may result.2
Epidemiology
This is one of the most common ocular problems seen in the community.3
In adults, bacterial conjunctivitis is less common than viral conjunctivitis; although
estimates vary widely, it is thought to account for no more than half of all cases of acute
infective conjunctivitis.4
It is most commonly caused by Staphylococcus spp., Streptococcus
pneumoniae, Haemophilus influenzae, and Moraxella catarrhalis.5
In children, bacterial conjunctivitis is more common than viral and is mainly caused by H.
influenzae,S. pneumoniae and M. catarrhalis.
Presentation
History
Relevant aspects of the history include:

Nature of the problem:


o Discomfort - burning or gritty but not sharp.
o Pain - should be minimal; significant pain suggests a more serious diagnosis.
o Vision - usually normal, although 'smearing', particularly on waking, is common.
o Discharge - this tends to be thick rather than watery.
o Associated symptoms such as photophobia which should be absent or, at most,
mild. Significant photophobia suggests severe adenoviral conjunctivitis or some
degree of corneal involvement.
Contact lens wear: could this be (or lead to) a problem of the (vulnerable) cornea?
Time course: onset, duration - where this is chronic, you may have to consider venereal
disease in people at a sexually active age.
Use of over-the-counter medication: could this be a reaction to previously administered
drops or ointment?
74

Social aspect - has anybody else had it (family, school, work?) and are there issues about
staying at home during the course of the illness?

Findings
'Red eye' with uniform engorgement of all the conjunctival blood vessels.
Bacterial conjunctivitis may often be distinguished from other types of conjunctivitis by
the presence of a yellow-white mucopurulent discharge. Eyes may be difficult to open in
the morning, glued together by discharge.
There is also usually a papillary reaction (small bumps on the palpebral conjunctiva,
appearing like a fine velvety surface). The presence of follicles is more likely to indicate
viral conjunctivitis.
Bacterial conjunctivitis is usually bilateral (but often sequential). 3
Check visual acuity - this should be normal, other than the mild and temporary blur
secondary to the discharge which can be blinked or wiped away.
Is this bacterial?
It is not always easy to determine whether the patient's simple, acute conjunctivitis is bacterial
or not but this is important as it may determine the subsequent management plan. Ultimately,
swabbing the eye provides the most accurate diagnostic answer but it is clearly not practical to
do this for every patient. A study has shown that, in adult patients, there is a significant chance
that the infection is bacterial when there is a combination of: 4
A positive previous history of infectious conjunctivitis.
An itch present.
A mucopurulent discharge ('glue eye').
However, in severe, resistant, atypical cases or in immunosuppressed patients, swabbing for
culture and sensitivities is important.3
When patients describe their eyes glued together in the morning, this doesn't necessarily mean
that there is a purulent discharge. Viral and allergic conjunctivitis often result in lids that are
matted shut in the morning with mucopurulent material. However, these patients actually have
crusting of the lashes due to drying of tears and serous secretions, not the wet, sticky,
mucopurulent matting characteristic of bacterial conjunctivitis.

http://www.patient.co.uk/doctor/Bacterial-Conjunctivitis.htm
__________________________________________________________________________
517-Patient presents with red, peeling rash at back of ears, on limbs and
over body. What is the first line treatment?
Topical steroid
Oral Steroid
Oral Antibiotic
The diagnosis:scarlet fever :
The cutaneous rash, , lasts for 4-5 days, followed by fine desquamation, one of the most
distinctive features of scarlet fever. The desquamation phase begins 7-10 days after resolution of
the rash, with flakes peeling from the face. Peeling from the palms and around the fingers occurs
about a week later and can last up to a month or longer. The extent and duration of this phase are
directly related to the severity of the eruption.
Antibiotic therapy is the treatment of choice for scarlet fever.
75

Penicillin remains the drug of choice (documented cases of penicillin-resistant group A


streptococcal infections still do not exist). A first-generation cephalosporin may be an effective
alternative, as long as the patient does not have any documented anaphylactic reactions to
penicillin. If this is the case, erythromycin may be considered as an alternative.

??518- 50-year old accountant, sedentary lifestyle, BMI 30, takes irregular
meals; arteries show signs of early atherosclerotic changes. What will you
advise?
No meds necessary
Prescribe diet of 600 kcal/day and reevaluate in 4 months
Prescribe over weight diet and reevaluate in 6 months
??519-Mechanism of Action of drugs that inhibit Conversion of estriol to
estrogen. (I forgot the exact question but it mentioned about ovulation
and who inhibits conversion of esterone to estrogen?) Options were:
a. Aromatase inhibitors??
520-What drug reverses the effect of Benzodiazepines:
a. Flumazanil
Flumazenil (Anexate) is a competitive benzodiazepine receptor antagonist that can be used
as an antidote for benzodiazepine overdose.

521- pateint C/o ictrus in skin and eye on investigation WBC 2500
plt 70,000 HG 7 lekocytosis 17% total bilirubin 51 and direct bilrubin 12 what is the
test most likly positve
+ve coomb test
In us obstructive billiary duct
antiparietal cells antibodies
522-Pts have history of cervical incompetence pregnant at 8w what the
management?
Do circulage at 14-16w
523--child presented with anemia he have family history of thalassemia
what the most diagnostic test?
-measuring of HB A2
-bone marrow
-serum feriten
524-Fracture of elbow common injury of ?
Radial
525-_pts presented with pruritis hepatosplenomegaly lef supraclavicular
LN ?
Cholinergic pruratis
Lymphoma
526- the antipsychotic drug have less pyramidal side
effect is ?
76

There was significant optimism when they were first developed and it was thought that they
represented a breakthrough in the treatment of schizophrenia due to having less extra-pyramidal
side effects at therapeutic doses. The extra-pyramidal side effects has been the one significant set
of side effect that has led to poor compliance with antipscyhotic medication.
he common atypical antispychotic drugs include risperidone, olanzapine, quetipaine, aripiprazole,
zyprasidone, clozapine and amisulpiride.

http://www.understand-schizophrenia.com/atypical-antipsychotics.html

527-_old age presented with vesicular rash on thoracic tell the


midline ?
Herpes zoster
528--female came with her baby with history of fatigue
palpitation and tremor this due to?
Postpartum anemia
Hashimotos thyroiditis
__________________________________________________________________________
__________________
__
a very short scenario)Female come to the clinic with her baby of 6 month , she had
tremor and other sign I forgot it, which of the following is most likely dx
... Hashimoto
Postpartum thyroiditis
hypertyrodism,
sub acute tyroditis
hypothriodism
Postpartum thyroiditis is a phenomenon observed following pregnancy[1] and may
involve hyperthyroidism, hypothyroidism or the two sequentially. It affects about 5% of all
women within a year after giving birth. The first phase is typically hyperthyroidism. Then,
the thyroideither returns to normal or a woman develops hypothyroidism.
The initial phase of hyperthyroid symptoms occurs transiently about two to six months
postpartum[2]. Typical symptoms include fatigue, irritability, nervousness, palpitations, and heat
intolerance. Hormonal disturbances during this phase tend to occur with lower intensity compared
with the hypothyroid phase[2

529-antidote of acetaminophen:
N-acetylcysteine.
??530-_pts with hypertension and cp of aortic dissection what the
management ?
531-wich true about hepatoma?(read about it)
77

Is common in female
Comes with Chronic liver disease
http://emedicine.medscape.com/article/197319-overview
532-The most powerful epidemiologic study is:
a. retrospective case control study
b. cohort study
c. cross-sectional study
d. historic time data
e. secondary data analysis
533- Evidence base medicine:
practice medicine as in the book
practice according to the department policy
practice according to available scientific evidence
practice according to facility
practice according to latest publish data
534- Pt had fever in the morning after he went through a surgery (I
couldnt remember the type of surgery). Whats your diagnosis:
Atelectasis if 0-2 days)
Wound infection
DVT
UTI
Post-operative fever
Days 0 to 2:
o Mild fever (T <38 C) (Common)
o Tissue damage and necrosis at operation site
o Haematoma
o Persistent fever (T >38 C)
o Atelectasis: the collapsed lung may become secondarily infected
o Specific infections related to the surgery, e.g. biliary infection post biliary surgery,
UTI post-urological surgery
o Blood transfusion or drug reaction
Days 3-5:
o Bronchopneumonia
o Sepsis
o Wound infection
o Drip site infection or phlebitis
o Abscess formation, e.g. subphrenic or pelvic, depending on the surgery involved
o DVT
After 5 days:
o Specific complications related to surgery, e.g. bowel anastomosis breakdown,
fistula formation
o After the first week
o Wound infection
o Distant sites of infection, e.g. UTI
78

DVT, pulmonary embolus (PE)

535-23 yrs old female has mobile breast lump in the upper outer
quadrant of the left breast. Size= 2 cm and doesnt change with
menstrual cycle. Whats the most likely diagnosis:
Fibroadenoma
1-the most common benign breast mass in adolescent
2-easy to move with well defined edges
3-often in upper outer quadrent
4-not affected with menstrual cycle ,the affected with menstrual
cycle is cystic breast changes
536- Breast feeding in the full term neonate:
a. Increase URTI rate
b. No need for vitamin supplementation
c. Food introduce at 3 months
d. Increase GE rate
537-Which of the following organisms can cause invasion of the
intestinal mucosa, regional lymph node and bacteremia:
a. Salmonella??
b. Shigella
c. E. coli
d. Vibrio cholera
e. Campylobacter jejeni
538- Pt has saddle nose deformity, complaining of SOB,
hemoptysis and hematiuria. The most likely diagnosis is:
Wagners granulomatosis
http://en.wikipedia.org/wiki/Wegener's_granulomatosis
539-OCP:
a. Changes the cervical mucus
b. increase pre menstrual tension
c. Have a failure rate of 3 %
a.

Combined oral contraceptive pills were developed to prevent ovulation by suppressing the release
of gonadotropins. Combined hormonal contraceptives, including COCPs, inhibit follicular
development and prevent ovulation as their primary mechanism of action. [5][24][71][72][73]

b.

Progestogen negative feedback decreases the pulse frequency of gonadotropin-releasing


hormone (GnRH) release by the hypothalamus, which decreases the release of follicle-stimulating
hormone (FSH) and greatly decreases the release of luteinizing hormone (LH) by the anterior
pituitary. Decreased levels of FSH inhibit follicular development, preventing an increase
inestradiol levels. Progestogen negative feedback and the lack of estrogen positive feedback on LH
release prevent a mid-cycle LH surge. Inhibition of follicular development and the absence of a LH
surge prevent ovulation.[5][24][71]

79

c.

Estrogen was originally included in oral contraceptives for better cycle control (to stabilize the
endometrium and thereby reduce the incidence of breakthrough bleeding), but was also found to
inhibit follicular development and help prevent ovulation. Estrogen negative feedback on the
anterior pituitary greatly decreases the release of FSH, which inhibits follicular development and
helps prevent ovulation.[5][24][71]

d.

A secondary mechanism of action of all progestogen-containing contraceptives is inhibition


of sperm penetration through the cervix into the upper genital tract (uterus and fallopian tubes) by
decreasing the amount of and increasing the viscosity of the cervical mucus.[73]

(wikipedia)
540-Pt has 2 cm dome shaped mass in the dorsum of his hand. Its covered
by keratin. Whats the most likely diagnosis:
Basal cell carcinoma
Malignant melanoma
Keratoacnathoma
KA is that it is dome-shaped, symmetrical, surrounded by a smooth wall of inflamed skin, and capped with keratin
scales and debris.(Wikipedia)

541-Pt has hemorrhagic lesion in the mouth and papules in the face and back.
He had SOB, fever, cough and mediastinal mass. Whats the diagnosis:
a. Kaposi sarcoma
1

KS lesions are nodules or blotches that may be red, purple, brown, or black, and are usually papular (i.e.,
palpable or raised).

They are typically found on the skin, but spread elsewhere is common, especially the mouth, gastrointestinal
tract and respiratory tract. Growth can range from very slow to explosively fast, and is associated with
significant mortality and morbidity.[16]

a.

[edit]Skin

Commonly affected areas include the lower limbs, back, face, mouth, and genitalia. The lesions are usually as
described above, but may occasionally be plaque-like (often on the soles of the feet) or even involved in skin
breakdown with resulting fungating lesions. Associated swelling may be from either
local inflammation or lymphoedema (obstruction of local lymphatic vessels by the lesion). Skin lesions may be quite
disfiguring for the sufferer, and a cause of much psychosocial pathology.

b.

[edit]Mouth

tttttttttttttttttttttttttttttttttttttttttttttttttttttttttttttttttttttttttttttttttttttttttttttttttttttttttttttttttttttttttttttttttttttttttt

a.

Intraoral AIDS-associated Kaposi sarcoma with an overlying candidiasisinfection

Is involved in about 30%, and is the initial site in 15% of AIDS-related KS. In the mouth, the hard palate is
most frequently affected, followed by the gums.[17] Lesions in the mouth may be easily damaged by chewing and bleed
or suffer secondary infection, and even interfere with eating or speaking.

80

c.

[edit]Gastrointestinal

tract

Involvement can be common in those with transplant-related or AIDS-related KS, and it may occur in the
absence of skin involvement. The gastrointestinal lesions may be silent or cause weight loss, pain,
nausea/vomiting, diarrhea, bleeding (either vomiting blood or passing it with bowel motions), malabsorption,
or intestinal obstruction.[18]

d.

[edit]Respiratory

tract

Involvement of the airway can present with shortness of breath, fever, cough, hemoptysis (coughing up blood),
or chest pain, or as an incidental finding on chest x-ray.[19] The diagnosis is usually confirmed by bronchoscopy when
the lesions are directly seen, and often biopsied

(wikipedia)
542-In the neck, esophagus is:
Posterior to the trachea
Anterior to the trachea
Posterior to vertebral column
543-High risk factor in CLL :
Age
Smoking
History of breast ca
History of radiation
Factors that may increase the risk of chronic lymphocytic leukemia include:

Your age. Most people diagnosed with chronic lymphocytic leukemia are over 60.

Your sex. Men are more likely than are women to develop chronic lymphocytic leukemia.

Your race. Whites are more likely to develop chronic lymphocytic leukemia than are people of other races.

Family history of blood and bone marrow cancers. A family history of chronic lymphocytic leukemia or
other blood and bone marrow cancers may increase your risk.

Exposure to chemicals. Certain herbicides and insecticides, including Agent Orange used during the
Vietnam War, have been linked to an increased risk of chronic lymphocytic leukemia.

http://www.mayoclinic.com/health/chronic-lymphocyticleukemia/DS00565/DSECTION=risk-factors
There are no other proven risk factors for CLL. The risk of getting CLL does not seem to be affected by smoking, diet, exposure to
radiation, or infections.

http://www.cancer.org/Cancer/LeukemiaChronicLymphocyticCLL/DetailedGuide/leukemia-chronic-lymphocytic-riskfactors
544-Which of the following medications is considered as HMG-CoA reductase
inhibitor:
Simvastatin
Fibrate

81

All statins act by inhibiting 3-hydroxy-3-methylglutaryl coenzyme A HMG-CoA reductase,


the rate-limiting enzyme of the HMG-CoA reductase pathway, the metabolic
pathway responsible for the endogenous production of cholesterol.

_______________________________________________________
545- Burn involved 3 layers of the skin called:
Partial thickness
Full thickness(entire dermis)
Superficial
Deep
__________________________________________________________________
546-Cherry red skin found in:
a. Polycythema
b. CO poisoning
http://www.thelancet.com/journals/lancet/article/PIIS0140-6736(05)79807X/fulltext
_________________________________________________________
547- Most serious symptom of CO poisoning is:
Hypotension
Arrhythmia
Cyanosis
Seizure
548-Patient with subconjuctival hemorrhage. What you will do for him:
Reassurance sure 100%
Send him to the ophthalmologist
549-Patient with renal transplant, he developed rejection one week post
transplantation, what could be the initial presentation of rejection:
Hypercoagulability
Increase urine out put
Fever
Anemia
Signs and Symptoms of Kidney Rejection
Fever over 38C or 100.4F
Decreased urine output
Weight gain over 2 pounds per day
Increased blood pressure
Pain over kidney

http://www.uihealthcare.com/topics/medicaldepartments/surgery/rejection/ind
ex.html
550- Pregnant lady in her 30 wks gestation diagnosed as having swine flu.
She has high grade fever and cough for 4 days and her RR= 25/min. what will
you do for her:
Give her Tamiflu 75 mg BID for 5 days
Refer her to ER for admission
Give her antibiotics
Refer her to OBGY doctor
82

551- Female patient has morning stiffness and pain involving the
metacarpophalengeal and proximal interphalengeal joints. Whats the likely
diagnosis:
Rheumatoid arthritis
552-DPT vaccine shouldnt given if the child has:
a. Coryza
b. Diarrhea
c. Unusual cry
d. Fever = 38
553-A female patient has clubbing, jaundice and pruritis. Lab results showed
elevated liver enzymes (Alkaline phosphatase), high bilirubin, hyperlipidemia
and positive antimitochondorial antibodies. Whats the most likely diagnosis:
Primary sclerosing cholangitis
Primary biliary cirrhosis
___________________________________________________________________
554-Shoulder pain most commonly due to:
Infraspinatus muscle injury
Referred pain due to cardiac ischemia
In acute cholecystitis
___________________________________________________________________
555-Female patient with fatigue, muscle weakness, parasthesia in the lower
limbs and unsteady gait. Do:
a. Folate level
b. vitamin B12 level
c. Ferritin level
___________________________________________________________________
556- Patient developed lightheadedness and SOB after bee sting. You should
treat him with the following:
d. Epinephrine injection, antihistamine and IV fluid
e. Antihistamine alone
557-Patient is 74 yrs old female complaining of pain and stiffness in the hip
and shoulder girdle muscles. She is also experiencing low grade fever and
has depression. O/E: no muscle weakness detected (Polymyalgia rheumatic).
Investigation of choice:
RF
Muscle CK
ESR
___________________________________________________________________
558- 2 yrs old boy with coryza, cough and red eyes with watery discharge (a
case of measles). Most likely diagnosis of the red eyes is:
Conjunctivitis
Blepharitis
559-foot ball player gt hurt by the football to the knee , on examnation +ve valgus
stress test, -ve macmerry and lachman test . he has:
1) tibial fracture
83

2) lateral meniscus tear


3) medial meniscus tear
4) lateral collateral ligament tear
5) medial collateral ligament tear
. A foot ball player his knee was hit from the left lateral side, valgus test
was positive; drawer test & Laschman test were negative. Which of the
following was injured?
a) Anterior cruciate ligament
b) Posterior cruciate ligament
c) Medial collateral ligament
d) Lateral collateral ligament
e) Medial meniscus
___________________________________________________________________
560-Patient has bilateral abdominal masses with hematuria. Most likely
diagnosis is:
a. Hypernephroma
b. Polycyctic kidney disease
Polycyctic kidney disease: Pain in the abdomen, flank, or back is the most common initial complaint, and it is
almost universally present in patients with autosomal dominant polycystic kidney disease (ADPKD). The pain can be
caused by any of the following:

Enlargement of one or more cysts


Bleeding, which may be confined inside the cyst, or lead to gross hematuria with passage
of clots or a perinephric hematoma
Urinary tract infection (eg, acute pyelonephritis, infected cysts, perinephric abscess)
Nephrolithiasis and renal colic
Rarely, a coincidental hypernephroma

In addition, patients with ADPKD may have abdominal pain related to definitively or presumably associated conditions.
Dull aching and an uncomfortable sensation of heaviness may result from a large polycystic liver. Rarely, hepatic cysts
may become infected, especially after renal transplantation.
Abdominal pain can also result from diverticulitis, which has been reported to occur in 80% of patients with ADPKD
maintained on dialysis, probably from altered connective tissue. However, this rate has not been demonstrated to be
higher than the rate among other patients on dialysis.
Patients with ADPKD may be at a higher risk of developing thoracic aortic aneurysms. Abdominal aortic aneurysms
are not increased among these patients.
Pain may also develop for reasons completely unrelated to the underlying disease; thus, abdominal pain in patients
with ADPKD may be a diagnostic challenge.

Hematuria
Hematuria frequently is the presenting manifestation and usually is self-limited, lasting 1 week or less. Polycystic
kidneys are unusually susceptible to traumatic injury, with hemorrhage occurring in approximately 60% of individuals.
Mild trauma can lead to intrarenal hemorrhage or bleeding into the retroperitoneal space accompanied by intense pain
that often requires narcotics for relief.

Hypernephroma(renal cell carcinoma): wide range of symptoms can be present with renal
carcinoma depending on which areas of the body have been affected. [3] The classic triad
is hematuria (blood in the urine), flank pain and an abdominal mass.

flank mass is uncommon (10%) and is indicative of advanced disease.


84

And I think the disese is unilateraral


The best choise is polycystic kidney

561- Male patient working in the cotton field, presented with 3 wks Hx of
cough. CXR showed bilateral hilar lymphadenopathy and biopsy (by
bronchoscopy) showed non-caseating granuloma. Whats your diagnosis:
Sarcoidosis
Amylidosis
Histiocustosis
Byssinosis
Pneumoconiosis

Byssinosis, also called "brown lung disease" or "Monday fever", is an occupational lung
disease caused by exposure to cotton dust in inadequately ventilated working environments.

___________________________________________________________________
562-Pt presented with severe epigastric pain radiating to the back. He has
past hx of repeated epigastric pain. Social hx: drinking alcohol. Whats the
most likely diagnosis:
MI
Perforated chronic peptic ulcer
If pancerititis come in choises I dont know if it is better answer_
__________________________________________________________________
563-Erosive gastritisdont know
Happened within one week of injury
Happened within 24 hrs of injury !!!
_________________________________________________________________
564-In brainstem damage:
a. Absent spontaneous eye movement
b. Increase PaCO2
c. Unequal pupils
d. Presence of motor movement
Commonly, brain stem damage causes a loss of consciousness. It may be temporary or more extended. People
with severe brain stem damage can enter comas and persistent vegetative states with limited probability of
waking up again. Other people may be conscious and aware, but could have severe breathing problems,
abnormal heart rates, or balance disorders. More mild injuries may result in a staggering gait and sensory
impairments associated with interruptions to sensory signals.
Brain stem consisting of the midbrain, pons, and medulla, which extends downwards to become the spinal cord
Controls respiration and various basic reflexes (e.g., swallow and gag)
Absent pupillary light reflex
Corneal reflexes are absent
PaCO2 levels greater than 60 mmHg, 20 mmHg over baseline

565-fluxtin half life =


1-4 days
85

566-35 yrs old male has SOB, orthopnea, PND, nocturia and lower limbs
edema. Whats the most common cause of this condition in this patient:
a. Valvular heart diease
b. UTI
c. Coronary artery disease
d. Chronic HTN
Coronary artery disease and heart attack. Coronary artery disease is the most common form of heart
disease and the most common cause of heart failure.

http://www.mayoclinic.com/health/heartfailure/DS00061/DSECTION=causes
567-Female patient had carpopedal spasm after measuring her BP. This is
caused by:
Hypocalcemia
Causes of Carpopedal spasm:
The following medical conditions are some of the possible causes of Carpopedalspasm. There are likely to be other
possible causes, so ask your doctor about your symptoms.

Hypocalcemia

Multiple blood transfusions

Parathyroidectomy

Ileostomy formation

Gastric resection with gastrojejunostomy

Hyperventilation

Tetany

Following thyroid surgery due to damage to the parathyroid glands

568- Patient known case of DM type 2 on insulin, his blood sugar


measurement as following: morning= 285 mg/dl, at 3 pm= 165 mg/dl, at dinner
time= 95 mg/dl. What will be your management:
Increase evening dose of long acting insulin
Decrease evening dose of short acting insulin
Decrease evening dose of long acting insulin
Increase evening dose of short acting insulin
569-Diabetic patient on insulin and metformin, has renal impairment. Whats
your next step:
Stop metformin and add ACE inhibitor
DM HTN patient with MI receiving metformin and diltiazem and other medication his
creatine clearance is high .. you will do:
a) add ACE inhbitor
86

b) remove metformin (contraindicated in renal failure)


c) continue same medication
570-4 weeks old male child with acute onset forceful non-billious vomiting
after feeding. He is the first child in the family. He is gaining normal wt and
looks hungry. Whats your diagnosis:
Pyloric stenosis
______________________________________________________
571- 1 week old infant presented with repeated forceful vomiting. Whats the
diagnosis ???(the same history of pyloric stenosis but the age is 1 wk):
Pyloric stenosis
Duodenal atresia
Volvolus
Hirschprung
572-55 y/omale,, c/o angina and syncope on exertion ,, normal ejection
fraction ,, normal coronary arteries ,, there is only calcified aortic valve
with total area < .75 cm ,, the rest of examination and investigations are
normal ..
Wt is ur manag. :
Avoid exertion
Medical therapy
Aortic ballon dilation
Aortoc valve replacrment
The only definitive treatment for aortic stenosis is aortic valve replacement. The development of symptoms due to
aortic stenosis provides a clear indication for replacement. For patients who are not candidates for aortic replacement,
percutaneous aortic balloon valvuloplasty may provide some symptom relief. [4]
Medical treatment (such as diuretic therapy) in aortic stenosis may provide temporary symptom relief but is generally
not effective long term.
In truly asymptomatic patients with severe aortic stenosis, the issue of valve replacement is less clear.

The recommendations of the ACC/AHA 2006 valvular heart disease guidelines for aortic valve replacement in patients
with valvular aortic stenosis are summarized below, in Table 5. [10] In most adults with symptomatic, severe aortic
stenosis, aortic valve replacement is the surgical treatment of choice. If concomitant coronary disease is present,
aortic valve replacement and coronary artery bypass graft (CABG) should be performed simultaneously.
Table 5. Recommendations for Aortic Valve Replacement in Aortic Stenosis(OpenTableinanewwindow)

Indication

Class

Symptomatic patients with severe aortic stenosis

Patients with severe aortic stenosis undergoing coronary artery bypass surgery

Patients with severe aortic stenosis undergoing surgery on the aorta or other
heart valves

87

Patients with severe aortic stenosis and LV systolic dysfunction (ejection fraction I
< 0.50)
Patients with moderate aortic stenosis undergoing coronary artery bypass
surgery or surgery on the aorta or other heart valves

IIa

Patients with mild aortic stenosis undergoing coronary artery bypass surgery
when there is evidence that progression may be rapid, such as moderate-tosevere valve calcification

IIb

Asymptomatic patients with severe aortic stenosis and abnormal response to


exercise (eg, hypotension)

IIb

Asymptomatic patients with severe aortic stenosis and a high likelihood of rapid IIb
progression (based on age, calcification, and coronary artery disease) or if
surgery might be delayed at the time of symptom onset
Asymptomatic patients with extremely severe aortic stenosis (valve area less
than 0.6 cm2, mean gradient greater than 60 mm Hg, and jet velocity greater
than 5 m per second) if the patients expected operative mortality is 1% or less

IIb

AVR is not useful for prevention of sudden death in asymptomatic patients with
none of the findings listed under asymptomatic patients with severe aortic
stenosis

III

emedicine
573-Patient with untreated bronchogenic carcinoma has dilated neck veins,
facial flushing, hoarsness and dysphagia (SVC syndrome). CXR showed
small pleural effusion. Whats your immediate action:
a. Consult cardiologist for pericardiocentesis
b. Consult thoracic surgeon for Thoracocentesis
c. Consult oncologist(with radiotherapy symptoms improve)
the most common cause of SVC syndrome is bronchogenic carcinoma
574-Fixs method in determining cardiac output ;??
1 BP
2 o2 saturation in blood
COP = PR/ BP

575-3 year old boy with acute UTI .. first thing to do in such acute
thing ;??
a-Indwelling foley cather drain
b voiding cytctogram
c- cystoscopyd-US
576-- drug contraindication hypertrophic obstructive cardiomyopathy;
88

A_ digoxin
B_ one of b-blocker
Avoid digitalis because glycoside are contraindicated except in pts with
uncontrolled atrial fibirallation (emedicine)
577-- BPH pt with hypertension what to give;
alpha blocker
578-- posterior hip dislocation : ( from reconstruction)
A flexion , adduction
b- flexion abduction
c- extension, adduction
_posterior hip dislocation :the hip is flexed ,internal rotation,adducted
(emedicine)
__________________________________________________________________________
_________________________
579-- action of ocp : ( from reconstruction)
A - inhibition of estrogen then ovulation
B inhibition of prolactin then ovulation
d. inhibition of mid cycle gonadotropin then ovulation
580- 30 age women with sharp pain in the index finger increase with using
scissors or nail cut which cause sharp pain at the base of the finger in
matacarpophlyngeal joint and the finger become directed downward in
(mean flexed DIPj) and cause pain when try to extend the finger..
1-trigger finger
2-tendon nodule
3-dupetren contracure
4- mullet finger
loss of extensor tendon continuity at the DIPJ causes the joint to rest on
an abnormaly flexed position
the classic mechanism of injury is a finger held rigidly in extension or
nearly full extension(emedicine)
581-- itching scale in pack of knee . face and ant elbow :
A scapis
B eczema
c- contact dermitis
eczema:the earliest lesion affect anticubital and popliteal fossa
lesions are ill defined erythematous,scaly, patches and
plaques(emedicine)
582- mitral stenosis :
A diastolic high pitch
B - systolic low pitch
C- diastolic low pitch
89

583- chylmedia non pregent treatment :


A doxycycline
Non-pregnant:azithromycin-doxycycline. Pregnant:erythromycin.
584- difference between unstable and stable angina :
A necrosis of heart muscle
B - ..
Stable angina:the classic triad of angina consist of :substernal chest pain
that is provoked by exertion and relieved by rest or nitrates.
Secondary to myocardial ischemia not necrosis(necrosis in MI)
Prescence of st segment eleveation in the absence of cardiac enzyme
elevation.
ASA and B blocker (mortility benefit in stable angina)
Unstable angina:chest pain is newonset,accelerating, or occurring at rest.
(occurs with less exertion,lasts longer,less responsive to medication)
It signals the presence of possible impending infarction.
Not associated with elevated cardiac markers but their can be st
segment elevation.
(first aid USMLE STEP 2)
585-- mechanism of destruction of joint in RA :
A swilling of synovial fluid
B anti inflamtory cytokines attacking the joint
0
.
Synovial hyperplasia and endothelial cell activation are early events in
the pathologic process to uncontrolled inflammation and consequent
cartilage and bone destruction.
586-TB outbreak ..and one pt. come to doing tubercalin test andit's
negative .. what to do??
a- BCG
d. isonized
e. give rifampicin
587-all of the following is extrapyramidal Sx exept ??
a- dyskinisia
b- akathesia
c- xxxxx esia
1
clonic - tonic convulsion
588-female come to with 3 UTIs history in last 6 months,, what is your
advise?
a- wipe frome behinde to front afterdefecation
b- take a bath insted of shaower.
a. increse flude intake
??589-in newborn exam .. what is more dangerous ?
a- hydrocele
90

b- abcent femoral pulses


ttttttttttttttttttttttttttttttttttttttttttttttttttttttttttttttttttttttttttttttttttttttttttttttttttt
CHD
f. - breast with milk discharge (normal it's with' milk)
??590-12- 28 gestation in NICU 900 gram wighet .. otherwise normal ..
what to do?? >>>>
a- give hem milk orally
b- glucose infusion (because they develop hypoglycemia)
c- broad spectren antibiotic
??591-healthy child with pRBC in urin 15 cells/hpf .. what to do ..??
a. repeat urine analysis for blood and proten .
b. urine cytology.
c. Cystoscopy.
d. Renal biopsy.
592-tebial tubercle pain .. in 13 y/o boyhaving growth spurt ...Dx? >>>>
..
a- osgood fracture
b- strees fracture
......... .a
most likely Osgood schlatter,condition occurs in active boys and girls
from 9-16
the condition is self-limiting
593- old dibetic man with sudden unilateral visual loss .. thare ismultible
pigmentation in retena with macular edema .. Dx??..

a- retenal detachment ( wrong .. come with floters )
b- retinal artry occlosion ( wrong .. no chirry red spots )
c- retinal vien thrombosis ( my answer .. it'sfit with all data given )
e. dibetic retinopathy
retinal detachment:A retinal detachment is commonly preceded by a posteriorvitreousdetachment which
gives rise to these symptoms:

flashes of light (photopsia) very brief in the extreme peripheral (outside of center) part of
vision

a sudden dramatic increase in the number of floaters

a ring of floaters or hairs just to the temporal side of the central vision

a slight feeling of heaviness in the eye

Although most posterior vitreous detachments do not progress to retinal detachments, those that do produce the
following symptoms:

a dense shadow that starts in the peripheral vision and slowly progresses towards the
central vision
the impression that a veil or curtain was drawn over the field of vision
91

straight lines (scale, edge of the wall, road, etc.) that suddenly appear curved
(positive Amsler grid test)
central visual loss

(None of this is to be confused with the broken retina which is generally the tearing of muscle and nerve behind the
eye)

retinal artery occlusion:The most common presenting complaint is an acute persistent painless
loss of vision. In central artery occlusions, visual loss is central and dense. In branch artery occlusions, visual loss
may go unnoticed if only a section of the peripheral visual field space is affected.

A complete visual field defect suggests central retinal artery occlusion (CRAO).

The cherry red spot and a ground-glass retina are the classic findings but may take hours to develop.

Retinal vein thrombosis : Central retinal vein occlusion (CRVO) is essentially a diagnostic finding of
painless unilateral loss of vision. In some cases, this loss of vision is subtle in character, with intermittent episodes of
blurred vision. In other cases, it may be sudden and dramatic. The nonischemic type is often the more subtle of the
two, while the ischemic type is prone to the more acute clinical presentations.

Nonischemic CRVO - Subtle, intermittent visual loss; painless; mild-to-moderate visual loss

Ischemic CRVO - Acute visual loss; pain may be present; marked visual loss
BRVO is similar in presentation to CRVO. BRVO is often noted with an onset of blurred vision or visual field defect.
Vision loss may be subtle. Patients with small occlusions of a branch retinal vein may often be asymptomatic. Larger
obstructions can lead to significant visual loss. It is uniformly a unilateral disease. Nine percent of cases are bilateral.
Nonischemic central retinal vein occlusion

Mild vision loss, usually better than 20/120 measured


Rare afferent pupillary defect
Ophthalmoscopy findings consist of variable dot and flame hemorrhages in all 4 quadrants, optic nerve
swelling, retinal vein engorgement and tortuosity, cotton wool spots are few
Ischemic central retinal vein occlusion

Marked visual loss, usually 20/200 to only hand motion


Afferent pupillary defect
Ophthalmoscopy findings of extensive retinal hemorrhages in all 4 quadrants, optic disc is edematous, retinal
vein markedly edematous and engorged

Macular edema is often severe.

Bleeding may result in vitreous hemorrhage.

Retinal detachment may occur.


Branch retinal vein occlusion

Patients with BRVO have retinal hemorrhages confined to the distribution of the retinal vein.
The ophthalmoscopic examination may note triangular and flame-shaped hemorrhages.
Mild obstruction of a branch may only show scant hemorrhage. Complete obstruction may have extensive
hemorrhage noted on examination, with cotton wool spots.

Diabetic retinopathy: In the initial stages of diabetic retinopathy, patients are generally asymptomatic; in
the more advanced stages of the disease, however, patients may experience symptoms that include floaters, blurred
vision, distortion, and progressive visual acuity loss.

Microaneurysms
Microaneurysms are the earliest clinical sign of diabetic retinopathy and occur secondary to capillary wall outpouching
due to pericyte loss. They appear as small red dots in the superficial retinal layers, and there is fibrin and red blood
cell accumulation in the microaneurysm lumen. A rupture produces blot/flame hemorrhages. Affected areas may
appear yellowish in time, as endothelial cells proliferate and produce basement membrane.

92

Dot and blot hemorrhages


Dot and blot hemorrhages occur as microaneurysms rupture in the deeper layers of the retina, such as the inner
nuclear and outer plexiform layers. These appear similar to microaneurysms if they are small; fluorescein angiography
may be needed to distinguish between the two.

Flame-shaped hemorrhages
Flame-shaped hemorrhages are splinter hemorrhages that occur in the more superficial nerve fiber layer.

Retinal edema and hard exudates


Retinal edema and hard exudates are caused by the breakdown of the blood-retina barrier, allowing leakage of serum
proteins, lipids, and protein from the vessels.

Cotton-wool spots
Cotton-wool spots are nerve fiber layer infarctions from occlusion of precapillary arterioles. With the use of fluorescein
angiography, there is no capillary perfusion. These are frequently bordered by microaneurysms and vascular
hyperpermeability.

Venous loops and venous beading


Venous loops and venous beading frequently occur adjacent to areas of nonperfusion and reflect increasing retinal
ischemia. Their occurrence is the most significant predictor of progression to proliferative diabetic retinopathy.

Intraretinal microvascular abnormalities


Intraretinal microvascular abnormalities are remodeled capillary beds without proliferative changes. These collateral
vessels do not leak on fluorescein angiography and can usually be found on the borders of the nonperfused retina.

Macular edema
Macular edema is the leading cause of visual impairment in patients with diabetes. A reported 75,000 new cases of
macular edema are diagnosed annually. This may be due to functional damage and necrosis of retinal capillaries.
Clinically significant macular edema is defined as any of the following:

Retinal thickening located 500 m or less from the center of the foveal avascular zone
(FAZ)
Hard exudates with retinal thickening 500 m or less from the center of the FAZ
Retinal thickening 1 disc area or larger in size located within 1 disc diameter of the FAZ

594-16 y/o female become deaf suddenly.. her mother become deafwhen
she was 30.. Dx:
a- otosclerosis 100% sure(autosomal dominant,positive family history)
b- acostic neuroma
c-tympanic perforation
595-major hazard in post-menepause: >>>>>>>>>>(all true )
a- osteoprosis>>i'm 90%sure
b- hot flush
c- deppresion
d- pelvic floor weakness
596-which of the folloing b- blocker .. havean alpha blocking effect :
>>>>>>>
a- metoprlol
b- atenalol
c- mesoprolol
a. xxxxxx lol
93

b. yyyyyy lol
labetalol and carvedilol (block beta and alpha)
597-theScreeningQuestionnaireto recognize primary snoring fros OSAS is :
..
a- otowaQuestionnaire
b- HorchoverQuestionnaire
c- .........
........ .a
598-ADHD Rx : >>>>> ADHD
a- olanzapine
b- atomixtin
cmethylphenidate,
magnesium pemoline
atomoxetine
dextroamphetomine
lisdexamfetamine
bupropion
venlafaxine
lmipramine
guanfacine
clonidine
599-patient with congistive heart failure and pulmonary edema, what is
the best treatment:
1-spronalctone
2-forsumide.
3
____________________________________________________________________
600-post partum women when she went back to work ,, she exposed
tothe sun and started to have brown discolortion in her face .. what is
thediagnosis:
1. uritcariA pigementosa (x)
melasma/chloasma (a patchy browen or dark brown skin discoloration,
that usually occurs on face and may result from hormonal
changes,generally found in sun exposed areas.
601-patient presented by left arm swelling , pain full axillary
lymphadenopathy ttt by ;
a- oral antibiotics (if only lymphadenitis)
b.IV antibiotics ??(if systematic symptoms)
602- which one of the following is prognostic factor for CML ;
a- age.
1
chromosomal abnormality

94

in CML there is chromosomal translacation (Philadelphia)


CML was the first malignancy to be linked with clear
genetic abnormality.
603-baby sit e out support ,,crawling , walking by pulling up ,
age ..
a- 12 month
b- 10 month
c- 9 month
604-18- patient came with left arm stiffness and pain , he cant
abducted his arm .. dx
a- subcromial bursitis
b- glenohumoral arthiritis
cdSubacromial bursitis is a condition caused by inflammation of the bursa that separates the
superior surface of the supraspinatus tendon (one of the four tendons of the rotator cuff) from the
overlying coraco-acromial ligament, acromion, coracoid (the acromial arch) and from the deep
surface of the deltoid muscle.
subacromial bursitis often presents with a constellation of symptoms called impingementsyndrome.
Pain along the front and side of the shoulder is the most common symptom and may cause weakness and stiffness.
[4]
If the pain resolves and weakness persists other causes should be evaluated such as a tear of the rotator cuff or a
neurological problem arising from the neck or entrapment of the suprascapular nerve.
The onset of pain may be sudden or gradual and may or may not be related to trauma.
Impingement may be brought on by sports activities, such as overhead throwing sports and swimming, or over head
work such as painting, carpentry or plumbing.
Activities that involve repetitive overhead activity, or directly in front, may cause shoulder pain. Direct upward pressure
on the shoulder, such as leaning on an elbow may increase pain.
Night time pain, especially sleeping on the affected shoulder, is often reported.
Localized redness or swelling are less common and suggest an infected subacromial bursa.
The patient actively abducts the arm and a painful arc occurs between 80 and 120.

Shoulder arthritis can be one of three types of arthritis in the glenohumeral joint of the shoulder
Themainsymptomofshoulderarthritisispain;thisisduetothegrindingofthebonesagainsteachotherbecauseofthelackof
cartilage.Painusuallyoccursinthefrontoftheshoulderandisworsewithmotion.Peoplewithshoulderarthritiswillalso
experiencemoderatetosevereweakness,stiffnessdevelopingovermanyyears,andtheinabilitytosleepontheaffectedshoulder.
[edit]Diagnosis
Diagnosisissimple;usuallythedoctorcantellifyouhaveshoulderarthritisbyyoursymptoms,butheorshewillmostlikelyalso
takeanxrayorMRI.
[edit]

95

605-- 10 yr old boy , dx as persistent moderate asthma , came


with exacerbation .. he is on bronchodialator q 4 hour,, how to
manage this patient to control his disease ;
a- no ttt needed
b- inhaler corticosteroid
c- anticholenergic
606-Which personality disorder is associated with inflexibility and perfectionism:
a. Narcissistic personality disorder
b. Borderline personality disorder
c. Obsessive compulsive personality disorder
d. Histrionic personality disorder
607-male fall from 10 stairs , on examination contusion over the
nose .. ur action will be ;
a- CT scan
b- referred to ENT I think this is the correct answer bcz in exam I
choose CT scan and it was wrong
608- treatment of somatization is ;(cognitive behavioral therapy
is the best)
a- reffered to pain clinic
b- anti depressant
609-1- 50 yr old male, presented with yellowish discoloration of
both eyes and body, fatigue O|E nothing except jaundice ,
pallor , vitiligo .. investigation ; wbc; 2500 , hgb ; 7.5 , plt ; 51 ..
LFT; elevation of total bilirubin and direct bilirubin..
Which one of the following is correct to complete this syndrome ;
a- positive coombs test
2
antibodies against parietal cells
autoimmune hemolytic anemia:
-antibodies directed against the persons own red blood cells
-the primary illness is idiopathic ,secondary can result from many
other illness (autoimmune)
-evidence of hemolysis (incresse unconjugated bilurbin,decrease
haptoglobin,increase lactic dehydrogenase)
-specific investigation:positive direct coombs test.
__________________________________________________________________________
_____________
610-3 yr old ingested aspirin the best gastric decontamination is:
a. Gastric lavage
b. Activated charcoal
c. Total bowel irrigation
Activated charcoal to soak up aspirin in the stomach
Laxative to cause bowel movements that help remove aspirin and charcoal from the body
Other medicines may be given through a vein, including potassium salt and sodium bicarbonate, which helps the body remove aspirin that
has already been digested.
If these treatments do not work or the overdose is extremely severe, hemodialysis may be needed to remove aspirin from your blood.

96

611- 14years old girl failed in math exam .. then she hadpalapitation ,,
tachypnea and paracethesia .. this is :
1-hyperventilation syndrome
2-conversion
1-hyperventilation 2-parasthesia 3-pscho problems 4-no organic causes.
612--child moved with his family to new city n he started to go tonew
school .. in the school he had low mood n doesn't want to interactive
withany activity .. this a case of:
1-hypomania
2-depression
612-origin of pancreatic carcinoma :
Ductal epithilum
Of all pancreatic cancers, 80% are adenocarcinomas of the ductal epithelium.

613-pt. using haloperidol , developed rigidity (dystonia) ttt :


Antihistamine + anticholinergic
614- difference between primary and secondary
hyperaldosteronism :
Increase rennin in secondary
Typically, renin levels are suppressed to less than 1 ng/mL/h in patients with primary hyperaldosteronism, and levels
do not stimulate above 2 ng/mL/h with diuretics and upright posture. Because of this finding, some experts suggest
that suppressed renin levels should be used as a screen for detecting primary hyperaldosteronism. However, in a
1993 series by Bravo, 30% of patients had renin levels that rose to greater than 2 ng/mL/h when appropriately
stimulated by sodium deprivation.[3]

615- attak rate for school children whom developed pink eye ,
first day 10 out of 50 , second day 30 out of 50 :
20
40
60
80
attack rate=number of infected/total number of exposedx100=80
616- group of diseases include , cystic fibrosis , liver failure , the
cause is :
Alpha one antitrpsin def
617- sinus tachycardia and atrial flutter , how to differentiate :
Carotid art message
Temporal art message
Adenosine iv
618- ear pain , headache . purulent discharge , right side
weakness and loss of sensation , diagnosis :
Epidural brain abscess
97

Subdural hematoma?
Spinal column abscess
HZV
Ramsay Hunts syndrome
Description:
Unilateral herpes zoster infection of the geniculate ganglion (respectively n. Intermedius) with lesions of the external ear and oral mucosa.
The symptoms include facial paralysis, severe pain in the ear with a bloody serous discharge due to vesicles on the tympanic membrane.
source
http://www.ramsayhunt.org/epon.shtml

619- all of the following drugs contraindiacted in breast feeding


except :
Tetracycline
Chlorophenicol
Erythromycin
620- GERD diagnosis :
By history only
History and UG Endoscopy
History and barium anema
621-old lady came to clinic as routine visit , she mention decrease
intake of Ca food , doctor suspect osteoporosis , next initial
investigation :
DEXA
Ca in serum , thyroid function test , vit.D
According to family medicine doctor pt >65 start with DEXA
622- long case of hemochromatosis with liver cirrhosis and
decrease weight last visit = 90 now 84 , next step
investigation :
Hepatitis C serology
Alpha phetoprotein
Abdominal ultra sound
623- pt with rheumatic fever after untreated strep infection after
many years presented with Mitral regurge,the cause of massive
regurge is dilatation of:
1. Rt atrium
2. Rt ventricle
3. Lt atrium
4. Lt ventricle
624- ibuproven contraindicated in :
Gastric ulcer
Hypertension
98

625- pregnant lady with cystitis , one of the following drugs


contraindicated in her case : imp.
Amoxicillin
Ceftriaxone
Flouroquiolone
626--female pt. with typical presentation of trichomons , ttt :
Metronidazole
__________________________________________________________________________
627 classical case of candidal infection itching , white
discharge from vagina , ttt is :
Miconazole
Amoxicillin
628- one of following true regarding systolic hypertension :
In elderly its more dangerous than diastolic htn
Occur usually due to mitral regurge
Defined as systolic , above 140 and diastolic above 100
( combined systolic and diastolic
629- 8 years boy BMI = 30 weight and hight above 95 percentile ,
next step :
Appointment after 1 year
Refer to surgeon
Life style modification
630 one of the following cause painful vision loss : imp.
Central retinal vein thrombosis
Retinal detachment
They are painless the painful condition is optic neuritis
631- Pt with symptoms of Mild intermittent asthma , converted to
mild persistant asthma and pt. on albterol
U have to add :
Long acting beta
Short acting inhaled steroid
__________________________________________________________________________
632-Scenario for pt. with sever asthma , tight chest , tachypnea
and Co2 = 50 , next step :
Aminophylin .
Intubation
Short acting beta and discharge him
(in acute asthma give venolin+ipratropium promide(atrovent))
633-18 month child with low grade fever , barking cough ,
inspiratory stridor your Dx :
99

Acute trachyobronchitis (croup)


Epiglottitis
634-Continuous murmur at left sternal area :
Patent ductus arteriosus
Mitral reg
Aortic stenosis
635- most commo cause of otorrhea ???(check question 606 in alqasim)
a) acute otitis media
b)cholesteatoma(chronic otitis media)
c)estichian tube dysfunction
Patients with acquired cholesteatomas typically present
with recurrent or persistent purulent otorrhea and hearing
loss. Tinnitus is also common.
636- the average menstrual cycle is
28 days
__________________________________________________________________________
__________________________
637-All features of tonsillar abscess except :
deviation of uvula to affected side:
638-Erethema nodosum :
painful red nodules
639- initial Inx in small bowel obstruction :
Erect& supine abdominal X- ray
640- Gold standard imaging in acute panceriatitis :
CT scan
641- pregnant lady 28 wks with chlamyda infection :(see 583)
azithromycin
erythromycin or amoxicillin.
doxcyline(non pregnant)
642- common cause of male infertility:
primary hypogonadism
secondary hypogonadism
ejaculation obstruction

100

...Causes of Male Infertility


Varicocele: Varicocele is a condition that affects approximately 40 percent of infertile males. It is caused by
enlargement of the veins in the scrotum. If these veins are enlarged it can cause the temperature to increase.
Increased temperature of the scrotum affects sperm production and sperm quality.
Cryptorchidism: If the testes do not properly descend into the scrotom this is called cryptorchidsm. Cryptorchidism is
fairly common in premature births and occurs in full term births at a much lesser rate. Usually this resolves itself within
a few weeks, but sometimes corrective surgery is needed. Cryptorchism can seriously impair fertility rates. If both
testes are affected the chance of a successful pregnancy is very small.
Disease or illness: Diseases such as cystic fibrosis can affect male fertility. Men who have undergone chemotherapy
or had sexually transmitted desases may also be at greater risk of infertility.
Obstructions in the reproductive tract: Obstructions in the male reproductive system is another common cause of
infertility. Men with obstruction problems are usually making sperm but it is not making its way outside of the penis.
Common causes of obstructions are urinary tract infections, scarring from sexually transmitted disease, injury,
vasectomy or anatomical defects.
Injury: Accidents or sports injuries can cause damage to the male reproductive system resulting in infertility.
Hormone Deficiencies: Deficiencies in male hormone production such as testosterone can affect sperm production.
Doctors will usually ask questions about when puberty was started. The age at which males enter puberty can be an
indicator of hormonal disorders.
Medications: There are many medications that affect fertility and cause sexual disfunctions. It is important for a doctor
to evaluate what medications a man is taking to determine the possible impact on his fertility.
Retrograde Ejaculation: Retrograde ejaculation is caused when the nerves or muscles in the bladder do not work
properly. What happens is sperm is transported backwards into the bladder instead of forward and out through the
penis. Sometimes men will have cloudy urine after an ejaculation because of this.
If you are having infertility issues it is important that both partners are evaluated. A doctor will do a thorough physical
exam as well as evaluate the man's sperm. Men may be uncomfortable going in for an exam but it is really necessary
that both partner's be evaluated during a fertility work up. Treatment options will vary depending on what the cause of
his infertility is.

Read more:http://www.justmommies.com/articles/male-infertility.shtml#ixzz1k5jLlr00

__________________________________________________________________________
__________________________
643-18years old boy with back pain investigation to do except :
CBC ?
ESR
X -ray
bone scan
644- Hypothyroidisim :
free T4
TSH
T4
..
__________________________________________________________________________
__________________________
645-in which group you will do lower endoscopy for patients with iron
deficiency aneamia in with no benign cause:
101

male all age group


children
permanupausal women
women + OCP
Upper and lower GI investigations should be considered in all
post-menopausal female and all male patients where IDA has
been confirmed unless there is a history of significant overt nonGI blood
loss. In the absence of suggestive symptoms (which are
unreliable) the order of investigations is determined by local
availability. The appropriateness of investigating patients with
severe co-morbidity or other reasons (in some circumstances
advanced age), especially if the result would not influence
management, should be carefully discussed with patients and
carers when possible.
http://www.bsg.org.uk/pdf_word_docs/iron_def.pdf
646- Patient with continous seizures for 35 min. despite taking 20 mg Iv
diazepam..what to do??
_ give 40 mg IV diazepam
_ give IV phenytoin
_ give IV Phenobarbital
when it reach to maximum dose of diazepam ( 20 mg ) or respiratory depression
occurs , we start second line of ttt : phynetoin at the dose of 15 mg /kg ( usually 900
- 1000 mg ) or at ratio of 10 mg of phyenoin per 1 ml normal saline but not
exceeding 100ml over 1- 2 hour ( not more than 50mg per minute ) ......
647- A 56 yr old his CBC showed, Hb=11, MCV= 93 Ret= 0.25% the cause is:
a. Chronic renal failure
b. Liver disease
c. Sickle cell anemia
d. G6P dehydrogenase deficiency
648- 3 yr old with symptoms of acute urinary tract infection which of the following
you would like to do in this acute state:
a. Renal U/S
b. Folly catheter
102

c. VSUG
d-US
649-2 months old child complaining of spitting of food , abd examination
soft lax , occult blood ve , what you will do ?
Reassure the parents
Abd CT
650- baby with streptococcus pharyngitis start his ttt after two days he
improved, Full course of streptococcus pharyngitis treatment with
amoxicillin is : imp.
10 days ( 9-11 days )
7days
14 days
651- Uncomplicated UTI ttt
TMP-SMX for 3 days
Ciprofloxacin 5 days
652- Facial injury suturing remove after? Imp.
24h
3 5 days
( most likely )
7 10 days
14 days
653- Lichen planus most common site ?
Scalp
Neck
Knee
Buttocks
As far as I know the common site will be near the wrist and the ankle
but those sites were not one of the choices ?
654-One of the following condition does not cause hypokalemia
Metabolic alkalosis
Furosemide
Hyperaldosteronism
Acute tubular necrosis
Diarrhea
655-Condition not associated with increase alpha feto protein
Breech presentation
Down syndrome
Gastroschisis
__________________________________________________________________________
__________________________
656- Pt came with trauma to left eye by tennis ball examination shows
anterior chamber hemorrhage you must exclude ?
Conjunctivitis
Blepharitis
103

Foreign body ( most likely )


keratitis
657- Pt talking to doctor and the pt look to his right side most of the time,
when the doctor asked him why is that? He said that his mother is there
but in fact no one is there, after asking the pt family they said that the
mother died when he is child Dx?
Visual hallucination (Or may be the doctor is blind )
Auditory hallucination
psychosis
__________________________________________________________________________
__________________________
658-Child after his father died start to talk to himself , walk in the street
naked when the family asked him he said that his father asked him to do
that , he suffer from those things 3 days after that he is now completely
normal and he do not remember much about what he did
Dx ???????????????????????
Schizophrenia
Schizoaffective
Schizophreniform
Psychosis
There was a fifth choice I do not remember it, I think they make from
his father death a cause.
+ve symptoms: Hallucinations (most often auditory), delusions, disorganizedspeech,
bizarre behavior, and thought disorder.
-ve symptoms: Flat affect, emotional reactivity, poverty of speech, lackof purposeful
actions, and anhedonia.

Schizophreniform disorder: Symptoms of schizophrenia with a


duration of < 6 months.
Schizoaffective disorder: Combines the symptoms of
schizophrenia with a major affective disorder (major depressive
disorder or bipolar disorder).
Axis I: Psychiatric disorders.
Axis II: Personality disorders and mental retardation.
Axis III: Physical and medical problems.
Axis IV: Social and environmental problems/ stressors.
Axis V: The Global Assessment of Functioning (GAF).
Delusion: A fixed false idiosyncratic belief.
Hallucination: Perception of an object or event without an
existing external stimulus.
Illusion: False perception of an actual external stimulus.
Evolution of EPS :
104

4 hours: Acute dystonia , 4 days: Akinesia , 4 weeks: Akathisia ,


4 months: Tardive dyskinesia
659- Pt with foreign body sensation in the eye , after the removal of the
foreign body it was insect ttt?
Local antibiotic
Local steroid
Systemic antibiotic
Systemic steroid
660- Q22/ Child with white yellow mouth lips lesion in erythmatous base
with gingivitis Dx?
HSV ( most likely )
EBV
CMV
__________________________________________________________________________
__________________________
661- Pregnant never did check up before, her baby born with
hepatosplenomegaly and jaundice imp.
Rubella
CMV
HSV
Toxoplasmosis
662- Old Pt with abnormal ear sensation and fullness, hx of vertigo and
progressive hearing loss , invx low frequency sensorial hearing loss Dx
imp.
Acoustic neuroma
Neuritis
Menieres disease
Mnires Disease :
o A cause of recurrent vertigo with auditory symptoms that affects at
least 1 in 500 in the United States. More common among females .
o Hx/PE: Presents with recurrent episodes of severe vertigo, h earing
loss, tinnitus, or ear fullness, often lasting hours to days. Nausea and
vomiting are typical. Patients progressively lose low-frequency
hearing over years and may become deaf on the affected side.
663- The most difficult mode of transmission to prevent is
Person to person
Air droplet
Vector
http://www.prep4usmle.com/forum/thread/32772/
664- Pt took high dose of acetaminophen C/O nausea vomiting Lab
increase alkaline phosphatase and bilirubin which organ is affected?
Liver
Brain
105

Gastro
665- Female pt with hypothyroidism, TSH high But he did not give the
total T4 nor free , pulse normal BP normal she is in thyroxin what you will
do? Imp.
Increase thyroxin follows after 6 months
Increase thyroxin follows after 3 months
Decrease thyroxin follows after 6 months
Decrease thyroxin follows after 3 months
666- Pt with polycethemia vera the cause of bleeding in this pt is
Increase viscosity
Low platelets

Thromboses and bleeding are frequent in persons with polycythemia vera (PV) and MPD, and they result from
the disruption of hemostatic mechanisms because of (1) an increased level of red blood cells and (2) an
elevation of the platelet count. There are findings that indicate the additional roles of tissue factor and
polymorphonuclear leukocytes (PMLs) in clotting, the platelet surface as a contributor to phospholipiddependent coagulation reactions, and the entity of microparticles. Tissue factor is also synthesized by blood
leukocytes, the level of which is increased in persons with MPD, which can contribute to thrombosis.

Rusak et al evaluated the hemostatic balance in patients using thromboelastography and also studied the
effect of isovolemic erythrocytapheresis on patients with polycythemia vera. They concluded that
thromboelastography may help to assess the thrombotic risk in patients with polycythemia vera. [9]

Hyperhomocystinemia is a risk factor for thrombosis and is also widely prevalent in patients with MPD (35% in
controls, 56% in persons with PV).

Acquired von Willebrand syndrome is an established cause of bleeding in persons with MPD, accounting for
approximately 12-15% of all patients with this syndrome. von Willebrand syndrome is largely related to the
absorption of von Willebrand factor onto the platelets; reducing the platelet count should alleviate the bleeding
and the syndrome.

667- Pt k/c of SCA , the doctor planning to give him pneumococcal


vaccine which true ?
Pt need antibiotic when there is hx of contact even with vaccine
( most likely )
668-50 years old pt complaining of episodes of erectile dysfunction , hx of
stress attacks and he is now in stress what you will do ?
Follow relaxation strategy
Viagra
Ask for invx include testosterone
669-Rx. Of scabies in pregnant women:
Permethrin and sulfur solutions
670- Child with enuresis, what is the most important single test you will
do ?
Urine analysis
IVP
US
Blood culture

Urinalysis is the most important screening test in a child with enuresis.


Children with cystitis usually have WBCs or bacteria evident in the microscopic urinalysis.

106

Children with overactive bladder or dysfunctional voiding, urethral obstruction, neurogenic bladder, ectopic ureter, or diabetes mellitus are
predisposed to cystitis.
If the urinalysis findings suggest cystitis, urine should be sent for culture and sensitivity.
Urethral obstruction may be associated with RBCs in the urine.
The presence of glucose suggests diabetes mellitus.
A random or first-morning specific gravity greater than 1.020 excludes diabetes insipidus.))
emedicine
The most commonly accepted cause of nocturnal enuresis, but also the most difficult to prove, is delayed functional maturation of the
central nervous system, which reduces the child's ability to inhibit bladder emptying at night.
...........
medscape

671- Old male pt came with fever, abd pain, diarrhea , loss of weight , +
ve occult blood , Labs shows that the pt infected with streptococcus bovis
, what you will do ?????
Give antibiotic
ORS
Abd X-Ray
Colonoscopy
Metronidazole
_Colonoscopy should be performed in all patients with S bovis bacteremia or endocarditis. (emedicine)
__________________________________________________________________________
_________________________
672-Mallory weiss syndrome
Mostly need surgery
Mostly the bleeding stops spontaneously
Associated with high mortality
673- Female pt around 35 years old, hx of thromboembolic disease, what
type of reversible contraceptive she can use imp.
OCP
Mini pills (no estrogen)
IUCD
674- Child with fever and runny nose, conjunctivitis and cough then he developed
Maculopapular rash started in his face and descend to involve the rest of the
body:this is case of measles
a. EBV
b. Cocxaci virus
c. Rubella virus
d. Vaccini virus
The first sign of measles is usually a high fever (often >104 o F [40o C]) that typically lasts 4-7 days. This prodromal
phase is marked by malaise, fever, anorexia, and the classic triad of conjunctivitis (see the image below), cough, and
coryza (the 3 Cs). Other possible associated symptoms include photophobia, periorbital edema, and myalgias.
Blanching, erythematous macules and papules begin on the face at the hairline, on the sides of the neck, and behind
the ears (see the images below). Within 48 hours, they coalesce into patches and plaques that spread
cephalocaudally to the trunk and extremities, including the palms and soles, while beginning to regress
cephalocaudally, starting from the head and neck. Lesion density is greatest above the shoulders, where macular

107

lesions may coalesce. The eruption may also be petechial or ecchymotic in nature.

675-Avascular necrosis detect clinically AFTER


1/ 3 month
2/ 6 month??
3/ 9 month
4/ 11 month
5/ 15 month
676- - 5 yr old adopted child their recently parents brought him to you with white
nasal discharge. He is known case of SCA. What you will do to him:
a. Give prophylactic penicillin
b.
c.
677- Wound, with greenish discharge, Gram + ve in long chain?
Streptococcus
Proteus
Chlamydia
678- Female pt present with dysuria, urine analysis shows epithelial cast
Contaminated sample
Chlamydia urethritis
Kidney disease
Cervical disease
679- During the third trimester of pregnancy , all of the following changes occur
normally except
Decrease paco2
Decrease in wbcs
Reduced gastric emptying rate
Diminshed residual lung volume
Diminshed pelvic ligament tension
Pregnancy in the final month and labor may be associated with increased WBC levels.

http://labtestsonline.org/understanding/analytes/wbc/tab/test
__________________________________________________________________________
__________________________
680- The physiologic hypervolemia of pregnancy has clinical significance in the
management of severely injured , gravid women by:
108

Reduced the need for blood transfusion .


Increase the risk of pul. Edema .
Complicating the management of closed head injury
Reducing the volume of crystalloid required for resuscitation
Increasing the volume of blood loss to produce maternal hypotention
General Approach to the Trauma Patient

The primary initial goal in treating a pregnant trauma victim is to stabilize the mother's condition. The priorities for
treatment of an injured pregnant patient remain the same as those for the nonpregnant patient.
Primary Survey
As with any other injured patient, the primary survey of the injured pregnant patient addresses the airway/cervical
spine control, breathing and circulation (ABC; volume replacement/hemorrhage control), with the mother receiving
treatment priority. Supplemental oxygen is essential to prevent maternal and fetal hypoxia. Severe trauma
stimulates maternal catecholamine release, which causes uteroplacental vasoconstriction and compromised fetal
circulation. Prevention of aortocaval compression is also essential to optimize maternal and fetal hemodynamics.
Pregnant patients beyond 20 weeks' gestation should not be left supine during the initial assessment. Left uterine
displacement should be used by tilting the backboard to the left or as a final measure, the uterus can be manually
displaced.
Hypovolemia should be suspected before it becomes apparent because of the relative pregnancy induced
hypervolemia and hemodilution that may mask significant blood losses. Aggressive volume resuscitation is
encouraged even for normotensive patients.
http://www.trauma.org/archive/resus/pregnancytrauma.html

Anemia of chronic disease will show imp.


high ferritin high iron low TIBC
Low ferritin low iron high TIBC
high ferritin low iron low TIBC ( this Q may come with ferritin or
without )
Low ferritin high iron low TIBC
__________________________________________________________________________
__________________________682- Iron deficiency anemia will show
Low ferritin low iron low TIBC
Low ferritin low iron high TIBC
high ferritin low iron low TIBC
Low ferritin high iron low TIBC
681

In anemia of chronic disease without iron deficiency, ferritin levels should be normal or high,
reflecting the fact that iron is stored within cells, and ferritin is being produced as an acute
109

phase reactant but the cells are not releasing their iron. In iron deficiency anemia ferritin should
be low.[5]

TIBC should be high in genuine iron deficiency, reflecting efforts by the body to produce
more transferrin and bind up as much iron as possible; TIBC should be low or normal in
anemia of chronic disease.

683- Establishing to diagnosis shock must include:


Hypoxemia
Hypotention
Acidosis
Increase vascular resistance
Evidance of inadequate organ perfusion(FROM ATLS)
684- 35 yo male pt K/C of allergic rhinitis and bronchial asthma poorly controlled
presented with history of skin rash ,diffuse severe abdominal pain and hand joints
pain for 2days , on examination there are diffuse purpuric skin rash and small joint
tenderness with mild effusion , the most likely diagnosis is ...
Churg strauss syndrome
685- 65 yo female pt who has a 10y history of symmetrical polyarthritis is admitted
to the hospital , examination reveals splenomegally, ulceration over lateral malleoli
and synovitis of the wrists, shoulders and knees . initial investigation shows WBC
2500 ,the most likely diagnosis is ....
Felty's syndrome !!
___________________________________________________________________
_______
686- Patient with histor
y of AF + MI , the best prevetion for stroke is : ?
a- Warfarin
b. Surgery procedure
c.
c- Shunt
687- What is the side effect of steroid on the eye ?
a- Glaucoma .
a. Cataract .(POSTERIOR SUBCAPSULAR CATARCT) AND GLAUCOMA
IN THOSE WITH FAMILY HISTORY
110

b. Keratoconus .
688- Young male pt present e and pain for....... He denied any hx of Truma or any
medical illness before
On ex..RR32b/min. Pulse .....
Abdomen not tender or garding
By invx
WBC e in normal
K >5
Na......
RBS 23mmol
What most important invx:
U/s
ABG
urine dipstick
689- The best investigation for kidney function :
a- 24 h collect urine
b- Creatinine clearance
690- Patient came to you with small swelling under his eye , on examination he
have inflammation in lacrimal duct , you refer him to ophthalmologist before that
what you will give him ?
a- Topical steroid
b- Topical antibiotic
c- General antibiotic
691- What is true regarding spountinous abortion:
Can lead to infertility in future
Usually fused by any utrine abnormality
That occur on 2ed trimester
..
692- The best investigation for acute diveticolitis is
:
a- US
111

b- Barium enema
c- CT
d- Colonscopy
e- Sigmidscopy
Diverticulitis:
o
Chest X-ray with the patient upright can aid detection of pneumoperitoneum.
o
Abdominal X-rays may demonstrate small or large bowel dilation or ileus, pneumoperitoneum, bowel obstruction,
o

or soft tissue densities suggesting abscesses.


Contrast enemas: limited value; findings suggestive of diverticulitis include extravasated contrast material outlining

an abscess cavity, intramural sinus tract or fistula.


CT scanning with intravenous, oral or rectal contrast: sensitivities and specificities for CT are significantly better

than for contrast enemas. When an abscess is suspected, CT scanning is the best modality for making the diagnosis and
following its course.
Because of risk of perforation, endoscopy is generally avoided in initial assessment of the patient with acute
diverticulitis. Its use should be restricted to situations when the diagnosis in unclear, to exclude other possible
diagnoses.

http://www.patient.co.uk/doctor/Diverticular-Disease.htm
693- Food poisoning , group of people came with diarrhea and vomiting diagnosis is:
a. Staphiloccous aureus poisoning
b- Salmonella poisoning
Usually if patients came with food poisoning after few hours of ingestion mostly the
cause is Staph due to preformed toxin, if they started to have symptoms after one
day of ingestion you are right compylobacter salmonella shigella or E. coli
694- Patient with Rhumatoid arthritis on hand X-Ray there is swelling what you will
do for him
a- NSAID
b. Injection steroid
c.

NSAIDs interfere with prostaglandin synthesis through inhibition of the enzyme cyclooxygenase (COX), thus
reducing swelling and pain. However, they do not retard joint destruction and, therefore, when used alone, are
not sufficient to treat RA

695- what is the drug that will preserve the histology in primary liver cirrhosis:
696- propylthiouracil drug contraindicated with :
*Maternal HTN
*Maternal DM
112

*Maternal asthma
697- Differant between uvitis and kertits
:
Dec visual acuty
Photophbia
Periorbiatl edma
in keratitis
Cillry flush
698- 55 y/o male,, c/o angina and syncope on exertion ,, normal ejection fraction ,,
normal coronary arteries ,, there is only calcified aortic valve with total area < .75 cm
,, the rest of examination and investigations are normal ..
Wt is ur manag. :
Avoid exertion
Medical therapy(for angina)
Aortic ballon dilation
Aortoc valve replacrment
699- 10 months old baby came to the clinic with his mother , she breastfeed him 3
times a day ,, she is known cace of epilepsy on phenobarbital,,,,,, What u going to
tell her final answer
Stope breastfeeding immediately
Weaning over 2 weeks period
Breastfeed after 8 h from taking the drug ??
Respond to what the mother and child wish
henobarbital in breastmilk apparently can decrease withdrawal symptoms in infants who were exposed ..
If phenobarbital is required by the mother, it is not necessarily a reason to discontinue breastfeeding. Monitor the infant for drowsiness,
adequate weight gain, and developmental milestones
i http://www.drugs.com/breastfeeding/phenobarbital.html

700- 65 yo male pt presented with history of backache and fatigue for the last 3
month , examination showed mild tenderness in lumbosacral region , initial
investigation revealed the following :
Hb 9
ESR 80
X ray spine showed osteolytic lesion, the most likely diagnosis is ......
Solitary myloma
113

701- medical therapy in ectopic pregnancy >>>


methotrexate
___________________________________________________________________
_______
702-Best for diagnosis E.P>>
serial beta HCG
___________________________________________________________________
_______
703-True about DUB >>
can occure in adolescent girls

704-Best anti HTN in pregnancy >>


hydralazine
methyldopa is the first line in treatment of pregnancy
___________________________________________________________________
_______
705-Pregnant ,, smoker ,, h/o trauma >> dark red vaginal bleeding ,, FHR 150
uterine contractions ...diagnosis :
Uterine contusion
Abruption
___________________________________________________________________
_______
706MC cause of 2ry amenorrhea e high LH & FSH >>
menopause
___________________________________________________________________
______
707-MC cause of milk discharge in non lactating women >>
prolactinoma
___________________________________________________________________
_______
114

708-RTA case found damage of sup femoral artery 5cm in diameter , do :


*end to end anastmosing of the artery
*artery graft
*venous graft
___________________________________________________________________
_______
709- Pt after URTI later on develop proximal muscle weakness , most probably:
*gurean barre syndrome.??
*osteoarthritis .??
710- pt have normal Na , Cl , urine PH ALL electrolyte were normal except HCO3
was low : ( serum PH not mention )
*met acidosis ( not sure )
*met alkalosis
*res acidosis
*res alkaloso .?? (compensated)
___________________________________________________________________
______

711- 70 ys old with sever muscle pain , diarrhea , disorientation , he is in diuretic the
cause :
*hyponatremia
*hypokalemia
712- Newly diagnosed Type 2 diabetic advice for diet and exercise but still gain
weight , which medication you want to add
1.insuline
2. metformin
3. sulfonylurea
713- mucopurulent discharge :
bacterial conjunctivitis
714- HCC :
10 % with liver disease
115

with chronic liver diseases


never come with smoking
Hepatocellular carcinoma (HCC, also called malignant hepatoma) is the most common type of liver cancer.
Most cases of HCC are secondary to either a viral hepatitide infection (hepatitis B or C)
or cirrhosis (alcoholism being the most common cause of hepatic cirrhosis).[1]
Compared to other cancers, HCC is quite a rare tumor in the United States. In countries where hepatitis is
not endemic, most malignantcancers in the liver are not primary HCC but metastasis (spread) of cancer from
elsewhere in the body, e.g., the colon. Treatment options of HCC and prognosis are dependent on many
factors but especially on tumor size and staging. Tumor grade is also important.
The main risk factors for hepatocellular carcinoma are:

Alcoholism

Hepatitis B

Hepatitis C (25% of causes globally)[3]

Aflatoxin

Cirrhosis of the liver

Hemochromatosis

Wilsons disease (while some theorize the risk increases,[4] case studies are rare[5] and
suggest the opposite where Wilson's disease actually may confer protection [6])

Type 2 Diabetes (probably aided by obesity)[7]


Ocp,tobacoo
Hepatocellular carcinoma (HCC) most commonly appears in a patient with chronic viral hepatitis (hepatitis B or
hepatitis C, 20%) or/and with cirrhosis (about 80%). These patients commonly undergo surveillance
with ultrasound due to the cost-effectiveness.
In patients with a higher suspicion of HCC (such as rising alphafetoprotein and desgammacarboxyprothrombin levels),
the best method of diagnosis involves a CTscan of the abdomen using intravenouscontrast agent and three-phase
scanning (before contrast administration, immediately after contrast administration, and again after a delay) to
increase the ability of theradiologist to detect small or subtle tumors. It is important to optimize the parameters of the
CT examination, because the underlying liver disease that most HCC patients have can make the findings more
difficult to appreciate.

715- female with problem in school -manula removal of her hair


(baldness)
: ((Trichotillomani )))
Trichotillomania is hair loss from repeated urges to pull or twist the hair until it breaks off. Patients are unable to stop
this behavior, even as their hair becomes thinner.

716- pregnant 6 days in CS - staining in her throbs from abdomen :


Fascial dehiscence
717-16 week , polydipsia polyurea , less than 126 mg fasting ,, 6.8 :
impaird DM
116

IFG: ( 6.1-7.0 mmol)


IGT : ( 7.8-11.1 mmol/l, 2h after 75g )
- First step: One-hour 50-g glucose challenge test; venous plasma glucose
is measured one hour later (at 2428 weeks). Values 140 mg/dL are
considered abnormal.
- Next step: Confi rm with an oral three-hour (100-g) glucose tolerance
test showing any two of the following: fasting > 95 mg/dL; one hour >
180 mg/ dL; two hours > 155 mg/dL; three hours > 140 mg/dL.
718- Pancreatitis :
increase by lying down
719-50 years , back pain , x ray sowed lytic lesion :
bone scan
1-bone marrow biopsy 2-protien electrophoresis of blood and
urine>paraprotien
720- exaggerated reflex in jaw , no fasculation , difficlty in swallowing :
pseudobulbar palsy
Pseudobulbar palsy results from an upper motor neuron lesion to the
corticobulbar pathways in the pyramidal tract. Patients have difficulty
chewing, swallowing and demonstrate slurred speech (often initial
presentation). Individuals with pseudobulbar palsy also demonstrate
inappropriate emotional outbursts.
S/S :
* Speech is slow, thick and indistinct - Gag reflex is normal,
exaggerated or absent - Tongue is small, stiff and spastic - Jaw jerk is
brisk - upper motor neuron lesion of the limbs - Dysphagia (difficulty in
swallowing) - Labile affect Dysarthria - Uncontrollable laughing or crying
Bulbar palsy refers to bilateral impairment of function of the lower cranial
nerves IX, X, XI and XII, which occurs due to lower motor neuron lesion
either at nuclear or fascicular level in the medulla oblongata or from
bilateral lesions of the lower cranial nerves outside the brainstem.[1]
S/S :
dysphagia (difficulty in swallowing) - difficulty in chewing - nasal
regurgitation - slurring of speech - choking on liquids - Nasal speech
lacking in modulation and difficulty with all consonants - Tongue is
atrophic and shows fasciculations - Dribbling of saliva - Weakness of the
soft palate, examined by asking the patient to say aah - The jaw jerk is
normal or absent - The gag reflex is absent - lower motor neuron lesions
of the limbs.
721- High senstive & specific for urolithasis :
CT scan
722- child in ER , with dyspnea , tachy pnea , subepiglottic narrowing in xray :
117

If thumb sign : epiglottitis , if steeple sign : croup


CROUP Diagnosis :
A frontal X-ray of the neck is not routinely performed, but if it is done, it
may show a characteristic narrowing of the trachea, called the steeple
sign. The steeple sign is suggestive of the diagnosis of CROUP , but is
absent in half of cases.
723- female with breast cancer :before 5 years received chmeotherpy .
diagnosed now CLL , causes of CML : risk factor .
The etiology is unknown, although there is some genetic contribution, as
fi rst-degree relatives of patients with CLL are three times more likely
than others to develop a lymphoid malignancy. Primarily affects older
adults (median age 65); the male-to-female ratio is 2:1.
724- antipsychotic drug side effect for onset :
4 hours: Acute dystonia , 4 days: Akinesia , 4 weeks: Akathisia , 4
months: Tardive dyskinesia (often permanent)
__________________________________________________________________________
725- chronic use of alcohol : first drug to give pt :
thiamine .
. All patients
being treated for AW should be given
100 milligrams (mg) of thiamine as
soon as treatment begins and daily
during the withdrawal period.
http://pubs.niaaa.nih.gov/publications/arh22-1/38-43.pdf
__________________________________________________________________________
726-female with vaginal bleeding , abdominal pain : first Inx :
US
Vaginal Examination
727-stroke pt , most propable cause :
Polycethmia vera
Sickle ceLl anemia
2 ry polycethmia .
__________________________________________________________________________
728-old male , back pain , ex is normal : gave him steroid , come again
with vesicle from back to abdomen : VZV
729- Female , Rt hand , lateral two radial styloid processes pain , since
month increase progressively , CS , ttt of De Quervain Tenosynovitis
+ve Finkelstein test initial ttt :
Nerve decomperrison
cast upper joint
118

cast with thumb raised


Initial treatment for DeQuervain's syndrome is nonoperative : first
thumb-spica splint , NSAIDS may also be of value , corticosteriod
injection into the first dorsal compartment may provide sustained
relief .
o In one study, Weiss et al. reported that injection was just as
effective in DeQuervain's syndrome as splinting alone or
splinting combined with injection. In another study of 63
patients, 71% of patients had sustained relief of their
symptoms with one first dorsal compartment injection.
730- male with auscultation , not clear , left sterna border , scratching
sound , vein distened in neck , muffled heart sound :
Cardiac tamponade
percarditis
731- Acanthosis Nigricans associated with :
polycystic ovary syndrome
732- celiac disease which not cause it :
rice&corn
oat
wheel
gluten
733- old man with generalized abdominal pain T:38.2,abscent bowel
sound,x ray:dilated small bowel and part of the transverse colon,no fluid
level:
Pancreatitis
perforated peptic ulcer
bacterial colitis
intestinal obstruction

Sentinel loop sign:isolated dilatation of a segment of gut consisting of jejunum, transverse colon or duodenum. Colon cutoff
sign:gas distition rt colon that abruptly stopes in mid or LF colon due spasm adjacent to inflammation

734- baby with tonic clonic convulsions,what drug you'll give the mother
to take home if ther is another seizure:
Diazepam
phenytoin
Phenobarb
735- Significant DPL direct peritoneal lavage in diagnosis :
10 ml blood or 100,000 RBC or 500 WBC
A positive DPL in an adult classically requires one of the following
results:
RBC : > 100,000/mm3
blood : 10 ml
Amylase level (IU/L) : 20
Alkaline phosphatase level (IU/L) : 3
WBCs (per mm3) :>500
119

the presence of enteric/vegetable matter


736-Initial management for Frostbite patient :
Debridement
b.blocker
corticosretoid
immersion in 40 C .
737- Patient with ACEI , was not uncontrolled , what do will add to the
patient :
B.Blocker
ARBs
CCBs
Thiazide
Frusemide
__________________________________________________________________________
738- Patient with HTN , discovered DM 2 ,what the medication will add to
him :
Blocker
ARBs
CCBs
Thiazide
ACEIs
739- Patient with hypertensive retinopathy grade 2 AV nicking , normal
BP , no decrease in vision , with cupping of optic disc , what will do do the
patient :
Reassurance , the problem is benign
Convert hitm to ophthalmologis
Laser operation
740- Patient with sudden skin eruption over face and neck then palm and
sole
????????
Erythema multiform
Drug eruption
Measles
If syphilis is there it is more accurate
741-100 - Patient witn BA , using B.agonist if needed , now become more
sever , attacks come 5 times in week., you will give patient , I expect
Moderate persistent

Low dose steroid inhaler


Oral steroids
Ipratorpium bromide
Long acting B.agonist
Large dose steroid inhaler
120

742- One of them causes conductive hearing loss :


Acute ottis media
Syphillis
Meneria disease
743- CSF : normal protein , decrease glucose , turbid in color :
Viral menengititis
Tubercular meningitis
Pyogenic meningitis
Encephalitis
___
Cause
Appearanc Polymorphonucle Lymphocyte Protein Glucose
e
ar cell
Pyogenic Yellowish, Markedly
bacterial
turbid
increased
meningitis

Slightly
Markedly Decrease
increased or increased d
Normal

Viral
Clear fluid Slightly
meningitis
increased or
Normal

Markedly
increased

Slightly Normal
increased
or Normal

Tuberculou Yellowish
s
and
meningitis viscous

Slightly
increased or
Normal

Markedly
increased

Increased Decrease
d

Fungal
Yellowish
meningitis and
viscous

Slightly
increased or
Normal

Markedly
increased

Slightly Normal or
increased decrease
or Normal d

__________________________________________________________________________
744- Patient with vomiting and diarrhea and moderate dehydration, how
to treat:

ORS only
745- in knee examination : +ve lechman test indicate injury :
Anterior cruciate ligament
746-2 weeks after delivery , mother said that the baby , he will die ,
baby lack healthy , what is diagnosis :??
Post partum no bonding
Post partum psychosis
Post partum depression
747- antidepressant in elderly :
Will take time to see effect
"I make sure that patients and their familywho I always try to includeunderstand how long it can take to
respond to an antidepressant. People need clear expectations. The drugs take longer to work in older people than
younger ones, usually 8 to 12 weeks. The longer they stay on the medicine, the more improvement they are likely
to see,"

http://www.health.com/health/condition-article/0,,20188909_2,00.html
121

748- children on chemotharpy , he developed septicemia after introduce


IV canula , what is causative organisms : imp.??
Hib
Psudeomonas??
E.coli
strept
klebsiella
749- 1-2012 pt came e" painful rectal spasm , diaphorisis , tachycardia at night what
the DX :
a. thrombosed hemorhoid
b. Proctalgia Fugax
c. >>syndrome
Proctalgia fugax (or levator syndrome) is a severe,
episodic, rectal and sacrococcygeal pain.[1] It can be caused by cramp of
thepubococcygeus or levator ani muscles.[2
It most often occurs in the middle of the night [3] and lasts from seconds to minutes,[4] an indicator for the differential
diagnosis of levator ani syndrome, which presents as pain and aching lasting twenty minutes or longer. In a study
published in 2007 involving 1809 patients, the attacks occurred in the daytime, (33 percent) as well as at night (33
percent) and the average number of attacks was 13. Onset can be in childhood, however, in multiple studies the
average age of onset was 45. Many studies showed that women are affected more commonly than men. [5]
During an episode, which sometimes occurs after orgasm, the patient feels spasm-like, sometimes excruciating, pain
in the anus, often misinterpreted as a need to defecate. Simultaneous stimulation of the local autonomic system can
cause erection in males. Because of the high incident of internal anal sphincter thickening with the disorder, it is
thought to be a disorder of the internal anal sphincter or that it is a neuralgia of pudendal nerves. It is recurrent and
there is also no known cure. However, some studies show effective use of botulinumtoxin, pudendal nerve block,
and calciumchannelblockers. It is not known to be linked to any disease process and data on the number of people
afflicted varies, but is more prevalent than usually thought.
The pain episode subsides by itself as the spasm disappears on its own, but may reoccur. [4]
Sometimes there is a drop in blood pressure that may cause loss of consciousness and possible injury. Staying down
is suggested if in bed and laying down is recommended.

750- old pt , e hx of MI 2 weeks back and discharge from hospital 24 hrs prior to his
presentation <<<came with sudden lower limb pain and numbness ,on ex the limb
pale , cold >>the other limb normal what is the DX :
a-Acute artery thrombosis
b- acute artery embolus
d. DVT
e. D- ???
751-7- Neonate with mass on his eye :
a-Neuroblastoma
b-Leukemia
122

(Neuroblastoma
is the most common extracranial solid tumor in infancy and the third most common
pediatric malignancy after leukemia and central nervous system (CNS) tumors.
It is an embryonal malignancy of the sympathetic nervous system arising from
neuroblasts (pluripotent sympathetic cells). Signs and symptoms of neuroblastoma
vary with site of presentation. Generally, symptoms include abdominal pain, emesis,
weight loss, anorexia, fatigue, and bone pain. At diagnosis, the site of neuroblastoma
is predictably age-dependent. Infants often present with compression of the
sympathetic ganglia in the thoracic region, which might result, for example, in Horner
syndrome (myosis, anhydrosis, and ptosis) or superior vena cava syndrome. Older
children typically present with abdominal symptoms because, as stated above, more
than 40% of neuroblastomas are adrenal in origin. Children who are preschool aged
should have working differential diagnoses for an abdominal mass, including
lymphoma, hepatoblastoma, rhabdomyosarcoma, renal cell carcinoma, and
neuroblastoma.

http://emedicine.medscape.com/article/988284-overview#a0104
Leukemia
is a type of cancer of the blood or bone marrow characterized by an abnormal
increase of immature white blood cells called "blasts".In most patients, a diagnosis of
leukemia has been made before presenting to an ophthalmologist. However, in some
patients, ocular symptoms and examination lead to a diagnosis of leukemia.Orbital
manifestations
o Leukemic cells may infiltrate the orbit during the course of acute or chronic leukemia.
Unusual orbital involvement with leukemia has been reported to include infiltration of the
lacrimal gland and drainage system, rectus muscles, and dermis.
o Orbital involvement in children is more common in acute leukemias, whereas orbital
involvement in adults is more common in chronic leukemias.
o The leukemic infiltrate may range from insignificant, where it is virtually asymptomatic, to a
space-occupying lesion with its concomitant symptoms.
o The patient may have proptosis, ecchymosis, chemosis, diplopia, visual disturbance, or
motility disturbances.
o In children, the orbital involvement is characterized by an acute and rapid process that may
be confused with orbital cellulitis. In general, these infiltrates are bilateral and do not destroy
bone. Granulocytic sarcoma of the orbit, also known as chloroma, is an extramedullary form
of myelogenous leukemia.
o Unilateral, painless proptosis develops over weeks to months prior to a diagnosis of
leukemia. Eyelid redness or violaceous discoloration may be present, which turns into
ecchymosis that may be confused with rhabdomyosarcoma or metastatic neuroblastoma. If
AML or CML is already present, then a rapid and fulminant bilateral proptosis is
characteristic.
o
o http://emedicine.medscape.com/article/1201870-clinical#a0217

123

752- Pt with 3 month hx of feeling like depression, recently


employ what you will do:
a- Tricyclic
b
c-Psychotherapy support
d- Moi
the recommended treatment for adjustment disorder is psychotherapy
____________________________________________________________________________________
______________________________

??753- WRIST WITH PAPULOVESICULAR RASH 12


CHICKEN POX >> Chickenpox is usually diagnosed clinically on the basis of the
characteristic rash and successive crops of lesions.
Small, erythematous macules appear on the scalp, face, trunk, and proximal limbs,
with rapid sequential progression over 12-14 hours to papules, clear vesicles, and
pustules and subsequent central umbilication and crust formation.
Vesicles may appear on the palms and the soles and on the mucous membranes,
together with painful, shallow, oropharyngeal or urogenital ulcers. Intense pruritus
commonly accompanies the vesicular stage of the rash.

Vesicular eruption on the trunk demonstrating papules, vesicles, and crusts

Dewdrop on rose petal characteristic vesicle of chickenpox

SCABES>>superficial burrows

124

A typical linear burrow on the flexor forearm

A subtle linear burrow accompanied by erythematous papules on the sole of the foot
in a child with scabies.

Erythematous papules and papulovesicles on the flexor wrist.

ATROFIC DERMATITIS>> red, flaky and very itchy. The skin on the flexural surfaces
of the joints (for example inner sides of elbows and knees) are the most commonly
affected regions in people.

Flexural involvement in childhood atopic dermatitis.

Dermatitis herpitiform>>is characterized by grouped excoriations; erythematous,


urticarial plaques; and papules with vesicles.

125

Classic vesicles of dermatitis herpetiformis.

754- CHILD SEVERLY ILL AND FEVER FOR 2 DAYS ANOREXIA NAUSEA
VOMITING THEN PETECHIA RASH APEAR IN TRUNK AND SPREAD
IN THE BODY ??
MEASELS
MENINGOCOCCAL MENINGITIS
MOUNTAIN FEVER
Varicella low grade fever for 2 days then rash in the trunk then it spread in the body
755- Patient on amitriptyline 30mg before bed time wake up with sever headache
and confusion what is the appropriate action ?
Shift him to ssri
Change the dose to 10mg 3time daily
756-Young patient with decrease hearing and familly history of hearing loss ear
examination normal rene and weber test revealed that bone conduction is more than
air conduction what would you do ?
Till him it is only temporal
Till him there is no ttt for this condition
Refer to audiometry
Refer to otolaryngologists
757- which one of these drugs causing hypertensive crisis when it is not
stoped gradually?
diltiazim>>Antidysrhythmics, IV; Calcium Channel Blockers,
colonidine>>Rebound hypertension on withdrawal
b blocker
Clonidine suppresses sympathetic outflow resulting in lower blood
pressure, but sudden discontinuation can cause rebound
hypertension due to a rebound in sympathetic outflow.
Clonidinetherapyshouldgenerallybegraduallytaperedoffwhendiscontinuingtherapytoavoidreboundeffectsfromoccurring.
Treatmentofclonidinewithdrawalhypertensiondependsontheseverityofthecondition.Reintroductionofclonidineformild
cases,alphaandbetablockersformoreurgentsituations.Betablockersnevershouldbeusedalonetotreatclonidinewithdrawal
asalphavasoconstrictionwouldstillcontinue
________________________________________________________________________________________________________

758- @1year old massive hepatosplenomegaly , brown skin nodule, lateral neck
lump investigation to diagnose
126

Ebv serology
Lp
bone marow aspiration
liver biopsy
759- child came with hematuria 15 RBC HPF , all examination normal what will you
do ?
a. urine cytology
b. renal bipsy
c cystosocopy
c. repeat urine for RBC and protein
760- 15 years old with palpitation and fatigue. Investigation
showed RT ventricularhypertrophy, RT ventricular
overload and right branch block what is the diagnosis :
a- ASD
b- VSD
Complications of VSDs include the following:
Heart block secondary to intracardiac repair

c- Coartaction of aorta

761- child came withor Toeing-In , set in W shape , when walk both feet and knee
inward with 20 degree , both femur inwarr rotation 70 degree <<like this """ what the
Dx :
a. metatarsus adductus
b-femoral anteversion (femoral torsion )>>
Femoral anteversion
Normalfemoralanteversionis40inthenewbornanddecreasesto10bytheageof8years.Theacetabulumisangledforward
15.Femoralanteversiondoesnotincreasetheriskofarthritisofthehip.Spontaneousimprovementintheanatomicpositioncan
occuruntilthepatientisaged8yearsandcanfurthercorrectbyimprovingthegaitthroughconsciouseffortuntiladolescence.

Femoral anteversion is the axial angle between the plane of the neck of the femur and
the femoral condyles. It can be clinically deduced by measuring the hip rotation.
Normal range of external rotation is 45-70, and internal rotation is 10-45. As femoral
anteversion increases, the amount of internal rotation increases and external rotation
decreases. These children can have as much as 90 of internal rotation and 0 of
external rotation. They sit in the W position with their legs turned out (a position not
attainable by normal adults), but they cannot sit cross-legged

c-???
762- anticoagulation prescribed for
- one month
- 6 months
- 6 weeks
- one year
127

763- child with moderate persistant BA On bronch.dilat inhaler. Presented


with acute exacerbation what will you add in ttt:
Corticosteroid inhaler
Ipratropum bromide inhaler
childrenwithasthmaaredividedinto3groupsbasedonage:04y,511y,12yandolder.

For all patients, quick-relief medications include rapid-acting beta2-agonists as


needed for symptoms. The intensity of treatment depends on the severity of
symptoms. If rapid-acting beta2-agonists are used more than 2 days a week for
symptom relief (not including use of rapid-acting beta2-agonists for prevention of
exercise induce symptoms), stepping up treatment may be considered
Stepwise Approach to Asthma Medications
Intermittent
Persistent Asthma: Daily Medication
Asthma
Age Step 1

Step 2

< 5 Rapidy
acting
beta2agonist
prn

Low-dose inhaled Medium-dose


corticosteroid
ICS
(ICS)

5-11 Rapidy
acting
beta2agonist
prn

Low-dose ICS

12 y
or
olde
r

Step 3

Alternate
regimen:
cromolyn or
montelukast

Alternate
regimen:
cromolyn,
leukotriene
receptor
antagonist
(LTRA), or
theophylline

Rapid- Low-dose ICS


acting
beta2agonist as

Step 4

Step 5

Step 6

Medium-dose
ICS plus either
long-acting
beta2-agonist
(LABA) or
montelukast

High-dose ICS
plus either
LABA or
montelukast

High-dose ICS plus


either LABA or
montelukast; Oral
systemic
corticosteroid

High-dose ICS
plus LABA

High-dose ICS plus


LABA plus oral
systemic
corticosteroid

Alternate
regimen: highdose ICS plus
either LABA or
theophylline

Alternate regimen:
high-dose ICS plus
LRTA or
theophylline plus
systemic
corticosteroid

Either low-dose Medium-dose


ICS plus either ICS plus LABA
LABA, LTRA, or
theophylline OR
Medium-dose
Alternate
regimen:
medium-dose
ICS plus either
LTRA or
theophylline

Low-dose ICS
plus LABA OR
Medium-dose
ICS

Medium-dose High-dose ICS


ICS plus LABA plus LABA (and
consider
omalizumab for

128

High-dose ICS plus


either LABA plus
oral corticosteroid
(and consider

needed

Alternate
regimen:
cromolyn,
LTRA, or
theophylline

Alternate
regimen: lowdose ICS plus
either LTRA,
theophylline, or
zileuton

Alternate
regimen:
medium-dose
ICS plus either
LTRA,

129

patients with
allergies)

omalizumab for
patients with
allergies)

pt is a known case of moderate intermittent bronchial asthma. He is


using ventoline nebulizer. He develops 3 attacks per week. The drug to
be added is:
a) Increase prednisolone dose
b) Add long acting B agonist
c) Add ipratropium >>Anticholinergic bronchodilator
d) IV aminophylline
___________________________________________________________________
764- A 20 years old male who is a known asthmatic presented to the ER
with shortness of breath. PR 120, RR 30, PEFR 100/min. examination
revealed very quite chest. What is the most propable management?
a) Nubelized salbutamol
b) IV aminophyline
c) Pleural aspiration
d) Hemlich maneuver
e) Chest drain
765- about crohn's disease are true :
Inflammation Involve superfacial layer of intestine
Involve sigmoid and rectum
if (skip lesions).
Decrease incidence of colon cancer
130

The rectum is often spared. Transmural infl ammation is seen.


(correct)
766- old age male with h/o of gastric acidity relieved by antiacids ,now he
c/o forceful vomiting at night of food content >>>>> dx:
Gastric outlet obstruction?>>is a medical condition where there is an
obstruction at the level of the pylorus, which is the outlet of the stomach.

767- which drug increase incidence of reflux oesophagitis:


Thiophylline (correct)
Amoxicillin
Metoclopramide
Rantidine
Lansoprazole
__________________________________________________________________________
768- which drug cause hypertensive crises when used with tyramine :
SSRI
Tricyclic antidepressant
MAOI (correct)
769- old ptn with HTN and migrane ttt:
B blockers (correct)
ACE I
Ca blockers
770- the most common fracture in osteoporosis : ( I couldnt remember
the age)
Spinal fractures (compressed vertebrae)are the most common
osteoporotic-related fractures

Colles fracture (if prior 75 y)


Fracture neck of femur
shaft of femur
hip fracture(if over 75y)
771- male ptn with scaly fine papular rash on fornt of scalp,nose and
retroauricular..(i think tinea capitis) ttt is:
Ketoconazole cream
Oral augmentin
cream
tinea capitis : single or multiple patches of hair loss, sometimes
with a 'black dot' pattern (often with broken-off hairs), that may be
accompanied by inflammation, scaling, pustules, and itching .
Treatment : oral antifungal agent; griseofulvin is the most commonly
used drug, but other newer antimycotic drugs, such as terbinafine,
itraconazole, and fluconazole have started to gain acceptance.
dx : Wood's lamp examination

131

772- Gram stain reveal gram negative diplococcic (you must know about
causative org. of this type).
Ceftriaxone IM or cefepime PO one dose..(Nisseria gonorrhea) >>
is a species of Gram-negativecoffee bean-shaped diplococcibacteria responsible for
the sexually transmitted infectiongonorrhea.
ttt : ceftriaxone (a third-generation cephalosporin)

773- A side effect of L-Dopa:


a) Dyskinesia (correct)
b) ..
c) ..
d) ..
774-Aspirin & clopedogril indicated for:
a) A 55 year old man with acute coronary syndrome.
b) A 65 year old man with hx of MI
c) A 65 yo man with stroke.
d) A 65 yo man with CHF
e) After cardiac catheterization ??
775-Plavix & aspirin used for:
a) Pt with previous history of ischemic stroke
b) Pt going for angiogram
c) MI
d) Acute coronary syndrome
__________________________________________________________________________
776- A pt who is a KCO posterior MI presented with syncope. Examination
showed canon (a) wave with tachycardia, unreadable BP & wide QRS
complexes on ECG. The diagnosis is:
a) Atrioventricular re-entrant nodal tachycardia
b) Ventricular tachycardia
if the individual has a past history of a myocardial infarction, congestive heart
failure, or recent angina, the wide complex tachycardia is much more likely to
be ventricular tachycardia

c) Pre-existing AV block
d) Anterograde AV block
e) Bundle branch block
cannon wave It is associated with heart block, in particular third-degree
(complete) heart block

777- The drug with the least side effects for the treatment of SLE is:
a) NSAIDs (correct)>>Common side effects of NSAIDs can include:

stomach upset

heartburn

diarrhea

fluid retention

132

b) Methotrexate>>Immunosuppressant drug. Side effects may include:

nausea

vomiting

hair loss

bladder problems

decreased fertility

increased risk of cancer and infection

c)

Corticosteroid>>Short-term side effects of corticosteroids include:

swelling

increased appetite

weight gain
These side effects generally stop when the drug is stopped. It is dangerous (even life threatening) to
stop taking corticosteroids suddenly

Long-term side effects of corticosteroids can include:

stretch marks

weakened or damaged bones (osteoporosis and osteonecrosis)

high blood pressure

damage to the arteries

high blood sugar (diabetes)

infections

cataracts

d) Hydroxychloroquin >>Side effects of anti-malarials can include:

stomach upset

damage to the retina of the eye (rare)

___________________________________________________________________
778- A young male who is a known case of sickle cell anemia presented with
abdominal pain & joint pain. He is usually managed by hospitalization. Your
management is:
a) In-patient management & hospitalization
b) Out-patient management by NSAID
c) Hydration, analgesia, monitoring. (correct)
d) Narcotic opioids
133

___________________________________________________________________
779- A lot of bacteria produce toxins which are harmful. Which one of the
following is used in amiddirs:
a) Botulism??
b) Tetanus
c) Diphtheria
d) Staph aureus
780-

All
a)
b)
c)
d)

the following cause hyponatremia except:


DKA
Diabetes insipidus<<Diabetes insipidus causes HYPER (correct)
High vasopressin level
Heart failure

781- Warfarin is given to all the following except:


a) Young male with Atrial fibrillation & mitral stenosis
b) Male with AF & cardiomyopathy
c) Male with AF & prosthetic heart valve
d) Elderly male with normal heart (correct)

782-. A 24 Y/o man presented with 4 month Hx of diarrhea with streaks of


blood & mucous. Ulcerative colitis was confirmed by colonoscopy. The initial
therapy for this patient:(481)
a) oral corticostreiod xx
b) azathioprine
c) infleximabe
d) 5-Aminosalicylic acid
e) Sulfasalazine
Sulfasalazine has been a major agent in the therapy of mild to moderate UC
for over 50 years. In 1977, Mastan S. Kalsi et al. determined that 5aminosalicylic acid (5-ASA and mesalazine) was the therapeutically active in
sulfasalazine.[citation needed] Since then many 5-ASA compounds have been
developed with the aim of maintaining efficacy but reducing the common side
effects associated with the sulfapyridine moiety in sulfasalazine
http://emedicine.medscape.com/article/183084-treatment#aw2aab6b6b2

783- A hypothyroid pt on thyroxin had anorexia, dry cough, dyspnea & left
ventricular dysfunction. She had normal TSH & T4 levels, Hyperphosphatemia
& hypocalcemia. The diagnosis is:
a) Primary hypoparathyroidism
b) Secondary hypoparathyroidism xx
134

c) Hypopituitaritism
d) Uncontrolled hyperthyroidism
784-A 55 yr old man presenting with Hx of streakes of blood in stool and dull
pain on defecation that persists for half an hour after defecation, on
examination there was a 3x2 cm thrombosed mass at 3 oclock.What is the
management?
a) Sitz bath 5 times/ day.
b) Application of local anesthetic and incision.
c) Application of antibiotic
d) Band ligation and wait for it to fall
e) Application of local anesthetic ointment
_________________________________________________________________________
785- A 42 year old woman presented with a painful breast mass about 4 cm
in the upper lateral quadrant. It increases in size with the menstrual period.
Examination showed a tender nodularity of both breasts. What is the
management:
a) Hormonal treatment with oral contraceptive pills
b) Hormonal treatment with danazol?? xx
c) Lumpectomy
d) Observation for 6 months
787786- A 48 year old man complaining of right lower quadrant pain, bleeding per
rectum, nausea & vomiting. What is the best pre-operative investigation?
a) Air contrast enema
b) Fecal occult blood
c) CBC
d)
788- Which of the following indicate large uncomplicated pneumothorax:
a) Symmetrical chest movement.
b) Increase breath sound
c) Dull percution note.
d) Tracheal deviation (correct)
e) Cracking sound with each heart beat
___________________________________________________________________
789-. A pregnant woman, multigravida, 38 weeks gestational age presented
with glucosuria. Gestational diabetes was confirmed by glucose tolerance
test. The next step is:
a) Repeat Glucose tolerance test
b) Cesarean section
c) Diet adjustment
d) Start sliding scale insulin

135

790-A young primigravida, 35 weeks gestation, had BP of 140/90, headache,


proteinurea & lower limb edema. What is the best management: (dx
preeclampsia)
a) Oral labetolol
b) Diuretics
c) Low sodium diet
d) Immediate C.section
e) Admission & observation of feto-maternal condition (correct)
_____________________________________________________
791- A 30 yo lady in the third trimester of her pregnancy developed a sudden
massive swelling of the left lower extremity extending from the inguinal
ligament to the ankle. The most appropriate sequence of work up & treatment:
a. Venogram, bed rest, heparin
b. Impedance plethysmography, bed rest, heparin
c. Impedance plethysmography, bed rest, vena caval filter
d. Impedance plethysmography, bed rest, heparin,
warfarin
e. Clinical evaluation, bed rest, warfarin
792-Elderly female married since 30 years had fever, chills, dysurea, and
diarrhea. No back pain. The diagnosis is:
a) Acute bacterial cystitis
b) Acute pyelonephritis >>In acute pyelonephritis, the classic triad of fever,
costovertebral angle pain, and nausea and/or vomiting may be present,
although they may not necessarily occur together temporally.

c) Bacterial gastroenteritis
d) Viral gastroenteritis
___________________________________________________________________
793- A young female patient who is an office worker presented with itching
in the vagina associated with the greenish-yellowish vaginal discharge.
Examination revealed red spots on the cervix. The diagnosis is:
a) Trichomoniasis (correct)Women with trichomoniasis frequently report a
frothy yellowish-green vaginal discharge
b) Candidiasis>>whitish or whitish-gray cottage cheese-like discharge

c) Gonorrhea
d) Gardnerella vaginalis
794- Uterovaginal prolapse:
a) Increase heaviness in erect position (correct)
b) More in blacks
c) A common cause of infertility
d) ..
795- A patient presented with fatigue, loss a petite & bloody urine. She gave
History of sore throat 3 weeks back. The most likely diagnosis is:
a) hemorragic pyelonephritis
136

b) Post streptococcal GN(correct)


c) Heamorragic cystitis
d) membranous GN
e) IgA nephropathy
___________________________________________________________________
?? 796- A child of an HIV positive mother. He is not immunized. Which of the
following vaccines should not be given to him?
a) Live oral polio
b) DTP
c) MMR
d) Hepatitis B
___________________________________________________________________
797- A child swallowed his relative's medication. What is the best way of
gastric decontamination? ????????????
a) Gastric lavage
b) Total bowel irrigation (whole bowel wash)
c) Syrup ipecac
d) Activated charcoal>>(correct) For most medication ingestions, singledose activated charcoal is the modality of choice for gastrointestinal
decontamination. This treatment can generally be used up to 1 hour after
ingestion of a potentially toxic amount of medication


___________________________________________________________________
798-The commonest chromosomal disease is:
a) Down syndrome (Trisomy 21) (correct)
b) Klinfelter syndrome
c) Turner's syndrome
?? 799-A young girl pt had URTI 1 week ago & received septra (trimethoprime
+ sulphamethoxazole). She came with crampy abdominal pain & proximal
muscle weakness. The diagnosis is:(the same question in alqasem but other
choises)474
a) Polymyositis
b) Gullian parre syndrome
c) Intermittent porphyria
d) Periodic hypokalemic paralysis
e) Neuritis
800-All the following are present in otitis media except:
a) Signs & symptoms of inflammation
b) Signs & symptoms of effusion
c) High grade fever
137

d) Pain
801-Best ttt for chronic pain management:
Ibuprofen >> NSAIDare most beneficial in cases of acute pain, or flare-ups in patients with chronic
pain. NSAIDs are excellent at treating inflammatory conditions including tendonitis, bursitis, and
arthritis.

acetaminophen (correct)>>in cases of chronic pain, no inflammation is at the site of the pain,
and thus Tylenol may be an appropriate treatment choice.

In general, NSAID use is limited for patients with chronic pain because of concerns about the development to
stomach problems

naproxen
802- CPR:
2 breaths raise the chest,
you can DC shock 3 successive times,
cardiac massage in aortic stenosis
40%recovery
803- Lump in the bake with punktem not increase in size for year when doctor press
it the punctum came discharge yellow fuel smell
Remove to avoid rapture in derm??
Antibiotic first then remove
804- T score of 70 years old osteoprotic male is :
T-score of greater than minus-1 is considered normal.
A T-score of minus-1 to minus-2.5 is considered osteopenia, and a risk for developing osteoporosis.
A T-score of less than minus-2.5 is diagnostic of osteoporosis

-3 (correct),
-2 ,
2,
3
__________________________________________________________________________

805-retinal detachment all of the following are true EXCEPT:


a) can lead to sudden loss of vision
b) more in far sighted than near sighted (correct)>> When looking at patients who already
have retinal detachments, you begin to see some interesting trends. Many of these patients are myopic
(near-sighted). Myopic eyes are physically larger and longer than normal eyes and have thinner retina at the
periphery this thin retina is more likely to break forming small holes and tears that may progress to a
detachment

c) follow cataract surgery


d) if you suspect it sent for ophthalmologist
806- A 40 year old man who become sweaty with palpitation before
giving a speech in public otherwise he does very good at his job, he is
having:
a) generalizes anxiety disorder
b) performance anxiety (correct)
138

c) agoraphobia
d) depression
807- A man is brought to the ER after having seizure for more than 30
min the most initial drug you will start with:
a) IV lorazepam (correct)>> initial treatment due to its relatively long (28 hour) duration of
action when injected, and its rapid onset of action, which is thought to be due to its high affinity for GABA
receptors and to its low lipidsolubility which causes it to remain in the vascular compartment

b) IV phenobarbital
c) IV phynetoin
d) IV haloperidol
___________________________________________________________________
_______
808- A women who lost her husband 2 weeks ago she is unable to sleep
at all you will give her:
a) floxitine
b) diazepam
c) halperidol
d) amytriptaline
___________________________________________________________________
_______
a 65 yrs old lady came to your clinic with Hx of 5 days insomnia and crying
( since her husband died ) the best Tx. For her is :
a- lorazipam
b- floxitein
c- chlorpromazine
d- haloperidol
809-17 year pt with dyspnea Po2 , PCO2 ,Xray normal PH increase so dd is
- acute attack of asthma (correct)
-PE
- pneumonia
-pnemothrax
810- Picture of rash which appear pink on wood light:
-erythrasma>>is a macular brown area with few symptoms, most often found in the armpits or groin. It is
caused by overgrowth of diphtheroids of the normal skin flora. These areas fluoresce coral PINK under longwave ultraviolet radiation (Wood's light).

Fungal infection
811-RTA pt with femur fx , he has laceration of the femoral artery .. What to do :
1- end to end anastomosis
2- prosthetic graft
3-arterial graft
4- venous graft
139

___________________________________________________________________
812- Psycho pt swallow open safety pins,, x-ray show pins in the small intestine,,
what your next step:
-do emergent surgery(correct)>>Very sharp or pointed objects may perforate the GI
tract (sewing needles are notorious). Therefore, such objects should be endoscopically
removed from the stomach. If such an object has passed into the intestines, early
consultation with a surgeon is recommended. Objects that are too long (eg, >6 cm) or
too wide (eg, >2 cm) to pass through the pyloric sphincter should be removed from
the stomach.

-reassurance and discharge


-admitt and observe the pt
813-Theophylline interfere with absorption of:
-names of antibiotics
rifampin,ciprofloxacin,clarithromycin,erythromycin
__________________________________________________________
?? 814- A child presented with respiratory distress & accessory muscle
contraction. What is your treatment?
a) Oxygen
b) Bronchodilator
c) IV corticosteroid
d) IV Theophylline
?? 815- A child presented with dysphagia, sore throat, postnasal drip, drooling
of saliva, rhonchi & fever of 38.50c. The treatment is:
a) Hydrocortisone injection immediately
b) Call otorhinolaryngology for intubation
c) Admit to ICU
d) Give antibiotics & send him home
816- A child was treated for otitis media with 3 different antibiotics for 6 weeks
but without improvement. Which antibiotic is the best treatment:
a) Amoxicillin??
b) Penicillin
c) Cepahlosporin (ceprofloxacin)
d) Amoxicillin + Clavulonic acid>>
e) Erythromycin + sulfamethoxazol
818- A 70 year old female brought to your clinic by her daughter. The
daughter said her mothers memory deteriorated in the last 2 years. She can
cook for her self but sometimes leave the oven on. She can dress herself but
with difficulties. The daughter mentioned that her mothers personality
changed into a more aggressive person(pt has Alzehimer's disease). According
to this history what is your appropriate management?
a) Prescribe diazepam for the daughter and haloperidol for the mother
b)
Refer the mother into chronic illness institute
c)
Refer the mother to geriatric clinic (correct)
d) Immediate hospitalization
__________________________________________________________________
140

819- A man was intent as if he is listening to somebody, suddenly started


nodding & muttering. He is having: ????????
a) Hallucination (correct)>>is a perception in the absence of a stimulus
b) Delusion>>is a false belief held with absolute conviction despite
superior evidence
c) Illusion>>distortion of the senses, revealing how the brain normally
organizes and interprets sensory stimulation
d) Ideas of reference>>Ideas of reference and delusions of reference
involve people having a belief or perception that irrelevant, unrelated or
innocuous phenomena in the world refer to them directly or have special
personal significance

e) Depersonalization
820- A 45 yo lady was complaining of dizziness, sensory neural hearing loss
on her left ear (VIIIth nerve palsy), tingling sensation & numbness on her face,
loss of corneal reflex. MRI showed a dilated internal ear canal. The diagnosis
is:
a) Acoustic neuroma>>any unilateral sensorineural hearing loss is caused
by an acoustic neuroma until proven otherwise

http://emedicine.medscape.com/article/882876-overview#a0112
b) Glue ear
c) Drug toxicity
d) Herpes zoster
e) Cholesteatoma
821- A pt had hairline metatarsal fracture. The x-ray was normal. What is the
2nd line?
a) CT scan
b) MRI(correct)>>MRI is sensitive for the diagnosis of fractures, it is not
required, because plain radiographic findings are fairly sensitive and specific.
MRI is useful in the assessment of fractures and dislocations, soft tissue, the
plantar plate, structures of the capsule, the extent of marrow hyperemia, the
exact number of bones involved, and small chip fractures
MRI is more sensitive than radiography and even scintigraphy in the early
diagnosis of stress fractures, because it shows bone marrow edema
exquisitely. MRI may be used to differentiate stress fractures from early
degenerative changes and early stress fractures from synovitis

c) US
d) ..
822- A Case scenario about a male patient present with prostatitis
(prostatitis was not mentioned in the question ), culture showed gram negative
rodes. The drug of choice is:
a) Ciprofloxacin (florqinlon)>>(correct)
b) Ceftriaxone
c) Erythromycin
d) Trimethoprime
e) Gentamicin
141

823- A pt complaining of left flank pain radiating to the groin, dysurea, no


fever. The diagnosis is:
a) Pyelonephritis >>It can cause high fever, pain on passing urine, and
abdominal pain that radiates along the flank towards the back. There is often
associated vomiting

b) Cystitis
c) Renal calculi >> (correct) The hallmark of stones that obstruct the
ureter or renal pelvis is excruciating intermittent pain that radiates from the
flank to the groin or to the genital area and inner thigh

824-A 10 yo boy presented with a 5 days history of skin lesion which was
scaly & yellowish. The diagnosis is:
a) Tenia corporum>> (ringworm,[1]tinea circinata,[2] and tinea
glabrosa[1]) is a superficial fungal infection (dermatophytosis) of the arms
and legs, especially on glabrous skin, however it may occur on any part of
the body.enlarging raised red rings with a central area of healing
(ringworm). The same appearances of ringworm may also occur on the
scalp (tinea capitis), beard area (tinea barbae) or the groin (tinea cruris,
known as jock itch or dhobi itch).
Otherclassicfeaturesoftineacorporisinclude:

The edge of the rash appears elevated and is scaly to touch.

Sometimes the skin surrounding the rash may be dry and flaky.

Almost invariably, there will be hair loss in areas of the infection

___________________________________________________________________
__________
825- A mother brought her baby & was complaining of diaper rash. She used
cornstarch, talc powder, zinc ointment & 3 different types of corticosteroids
prescribed by different physicians but with no benefit. The rash was well
demarcated & scaly with satellite lesions. The most likely diagnosis:
a) Candidal rash (correct) >>fungal infection
b) Seborrhic dermatitis>>present with scaly, flaky, itching
red skin ,affect sebaceous gland-rich areas of skin
ttt:1/antifungal 2/anti-inflammatory 3/ topical steroid ..
c) Allergic contact dermatitis>>ttt : corticosteroid
826-A child presented with honey comb crust lesion. Culture showed staph
aureus. The diagnosis is:(539)
a) Impetigo (correct)
b) ..
c) ..
d) ..
827-. A pt presented with a 6 week history of itching & redness all over the
body with wheals. Which type of urticaria this pt has:
142

a) Chronic urticaria. (correct) >>Chronic urticaria (also known as

"Ordinary urticaria"[1]) is defined as the presence of evanescentwheals which


persist for 6 weeks or more

b) Solar urticaria
c) Allergic urtecaria
d) ..
828- A middle age man presented with sever headache after lefting heavy
object. His BP was high. He was fully conscious. Examination was otherwise
normal. The most likely diagnosis is:
a) Subarachnoid hemorrhage
b) Central HTN
c) Tension headache
d) Migraine
e) Intracerebral hemorrhage
829- You were working in a clinic with a consultant who prescribed a drug that
was contraindicated to the pt (the pt was allergic to that drug) but you didn't
interfere & assumed that he knows better than you do. Which of the following
you have violated:
a) Professional competence
b) Quality of caring of patient. (correct)
c) Honesty.
d) Pt relationship
e) Maintaining trust
___________________________________________________________________
830- Physician's carelessness is known as:
a) Malpractice (correct)
b) Criminal neglect
c) Malfeasance>>
d) Nonfeasance >>
831-.The most important factor in attempt of successful cessation of smoking
is?
a) The smokers desire to stop smoking . (correct)
b) The pharmacological agents used in the smoking cessation program.
c) Frequent office visits.
d) Physicians advice to stop smoking
e) Evidence of hazards of smoking
832- For health education programs to be successful all are true except :
a- human behavior must be well understood
b- Information should be from cultural background
c- Doctors are only the health educators(correct)>>health educator is a

professionally prepared individual who serves in a variety of roles and is specifically


trained to use appropriate educational strategies and methods to facilitate the
143

development of policies, procedures, interventions, and systems conducive to the


health of individuals, groups, and communities

d- Methods include pictures and videos (mass media)


e- Involve society members at early stage
833- a 27 yrs. old female C/O abdominal pain initially periumbilical
then moved to Rt. Lower quadrant she was C/O
anorexia,nausea and vomiting as well ..
O/E : temp.38c , cough , tenderness in Rt lower quadrant but no rebound
tenderness.
Investigations : slight elevation of WBC's otherwise insignificant ..
The best way of management is:
a- go to home and come after 24 hours
b- admission and observation(correct)
c- further lab investigations
d- start wide spectrum antibiotic
e- paracetamol
what is the most likely diagnosis for the above patient ?
a- mesenteric lymph adenitits
b- acute appendicitis (correct)
c- peptic ulcer
834- a 24 yrs old pt. came for check up after a promiscuous
relation 1 month ago .. he was clinically unremarkable, VDRL :
1/128 he was allergic 2 penicillin other line of management is
(dx syphilis ):
a- ampicillin
b- amoxicillin
c- trimethoprim
d- doxycyclin (correct) >>Doxycycline and tetracycline have been used

Tetracycline, erythromycin, and ceftriaxone [27] have shown antitreponemal activity in


clinical trials; however, they currently are recommended only as alternative treatment
regimens in patients allergic to penicillin

__________________________________________________________
835-a 24 years old female pt. C/O : gray greenish discharge ,
itching .. microscopic examination of discharge showed :
flagellated organism most likely diagnosis is :
a- trichomoniasis ( trichomonas vaganalis )(correct)
144

836- a 43 yrs. old female pt. presented to ER with H/O : paralysis


of both lower limbs and parasthesia in both upper limbs since 2
hours ago .. she was seen lying on stretcher & unable to move
her lower limbs (neurologist was called but he couldn't relate her
clinical findings 2 any medical disease !!! ) when history was
taken , she was beaten by her husband the most likely
diagnosis is :
a- complicated anxiety disorder
b- somatization disorder
c- conversion disorder (correct)>>is a neurosis marked by the appearance of

physical symptoms such as partial loss of muscle function without physical cause but
in the presence of psychological conflict

d- psychogenic paralysis
e- hypochondriasis
the best treatment for the previous case is :
a- benzodiazepines
b- phenothiazine
c- monoamine oxidase inhibitor
d- selective serotonin reuptake inhibitor
e- supportive psychotherapy (correct)
837- a 58 yrs. old male pt. came with HX of fever, cough with
purulent foul smelling sputum and CXR showed : fluid filled cavity
the most likely diagnosis is :
a.abscess>>Presence of air-fluid levels
b- TB
c- bronchieactesis >>
_______________________________________________________________________________________
______________________________

838- a patient ( known case of DM ) presented to u with diabetic


foot ( infection) the antibiotic combination is :
a- ciprofloxacin & metronedazole
__________________________________________________________
839- a young pregnant lady (Primigravida) , 32 weeks of
gestation came to you C/O : lower limbs swelling for two weeks
duration .. she went to another hospital and she was prescribed (
thiazide & loop diuretic ) .. O/E : BP : 120/70 , mild edema , urine
dipstick : -ve and otherwise normal. The best action is : normal
145

a. continue thiazide & stop loop diuretic


b- cont. loop diuretic & stop thiazide
c- stop both
d- continue both and add potassium sparing diuretic
e- cont. both & add potassium supplement
840- a 17 yrs. old football player gave HX of Lt. knee giving off ..
the most likely diagnosis is :
a. Lat. Menisceal injury

b- medial menisceal injury


c- lateral collateral ligament
d- medial collateral ligament
e ant. Curciate ligament (correct)

841- a 10 yrs. old boy presented to clinic with 3 weeks HX of


limping that worsen in the morning .. this suggests which of the
following :
a- septic arthritis>>patient with rapid onset of joint pain
b-leg calve parthes disease>>LeggCalvPerthes syndrome is characterized by

idiopathic avascular osteonecrosis of the capital femoral epiphysis of the femoral head
leading to an interruption of the blood supply of the head of the femur close to the hip
joint
Onset of pain may be up to 4 hours after inactivity. Knee pain is felt in the back of the
knee rather than in the front, not unlike a localized charley horse. This lasts for an
hour or so and returns nightly on inactivity

c- RA??>>

Morning stiffness and a limp that is worse in the morning suggest juvenile rheumatoid arthritis

http://www.medscape.com/viewarticle/490135_4
d- a tumor
e- slipped capital femoral epiphysis >>

is a medical term referring to a fracture through the physis (the growth plate), which
results in slippage of the overlying epiphysis. Symptoms are waddling gait, loss of
motion in the hip joint, externally rotated foot, pain in the knee / groin / hip and
shortening of the hip. In up to 20% of cases slippage is bilateral. the knee starts to get
sore about 2-4 months before the actual hip goes. the pain in the knee can come and
go.

842-a 38 yrs old female came to you at your office and her pap smear report
was unsatisfactory for evaluation .. the best action is :
a- consider it normal & D/C the pt.
b- Repeat it immediately
146

c- Repeat it as soon as possible


d- Repeat it after 6 months if considered low risk
e- Repeat it after 1 year if no risk
?? 843-- a 62 yrs. old female pt. a known case of osteoporosis & on 1 alpha +
Ca supplement .. her lab works shows normal level of PO4, Ca & ALP her Xray shows osteopenia with SD = -3.5 . The best action is to :
a- continue on same medications
b- start estrogen
c- start estrogen & progesterone
d-add alevdonate ( bisthmus phosphate)
844- a 17 yrs. old school boy was playing foot ball and he was kicked in his Rt.
eye .. few hours later he started to complain of : double vision & echymoses
around the eye .. the most likely Dx. Is :
a- cellulites
b- orbital bone fracture (correct)
c- global eye ball rupture
e- subconguctival hemorrhage
845- a 35 yrs old female pt. C/O : acute inflammation and pain in her Lt. eye
since 2 days .. she gave Hx of visual blurring and use of contact lens as well
O/E : fluorescence stain shows dentritic ulcer at the center of the cornea ..
the most likely diagnosis is :
a- corneal abrasion
b- herpetic central ulcer >>Herpes virus cause a dendritic ulcer
c- central lens stress ulcer
d- acute episcleritis
e- acute angle closure glaucoma
846-a 25 yrs old Saudi man presented with Hx of mild icterus , otherwise ok ..
hepatitis screen : HBsAg +ve , HBeAg +ve , anti HBc Ag +ve (this should be
core anti body, because core antigen doesnt leave hepatocyte to the blood
"prof. Yasawi" ) , the diagnosis :
a. acute hepatitis B?(correct)
b- convalescent stage of hep. B
c- recovery with seroconversion Hep . B
147

d- Hep B carrier
e- chronic active Hep. B
Serological test findings at different stages of HBV infection and in convalescence
anti-HBc
Stage of infection

HBsAg

anti-HBs

IgG

IgM

HBeAg

anti-HBe

+ or -

+++

+ or -

+++

+ or -

++

++

+ or -

+ or -

+ or -

++

late incubation period

acute hepatitis B or persistent carrier


state

HBsAg-negative acute hepatitis B


infection

recovery with loss of detectable antiHBs

healthy HBsAg carrier

chronic hepatitis B, persistent carrier


state

HBV infection in recent past,


convalescence
HBV infection in distant past, recovery

recent HBV vaccination, repeated


exposure to antigen without infection,
or recovery from infection with loss of
detectable anti-HBc

a.
847-8 wk Primigravida came to you with nausea & vomiting choose the
statement that guide you to hyperemesis gravidarm :
a- ketonia (correct)
b- ECG evidence of hypokalemia
c- Metabolic acidosis
148

d- Elevated liver enzyme


e- Jaundice
?? 848-60 year old male was refer to you after stabilization investigation show
Hgb 8,5 g/l , hect. 64% , RBC 7.8 , WBC 15.3
& Plt. 570 Diagnosis :
a- iron def. Anemia
b- Hgb pathy
c- CLL
d- 2ry polycythemia
e- Polycythemia rubra Vera
?? 849-Pregnant women G4P3+1 on GA 10 wk came to you with IUCD
inserted & the string is out from O.S what is the most important measure :
a- leave the IUCD & give A.B
b- leave the IUCD & send to Ob/ Gynaecologist to remove
c- leave the IUCD
d- do laparoscopy to see if there is ectopic preg.
e- Reassurance the pt
850-30-Placenta previa excludes :
a- Pain less vaginal bleeding
b- Tone increased of uterus (correct)
c- Lower segmental abnormality
d- Early 3rd trimester
851-Pregnancy test +ve after :
a- one day post coital
b- 10 day after loss menstrual cycle (correct)>>qualitative hCG test .Doctors often order
these tests to confirm pregnancy as early as 10 days after a missed period

c- One wk after loss menstrual cycle


______________________________________________
852-- 45 year old female complaining of itching in genitalia for certain period,
a febrile, -ve PMH, living happily with here husband since 20 year ago on
examination no abdominal tenderness , erythema on lower vagina , mild Gray
discharge no hx of UTI . pyleonephritis
Most probable diagnosis:

149

a- Vaginitis (correct)
b- Cystitis
c- CA of vagina
d- Urithritis ( non gonococal )

Common Types of Vaginitis


Disorder

Typical Symptoms and

Criteria for Diagnosis

Microscopic Findings

Differential Diagnosis

Signs

Bacterial

Gray, thin, fishy-

Three of the

Clue cells, decreased

Trichomonal

vaginosis

smelling discharge,

following: Gray

lactobacilli, increased

vaginitis

often with pruritus and

discharge, pH

coccobacilli

irritation; no

> 4.5, fishy odor,

dyspareunia

and clue cells

Candidal

Thick, white discharge; Typical discharge,

Budding yeast,

Contact irritant or

vaginitis

vaginal and sometimes

pH < 4.5, and

pseudohyphae, or

allergic vulvitis

vulvar pruritus with or

microscopic

mycelia; best

Chemical irritation

without burning,

findings*

examined with 10%

Vulvodynia

irritation, or

K hydroxide diluent

dyspareunia
Trichomonal

Profuse, malodorous,

Identification of

Motile, flagellated

Bacterial vaginosis

vaginitis

yellow-green

causative

protozoa, increased

Inflammatory

discharge; dysuria;

organism by

PMNs

vaginitis

dyspareunia; erythema

microscopy*
(occasionally by
culture)

Inflammatory
vaginitis

Purulent discharge,

pH > 6, negative

Increased PMNs,

Erosive lichen
planus

vaginal dryness and

whiff test, and

parabasal cells, and

thinning, dyspareunia,

characteristic

cocci; decreased

dysuria; usually in

microscopy

bacilli

postmenopausal

findings

women

*Culture is needed if microscopic findings are negative or symptoms persist.

853-20 year lady come to ER with Hx of Rt sever lower abdominal pain with
Hx of amenorrhea for about 6 wk the most serious diagnosis of your deff.
Diagnosis could reach by:

150

a- CBC
b- ESR
c- U/S of the pelvis (ectopic pregnancy) (correct)
d- Plain X-ray
e- Vaginal swab for C/S
__________________________________________________________
854-Pt had arthritis in two large joint & pansystolic murmur ( carditis )
Hx of URTI the most important next step: (dx rheumatic fever)
a- ESR
b- ASO titre (correct but I'm not sure ) >>ASO is a test used to detect
streptococcal antibodies directed against streptococcal lysin O. An
elevated titer is proof of a previous streptococcal infection.
c- Blood culture?>>Blood cultures are obtained to help rule out infective
endocarditis, bacteremia, and disseminated gonococcal infection.
N.B :
Throat culture remains the criterion standard for confirmation of group A
streptococcal infection.
855-women complain of non fluctuated tender cyst for the vulva . came pain
in coitus & walking , diagnosed Bartholin cyst . what is the ttt:
a- incision & drainage
b- refer to the surgery to excision (after you reassure her)
c- reassurance the pt
d- give AB
______________________________________________________
856- 42years old male presented with history of sudden appearance of rash
maculopapular rash including the sole,& the palm, the most likely diagnosis
is :
a- syphilis
b- erethyma nodosum
c- erythema marginatum
d- pitryasis rocae
e- drug induced
857- years old lady on tricyclic antidepressent feels dizzy on standing,
resolves after 10-15 minutes on sitting, decrease on standing, most likely she
is having :
151

a- orthostatic hypotension>>Orthostatic hypotension, also known as postural


hypotension, is a form of hypotension in which a person's blood pressure suddenly
falls when the person stands up or stretches. The decrease is typically greater than
20/10 mm Hg,[2] and may be most pronounced after resting. The incidence increases
with age.

_______________________________________________________
858- what is the most appropriate treatment for the above patient :
a- antiemetic
b- antihistamine
c- change the antidepressant to SSRI
d- thiazide diuretics
e- audiometry
_________________________________________________________85
9- 23 years old lady with one month history of nasal discharge & nasal
obstruction, she complained of pain on the face, throbbing in nature , referred
to the supraorbital area, worsen by head movement, walking,& stopping. On -- --------- examination , tender antrum with failure of transillumination ( not
clear ), the most likely the diagnosis is:
a- frontal sinusitis (we can NOT trannsiiluminate it)
b- maxillary sinusitis??
c- dental abscess
d- chronic atrophic rhinitis
e- chronic sinusitis
860-the cardiac arrest in children is uncommon but if occur it will be due to primary
respiratory arrest (correct)
hypovolemic shock
neurogenic shock
861- Middle aged patient with an acyanotic congenital heart disease the X-ray
show ventrical enlargement and pulmonary hypertension:
a. VSD??
b. ASD
c.
Trancus arteriosus>>cyanotic CHD
d. Pulmonary stenosis>>cyanotic CHD

152

862- role of surgery in. Stage C2 colon cancer ?


Curative
Palliative
Diagnostic
Exploratory
Stage 0
Tis, N0, M0:Thecancerisintheearlieststage.Ithasnotgrownbeyondtheinnerlayer(mucosa)ofthecolonorrectum.This
stageisalsoknownascarcinoma in situ orintramucosal carcinoma.

Stage I
T1-T2, N0, M0:Thecancerhasgrownthroughthemuscularismucosaintothesubmucosa(T1)oritmayalsohavegrowninto
themuscularispropria(T2).Ithasnotspreadtonearbylymphnodesordistantsites.

Stage IIA
T3, N0, M0:Thecancerhasgrownintotheoutermostlayersofthecolonorrectumbuthasnotgonethroughthem(T3).Ithasnot
reachednearbyorgans.Ithasnotyetspreadtothenearbylymphnodesordistantsites.

Stage IIB
T4a, N0, M0:Thecancerhasgrownthroughthewallofthecolonorrectumbuthasnotgrownintoothernearbytissuesororgans
(T4a).Ithasnotyetspreadtothenearbylymphnodesordistantsites.

Stage IIC
T4b, N0, M0:Thecancerhasgrownthroughthewallofthecolonorrectumandisattachedtoorhasgrownintoothernearby
tissuesororgans(T4b).Ithasnotyetspreadtothenearbylymphnodesordistantsites.

Stage IIIA
Oneofthefollowingapplies.
T1-T2, N1, M0:Thecancerhasgrownthroughthemucosaintothesubmucosa(T1)anditmayalsohavegrownintothe
muscularispropria(T2).Ithasspreadto1to3nearbylymphnodes(N1a/N1b)orintoareasoffatnearthelymphnodesbutnot
thenodesthemselves(N1c).Ithasnotspreadtodistantsites.
T1, N2a, M0:Thecancerhasgrownthroughthemucosaintothesubmucosa(T1).Ithasspreadto4to6nearbylymphnodes
(N2a).Ithasnotspreadtodistantsites.

Stage IIIB
Oneofthefollowingapplies.
T3-T4a, N1, M0:Thecancerhasgrownintotheoutermostlayersofthecolonorrectum(T3)orthroughthevisceralperitoneum
(T4a)buthasnotreachednearbyorgans.Ithasspreadto1to3nearbylymphnodes(N1a/N1b)orintoareasoffatnearthelymph
nodesbutnotthenodesthemselves(N1c).Ithasnotspreadtodistantsites.
T2-T3, N2a, M0:Thecancerhasgrownintothemuscularispropria(T2)orintotheoutermostlayersofthecolonorrectum(T3).
Ithasspreadto4to6nearbylymphnodes(N2a).Ithasnotspreadtodistantsites.
T1-T2, N2b, M0:Thecancerhasgrownthroughthemucosaintothesubmucosa(T1)oritmayalsohavegrownintothe
muscularispropria(T2).Ithasspreadto7ormorenearbylymphnodes(N2b).Ithasnotspreadtodistantsites.

Stage IIIC
Oneofthefollowingapplies.
T4a, N2a, M0:Thecancerhasgrownthroughthewallofthecolonorrectum(includingthevisceralperitoneum)buthasnot
reachednearbyorgans(T4a).Ithasspreadto4to6nearbylymphnodes(N2a).Ithasnotspreadtodistantsites.
T3-T4a, N2b, M0:Thecancerhasgrownintotheoutermostlayersofthecolonorrectum(T3)orthroughthevisceralperitoneum
(T4a)buthasnotreachednearbyorgans.Ithasspreadto7ormorenearbylymphnodes(N2b).Ithasnotspreadtodistantsites.

153

T4b, N1-N2, M0:Thecancerhasgrownthroughthewallofthecolonorrectumandisattachedtoorhasgrownintoothernearby


tissuesororgans(T4b).Ithasspreadtoatleastonenearbylymphnodeorintoareasoffatnearthelymphnodes(N1orN2).Ithas
notspreadtodistantsites.

Stage IVA
Any T, Any N, M1a:Thecancermayormaynothavegrownthroughthewallofthecolonorrectum,anditmayormaynothave
spreadtonearbylymphnodes.Ithasspreadto1distantorgan(suchastheliverorlung)orsetoflymphnodes(M1a).

Stage IVB
Any T, Any N, M1b:Thecancermayormaynothavegrownthroughthewallofthecolonorrectum,anditmayormaynothave
spreadtonearbylymphnodes.Ithasspreadtomorethan1distantorgan(suchastheliverorlung)orsetoflymphnodes,orithas
spreadtodistantpartsoftheperitoneum(theliningoftheabdominalcavity)(M1b).

863- a patient with a large nodule in the nose which is painful and
talangectasia on the face you will give:
a) deoxycycline (correct)
b) clindamycin
c) retenoid
864-18 years old not sexually active came to your clinic complaining of missed 2
period with sever abdominal pain on examination abdomen can't examine because
sever tenderness what you will do ?
Pregnancy test
Ultrasound
progesterone 100 mg for 10 days
865- Which of the follwing drug used in mycardial infaction to prophylaxsis against
arrythmia ?
Metoprlol
Adenosin
Atropin
Ca Channel blocker
__________________________________________________________________________
866- In a day care center10 out of 50dovelop red eye. another 30 develop same
condition in the next 2 week , what is the
attack rate
a) 40%
b) 60%
c) 80%
d) 20%
The term is defined as the number of exposed persons infected with the disease
divided by the total number of exposed persons

So 10+30/50 =80
867- Elderly patient who was smoking 2pack /day for 35 years complaing of
shortness of breath X ray done show plural effusion plurocentesis show PH less
than 7
What is the diagnosis ?
154

Cardiopulmonary edema
Empyema
Brochogenic carcinoma
868-most specific and sensitive Ix for renal stone
KUP
IVP
U/S
CT (correct)>>All stones are detectable on CT scans except very rare stones
composed of certain drug residues in the urinesuch as from indinavir.

869-Side effect of atropine :


Dryness of the mouth
Adverse reactions to atropine include ventricular fibrillation,
supraventricular or ventricular tachycardia, dizziness, nausea,
blurred vision, loss of balance, dilated pupils, photophobia, dry
mouth and potentially extreme confusion, dissociative
hallucinations and excitation especially amongst the elderly.
These latter effects are because atropine is able to cross the
blood-brain barrier. Because of the hallucinogenic properties,
some have used the drug recreationally, though this is
potentially dangerous and often unpleasant
870- diastolic" blowing" murmr best to heard in the left sternal
border increasing with squating imp.
AS
AR (correct)
MS
MR
MVP

Valve incompetence allows


backflow from aorta to ventricle.
Descriptio Caused by rheumatic heart disease,
n
endocarditis, aortic diseases
(Marfans syndrome, medial
necrosis), syphilis, ankylosing
spondylitis, dissection, cardiac
trauma.
Type and Heard with diaphragm, patient
Detection sitting and leaning forward; AustinFlint murmur heard with bell;
ejection click heard in 2nd
155

intercostal space

Early diastolic, high pitch, blowing,


Findings often with diamond-shaped
on
midsystolic murmur, sounds often
Examinati not prominent; duration varies with
on
blood pressure; low-pitched
rumbling murmur at apex common
(Austin-Flint); early ejection click
sometimes present.
In left ventricular hypertrophy,
prominent prolonged apical impulse
down and to left Pulse pressure
wide; water-hammeror biferiens
pulse common in carotid, brachial,
and femoral arteries.
Heart
Sound
S1 soft; S2 split may have tambourCompone like quality; M1 and A2 often
nts
intensified; S3-S4 gallop is common.

871- the most lethal injury to the chest is


pneumothorax
rupture aorta (correct)>>Rupture of the thoracic aorta is the most lethal
injury following blunt chesttrauma

flail chest
cardiac contusion
872- lethal injury to the chest after motor accident:
puncture lung
spontaneous pneumothorax
rupture aorta (correct)
flail chest
all of the above
__________________________________________________________________________
_
873-in acute abdomen the type of respiration is:
rapid and shallow (true)
156

rapid and deep


slow and shallow
slow and deep
__________________________________________________________________________
_
874- pregnant female complaining of constipation what is the
managment :
tell her to to stop taking the iron supplements
give her fibers
give her bulking agents
all of the above
Eat high-fiber foods such as whole-grain cereals and breads, brown rice, beans, and fresh fruits and vegetables
every day. Adding a couple of tablespoons of unprocessed wheat bran (available at health food stores) to your
cereal in the morning and following it with a glass of water can help, though it may take a few days before you
notice a difference.

Drink plenty of water at least six to eight glasses a day. A glass of fruit juice every day, especially prune
juice, can also be helpful. Some people find that drinking a warm liquid right after waking up helps get things
moving.

Exercise regularly. Walking, swimming, riding a stationary bike, and yoga can all help ease constipation and
leave you feeling more fit and healthy.

Your bowels are most likely to be active after meals, so make time to use the bathroom after you eat. Listen
to your body. Never put off going to the bathroom when you feel the urge.

If your prenatal multivitamin contains a large dose of iron (and you're not anemic), ask your healthcare
provider about switching to a supplement with less iron.

If the measures above don't help, talk to your caregiver about taking an over-the-counter fiber supplement
or stool softener.

__________________________________________________________________________
875- patient has terminal ovarian carcinoma came to u complaining of
dull aching abdominal pain when u did xray u found a 10 cm metalic
clamp what will u do :
Call your lawyer for advice lol
call the surgeon for advice
no need to inform the patient since she is terminal and would not find out
about it
inform the patient and inform the surgeon and tell her it will dissolve most
likely in a ..... (certain abount of time dont remeber i think it was a month
) (correct)
876-which one of the following Rx has lowset risk of tardive dyskinesia:
Clozapine >>clozapine has been shown to have a lower risk of tardive dyskinesia
than older antipsychotics

chloropromazine
haloperidol
___________________________________________________________
877-family came to you complaining that their son sees humans as
(something... objects i think it was innate objects not sure ) and plays
alone and doesnt play with other children and says "you" when he wants
to say "I"
157

which one of the following should not be done for the management of this
Patient:
narcoleptic medication
high ..... care program in school
mood stabilizers
878-patient male suddenly had bahevoral and cognitive imparment and
now sees a monkey in the room most likely Dx :
schizophrenia
Dementia like in parkinsonisim
delrium
depression
__________________________________________________________________________
__________________________
879-which of the following is correct about use of systemic retiniods :
*teratogenic(correct)
__________________________________________________________________________
__________________________
880-bupropion is contraindicated in which of the following :
*Hx of eating disorder ( Contraindicated in bulemia sure ) (correct)
bupropion should not be prescribed to individuals with epilepsy or other conditions
that lower the seizure threshold, such as alcohol or benzodiazepine withdrawal,
anorexia nervosa, bulimia, or active brain tumors.

881-8TB outbreak ..and one pt. come to doing tubercalin test and it's
negative .. what to do??
a- BCG
a. isonized
b. rifampin
882- baby with congugated hyperbilirubinemia:
Biliary atresia(correct)
ABO comp
G6PD
883- ld with URTI what is the most helpfully sign that it is viral:
Colorless nose discharge
Clinically significant rhinorrhea is more characteristic of a viral infection rather than a
bacterial infection. In viral URI, secretions often evolve from clear to opaque white to
green to yellow within 2-3 days of symptom onset. Thus, color and opacity do not
reliably distinguish viral from bacterial illness.

__________________________________________________________________________
884-child with picture of SCA he should be maintained on :
Penicillin and folic acid (correct)
885-signs of androgen excess and ovarian mass , most likely tuner :
Sertoli-Leydig cell tumour (correct)
158

886-girl with hirsitism , deep voice , receding hair line :


Androgen excess (correct)
887- pregnant in the third trimester came with pain less vaginal
bleeding :
Placenta previa(correct)
888- picture of child with red rash on flexor surfaces :
Atopic dermatitis (correct )

Flexural involvement in childhood atopic dermatitis.

______________________________________
889- child with round palpable red rash on his right leg no pain or itching
for long time :
- granuloma annular
- tenia corpora
-erythema nodosum
-migratory
__________________________________________________________
890- the goal of early management of inflammatory acne:
- to prevent physical scar
-to prevent spread of infection
891- prevention of malaria
Eradication of vector and protect against bites (correct)
http://www.netdoctor.co.uk/diseases/facts/malaria.htm
892- case of TB , what knd of injections u will give the contacts :
BCG
893- a picture of Snelling chart the q was how far should the patient stand
:
3m
6m (correct)
9m
894- pt with epistaxis , what is the most apropriate initial management :
Tampon ,
159

direct pressure on the soft part of the nose , (correct)


do nothing .
895- Sickle cell anemia patient non painful loss of vision in left eye, on
examination afferentflame sing and macula get??, what is the cause?
- Retinal detachment
- Central retinal Artery occlusion
- Central retinal Vein occlusion
- ???
896- Patient with Obsessive compulsive disorder we need to:
A. Decrease serotonin in the blood
B. Increase Serotonin in the blood
C. Increase sensitivity
D. Decrease synthesis
897- Patient came to ER complain of pluratic chest pain and SOB. On
Examination the left lung is hyper resonant and no breath sound. What is
the diagnosis:
A. Pulmonary effusion
B. Pneumonia
C. MI
D. Cardiac Temponate
E. Pneumothorax (correct)
898- Treatment of Pruritic Folliculitis:
A. Topical Antibiotic
B. Oral Antibiotic
C. Oral steroid
Treatment
Pruritic folliculitis of pregnancy is typically treated like mild acne. Benzoyl peroxide has
been used with some success, but antibiotics are not needed. Oral antihistamines are
useful to treat the itching.

http://dermatology.about.com/cs/pregnancy/a/prurfoll.htm
__________________________________________________________________________
_ 899-51 year old male Hb7 MCV 112 AST 250 with Giant cells, whats the
diagnosis:
A. B12 deficiency anemia
B. Folic acid deficiency anemia
C. Alcoholic anemia
D. Thalasemia
E. Iron deficiency Anemia
900- Patient was diagnosed to have Otitis media today you examined the
patient he is fine tempanic membrane is no longer erythematus but there
is collection of fluid behind it, whats your next step:
A. Do Nothing
B. Decongestion
C. Antibiotics
160

901- Patient complaining of vetligo and he must hold something while


walking and sitting down, This is found in 75% of patients taking :
A. INH side effect
B. Penicillin side effect
C. Streptomycin side effect (correct)
D. Allergy to penicillin
E. One more drug side effect
__________________________________________________________________________
_
902-9year old boy cam to PHC with URTI and swap was taken and sent
home, after 5 days the result was Group A streptococcus and then you
called the family and they told you the boy is fine and no symptoms
whats you next step:
A. Give Ceftixim IM one dose
B. Penicillin for 7 days
C. Penicillin for 10 Days
D. Do Nothing
903-After start ttt of depression for Pt. who show improvement, the risk of suicide :
Increase
Decrease (correct)
The same after and before ttt
Will not suicide at all
904-the symptom/sign that comes 2ry rather than presented symptom in panic pt.
tachycardia
epigastric pain
chest pain
phobia
905-Mechanism of Action of drugs that inhibit Conversion of estriol to estrogen

.
(I forgot the exactquestion but it mentioned about ovulation and who inhibits
conversion of esterone toestrogen?) Options were:
aAromatase inhibitors

synthesis of estrogens starts in theca interna cells in the ovary, by the synthesis of
androstenedione from cholesterol. Androstenedione is a substance of moderate
androgenic activity. This compound crosses the basal membrane into the surrounding
granulosa cells, where it is converted to oestrone or oestradiol, either immediately or
through testosterone. The conversion of testosterone to oestradiol, and of
androstenedione to oestrone, is catalyzed by the enzyme aromatase.
Oestradiollevelsvarythroughthemenstrualcycle,withlevelshighestjustbeforeovulation.

906-year old female patient of Cushings syndrome, had hip fracture falling off
stool, what will you screen for while also treating her fracture:
Hyperparathyroidism
161

Osteomyelitis
Osteoporosis
Osteomalacia
907-Drug table given each with3-4drugs, question which group causes
hyperuricemia.
ABCD
(I choose the group which had anti-neoplastics.)

The principal drugs that contribute to hyperuricemia by decreased excretion are the
primary antiuricosurics. Other drugs and agents include diuretics, salicylates,
pyrazinamide, ethambutol, nicotinic acid, ciclosporin, 2-ethylamino-1,3,4-thiadiazole,
and cytotoxic agents

908- Child 3 weeks ago had chicken pox, came to ER c/o sob, x-ray shows
enlarged epiglottis, what is the cause?
- Hemophlus influ, type b
- Diphtheria pertusis
- Rubella
- Measles
909-2 years old child fall down in homein x-ray there is spiral # in radial
bone the best management :
-call pediatric.
-call orthopedic.
-splint.
-open for fixation.
910-female married since four month and she noticed her husband
washing his hand several times the most likely diagnosis:
obsessive compulsive disorder
911-pt. admitted with pneumonia and the vaccine for this pt. is:
-pneumonia with H.influnza
-influenza alone.
-pneumonia alone.
912-pt. have peptic mass the most common organism for infection:
-H pylori
-HPV
-HIV
913-pt have swelling in lower eye lid and the lid was erythemic and
edemoutis with hair inside make corneal ulcer the dx:
-entropic.
-extropic
914-evidence based medicine:
-as in text book.
-according to departmental policy.
-according to latest published articles.
162

-according to strong scientific evidence.


915-16 wk GA with ++glycosuria FBS:4.4, 1Hr PP 8.2 , 2Hr PP 7.2:
-renal glycosuria??
-GDM.
-K.M syndrome
916-what is the best source of iron in a 3 month old infant?
-breast milk.
-low fat cow milk.
-yellow vegetables.
-fruit.
-iron fortified.
917-#pt with bilateral infiltration in lower lobe (pneumonia )which organism is
suspected :
-ligonella
-klibsella
-strept pnemoniae
- staph
-pnemococcus pnemonia
918- 33 year old make C/O of pain in his lip and right check . Pain was stabbing like
triggered with touch. O/E, cranial nerves were intact , The best Rx Is:
a- Oxcrazepine
b- Propanol
c- Ergotamine
d- Lithium
919- Avoid with abnormal ALT >>(read about hyperlipidemia drug)
?!! HMG coA
920-isolated closed Fracture femur treatment is:
a. internal fixation with plate
b. Internal fixation with antegrade intramedullary locked nail ( Sabiston 17 th
ed page 563 )??
c. External fixation with intramedullary locked nail
d. traction & balance
e. cast around hip
The method of fixation of femoral shaft fractures has become fairly standardized. The treatment of
choice for closed fractures and types I through IIIA
open fractures is closed, locked IM nailing. In contrast to open reduction methods, this practice reduces
bleeding and soft tissue disruption at the
fracture site. These minimally invasive techniques reduce perioperative stress and decrease the incidence
of infection and nonunion.

921-27 yr old lady primi 35 wks pregnant, presented with mild Pre-eclampsia , BP
140/? Edema in her hands & feet, best treatment is:
163

f.
g.
h.
i.

Immediate delivery
Diuretics
Send home?
Hospitalize & materno-fetal monitoring ( this is the most likely answer

Johns hopkins manual of gynecology and obstetrics 3rd ed Pages 183-184


Mild pre-eclampsia
i. If the pregnancy is >37 weeks, then the patient should be managed
as follows:
1. If cervix is favorable (or patient is noncompliant), then
proceed to induction of labor.
2. If cervix is unfavorable (bishops score <6), then close
maternal and fetal observation is essential.
a. FetalUltrasound for growth/AFI, and, if reassuring, no
further sonography is required barring a progression in
symptoms or signs of poor fetal growth or distress;
NST/BPP 1 to 2 times per week.
b. MaternalWeekly evaluation and laboratory testing, as
described earlier. Weekly cervical examwhen
favorable proceed to induction of labor. Despite an
unfavorable cervix, induction of labor should be initiated
by 40 weeks' gestation.
ii. Similarly, if the gestational age is >34 weeks and is accompanied by
progressive labor, preterm premature rupture of membranes,
abnormal fetal testing, or growth restriction, the patient should also
be delivered.
iii.
Outpatient management. Some compliant patients with mild
pre-eclampsia (BP <150/100, 24-hr urine protein <1 g, no
symptoms, normal liver transaminases and platelet count) can be
followed from home.
1. Outpatient management with decreased physical activity.
Strict bed rest is not recommended (3).
P.183

2. No antihypertensivesnot shown to improve perinatal


outcome (14)
3. Monitoring for symptoms of pre-eclampsia; daily kick counts

4. At least, weekly blood pressure checks and physical

examinations that evaluate for signs of worsening preeclampsia


5. Nonstress test/biophysical profile 1 to 2 times per week;
ultrasound for growth/AFI every 3 to 4 weeks
6. Laboratory testing: baseline 24-hour urine protein collection
and weekly laboratory testing for platelet count, liver
enzymes, and serum creatinine

164

iv.

Inpatient management. Management of these patients is


essentially the same, but monitoring is more intensive by hospital
personnel.

922-Not use in the prevention of preeclampsia with + protein urea & LL edema :
j. Admission &bed rest
k. Diuretics ( if you are understanding the underlying physiology you will
exclude this answer )
l. Non-stress test
m. Regular sonogram of baby
92340 yr old male with 4 days history of sudden eruption over the entire body
including palms & feet :
n. erythema nodosum
o. erythema multiforme
p. pit. rosea
There is no enough information to diagnose the case as whether it is itchy or not
but generally speaking it is either Pityriasis rosea ( more likely) or Sec. Syphilis
but it is rare now and in 75% it doesnt involve face
_____________________________________________________________
925-urticaria, all true EXCEPT:
q. can be part of anaphylactic reaction
r. is not always due to immune reaction
s. always due to deposition of immune complex in the skin ( due to increase
permeability of capillaries )
t. due to ingestion of drug
u. due to ingestion of strawberry

926-10 yr old boy woke up at night with lower abdominal pain, important area to

check:
v. kidney
w. lumbar
x. rectum
y. testis (the Q is not clear for me but it seems to be related to testicular
torsion or something in inguinoscrotal area

927-All are true about hoarsness in adult , EXCEPT :


z. due to incomplete opposition of the vocal cord
aa.
if > 3 weeks : need laryngoscopy
bb.
if due to overuse, advise to whisper a few weeks
cc.
commonly seen in bronchus Ca
dd.
feature of myxedema
I can't judge
165

________________________________________________________________
928- years old boy for evaluation of short stature. His height is of 6 year old & bone
scan of 5.5 years ,, Dx is:
ee.
steroid therapy
ff.
genetic
gg.
constitutional
hh.
hypochondroplasia (not sure)
ii. hypothyroidism ( not mentioned in our exam)
_______________________________________________________________
929- Diagnosis of Alzheimer confirmed by:
CT brain
EEG
Neurological examination
None of the above(investigation is aimed at excluding other treatable causes
of dementia, as histological confirmation of Dx usually occurs only after
death Davidson's 19th 1173

930-12 yr old girl with malaise, fatigue, sore throat & fever. On examination:
petechial rash on palate, large tonsils with follicles, cervical lymphadenopathy &
hepatosplenomegaly. All are complications EXCEPT:
Aplastic anemia
Encephalitis
Transverse myelitis
Splenic rupture
Chronic active hepatitis

931-1 month old with massive hepatosplenomegaly, bluish skin nodules, & lateral
neck swelling, the next step is:
CBC
lumber puncture
Do EBV serology
BM scan ( It seems to be congenital Leukemia)
Liver biopsy

932-8 month old baby came with dehydration, fever 40 C, poor feeding &

convulsions. depressed ant. Fontanelle, vomiting, & crying with red ears. No neck
stiffness. Her 3 yr old brother is asymptomatic. What is the most important
investigation to do:
Blood culture
CBC & differential
166

CSF examination ( Our Prof. told us that british says if meningitis crosses
your mind do LP ) provided that C/P is not specific in this age
Chest xray
Urine analysis

933-17 year old boy presented to the ER complaining of sudden onset of abdominal
pain & leg cramps, he had history of vomiting2 days ago, he was dehydrated .
Na = 150 , K = 5.4 ,, glucose = 23mmol
The best initial investigation is
CBC
Blood culture
ABG ( tha Dx is DKA)
Urinanalysis (dipstick)
U/S Iremember that this option was Serum amylase
_____________________________________________________________
934-young age male presented after RTA with injured membranous urethra , best
initial ttt is :
Passage of transurethral catheter
Suprapubic catheter
Perineal repair
Retropubic repair
Transabdominal repair

935-young male presented to ER with a stab wound in his abdomen , u should:


should Explore the abdomen
observe patient & not explore if vitals remain stable
Exploration depend on U/s
Exploration depend on DPL
Exploration peritoneum penetrated
______________________________________________________________
__
936-Contraindication of gastric lavage if the ingested material is:
Aspirin
diazepam
Dry clean (clorex) Corrosive
Castor beans
Vit D

937-65 year male presented with 10 days Hx of hemiplagia , CT shows :

infarction , he has HTN. He is on lisonipril & thiazide , 2 yr back he had gastric


ulcer . ttt that U should add :
continue same meds
167

Aspirin 325
aspirin 81
warfarin
dipyridamole ( Antiplatlet agent)

938-All of the fallowing are criteria of subarachnoid hemorrhage EXCEPT:

Paraplegia ( Davidson's 19th 1162)


confusion
nuchal Rigidity
Due to berry aneurysm rupture
Acute severe headache
______________________________________________________________
__
939-After infarction , the patient become disinhibeted , angree & restless . The
area responsible which is affected:
premotor area
temporal area
pre- frontal area ( Davidson's 19th 1148)

940-All are true about the best position in hearing the murmurs, EXCEPT:
supine : venous hum
sitting : AR
sitting : pericardial rub ( opinion : according to my knowledge no special
position for it to be accentuated)
supine : innocent outflow obstruction
Lt lateral in : MS

941-All are true about the best position in hearing the murmurs, EXCEPT:

supine : venous hum


sitting : AR
sitting : pericardial rub ( opinion : according to my knowledge no special
position for it to be accentuated)
supine : innocent outflow obstruction
Lt lateral in : MS
_____________________________________________________________
942-3 year old child needs oral surgery & comes to your clinic for checkup. On
examination 2/6 continuous murmur , in upper Rt sternal borders that disappear
with sitting , next step:
Give AB prophylaxis
Ask cardiology consult
Clear for surgery
168

Do ECG

943-Glue ear
Managed by grommet tube ( Nelsson 19th 1955)
Lead to sensorineural hearing loss
Pus in middle ear
Invariably due to adenoid

944-12 months baby can do all except:


Walk with support one hand
Can catch with pincer grasp
Can open drawers
Response to calling his name
Can play simple ball

945-max dose of ibuprofen for adult is :


800 if they mean by dose
1600
3000
3200 per day

946-Regarding SEM (standard error of the mean):


SEM is observation around the mean
Standard deviation is measure of reliability of SEM
Is bigger than SD
Is square root of variance
Standard deviation advantage can be math manipulated
standarderror is as a measureof the precision of the sample mean.

947- Indications of surgery in crohns disease


internal fistula
external fistula
intestinal obstruction
Stagnant bowel syndrome
____________________________________________________
948-45 yr old lady presents with nipple discharge that contains blood. What is the
MOST likely Dx:
ductal papilloma
ducta ectasia
fibroadenoma
duct CA

949-after aspiration of cystic mass in the breast the result was clear fluid, next
step
a-Send the aspirated content for cytology and if abnormal do mastectomy
169

b-Reassure the patient that this lump is a cyst and reassess her in 4 weeks
c-Book the patient for mastectomy as this cyst may change to cancer.
d-Put the patient on contraceptive pills and send her home

950-after 2 wks ant. wall MI , old age female developed sudden leg pain , it is pale
& pulsless. Dx :
acute arterial thrombus
acute arterial embolus
DVT
Ruptured disc at L4-5 with radiating pain
Dissecting thoraco-abdominal aneurysm
_____________________________________________________________
951-a 34 yr old divorced lady complains of 15 months amnorrhea , FSH very high ,
Dx :
Pregnancy
ovulation
Premature ovarian failure
Hypothalamic lesion
Pituatary microadenoma

952-he developed severe pain over the wound site , with foul smelling discharge ,
his temp is 39 & HR is 130/min . Gram stain showed G+ve rodes with terminal
spores , ttt
Massive IV pencillin V
clostridium antitoxin
wide surgical debridement
chlormphinicol
wide surgical debridment & Massive pencillin V

953-ectopic pregnancy, all true EXCEPT:


20% ovarian
doubling HCG useful clinical tool
empty uterus + HCG before 12 wks is Dx
laparascopy can dx it
_________________________________________________________
954-a 28 yr lady with 7 week history of amnorrhea has lower abdominal pain , home
pregnancy test was +ve , comes with light bleeding, next step:
Check progesterone
HCG
Placenta lactogen
Estrogen
Prolactin
_____________________________________________________________
170

955-All causes hyperprolactenemia, EXCEPT:


pregnancy
acromegaly
methyldopa
allopurinol
Hypothyroidism
____________________________________________________________________________________________________
____

956-complication of long term use of steroid:


asthma
Breast CA
Other Ca
myopathy in pelvic girdle
osteomalacia

957-All are complications of long term use of phenytoin, EXCEPT:


Ataxia
osteoporosis
Osteomalacia
Macrocytosis

____________________________________________________________________________________________________

958-physiological cause of hypoxemia


hypoventilation
improper alveolar diffusion
perfusion problem(ventilation-perfusion mismatch)
elevated 2.3 DPG>>increase o2 supply in the blood.

959- A 15 yr old boy came to your clinic for check up. He is asymptomatic. His CBC
showed: Hb 118 g/l WBC 6.8 RBC 6.3 (high) MCV 69 (low) MCH (low) Retic 1.2 (13)% what is the most likely diagnosis?
Iron deficiency anemia
Anemia due to chronic illness
-thalssemia trait
Sickle cell disease
Folic acid deficiency
_____________________________________________________________
960-Hb electrophersis done for a patient shows HbA1=58% , HbS = 35% , HbA2 =
2% , HbF = 5 % , Dx :
Thalasemia minor
Thalasemia major
Sickle cell trait
Sickle cell anemia
Sickle cell thal.
171

961-first sign of LSHF

orthopnea
dyspnea on exertion
pedal edema
PND
chest pain

_________________________________________________________________________________________

962-all can cause congenital infection IUGR, EXCEPT:


Rubella
CMV
Syphilis
HSV II
Toxoplasmosis
____________________________________________________________
963-75 yr old female with 2 days hx of MI is complaining of abdominal pain ,
vomiting , bloody stool . X-ray shows abd distension with no fluid level , serum
amylase is elevated. Dx :
Ulcerative colitis
acute pancreatitis
Ischemic colitis
Diverticulitis
_____________________________________________________________
964-CCB drugs like verapmil , dilitazem, nifedipine are effective EXCEPT:
Prinzmetal angina
Hypertension
Atrial tachycardia
Ventricular tachycardia
Effort angina
_____________________________________________________________
965-a 5 day old child vomited blood twice over the last 4 hr , he is healthy , active
& feeding well by breast , Dx :
esophigitis
esophageal varices
gastritis
duodenal ulcer
cracked maternal nipple

___________________________________________________________________

966-5 yr old seen in ER presented with fever & sore throat , which of the
fallowing suggest viral etiology :
Presence of thin membrane over the tonsils
Palpable tender cervical LN
172

Petechial rash over hard or soft palate


absence of cough
Rhinorrhea of colourless secretion
967-The following are complicationsof laproscopic cholecystectomy EXCEPT:
Bile leak
Persistent pneumoperitonium
shoulder tip pain
ascites
Supraumbilical incisional hernia
_____________________________________________________________
968-one of the fallowing drug combination should be avoided :(the same in
alqaseem but all choises are here)
Cephaloridine & paracetamol
Penicillin & probenicid
Digoxin & levadopa
sulphamethaxazole & trimethoprim
tetracycline & aluminum hydroxide

969-40 yr old male presented to ER with 6 hr hx of severe epigastric pain,


radiating to the back like a band , associated with nausea . No vomiting , diarrhea.
No fever . On examination he was in severe pain & epigastric tenderness. ECG was
normal, serum amylase was 900 u/l, AST & ALT elevated double the normal. Which
of the following is the LEAST likely precipitating factor for this patient:
Hypercalcemia
chronic active hepatitis
chronic alcohol ingestion
hyperlipidemia
cholelithesis
970-Which of the following not transmitted by mosquitoes
Rift valley fever
Yellow fever
Relapsing fever (caused by ticks)
Filariasis
Dengue fever
__________________________________________________
971-A on-opaque renal pelvis filling defect seen with IVP , US revels dense echoes
& acoustic shadowing , The MOST likely Dx:
blood clot
tumor
173

sloughed renal papilla


uric acid stone
crossing vessels

972-coarctation of the aorta in commonly associated with which of the


following syndromes?
A)down
b)turner
c)patau
d)Edward
973-46 yr old female presented for the third BP reading, high blood pressure
160/100 . she is not on any medication. Lab investigation showed
Urea: normal
Creatinine: normal
Na=145 (135-145)
K= 3.2 (3.5 5.1)
HCO3= 30 (22-28)
What is the Dx?
Essential hypertension
Pheochromocytoma
Addisons Disease
Primary Hyperaldosteronism
_____________________________________________________________
974-32 yr old lady works as a file clerk developed sudden onset of lower back pain
when she was bending to pick up files, moderately severe for 3 days duration.
There is no evidence of nerve root compression. What is the proper acrion:
Bedrest 7-10 days
Narcotic analgesia
Early activity with return to work immediately
CT for lumbosacral vertebra
975-fracture of rib can cause all except:
pneumothorax
hemothorax
esophageal injury
liver injury

976-anal fissure more than 10 days, which is true:


Loss bowel motion
Conservative management
Site of it at 12:00
____________________________________________________________
977-after delivery start breast feeding :174

as soon as possible
8 hrs
24 hrs
36 hrs
48 hrs

978-All are true for the prescripsion of antidepresents ttt for patient with

depression & somatisation disorders, EXCEPT:


Smaller doses may needed in elderly
potential side effect should not explain 2 the patient , b/c he will develop it
fluoxetine safe for elderly
________________________________________________________________
979-the most specific investigation to detect pulmonary embolism is :
perfusion scan
pul angiogram
ventilation scan
CXR
________________________________________________________________
980-obstructed labor, which is true:
common in primi
excessive caput & molding are common signs
most common occipto- ant
can not be expected before labor
________________________________________________________________________________________________________

981-45 year old female come to the ER complaining of Rt hypochondrial pain which
increases with respiration , on Ex there is tenderness over the Rt hypochondrium,
Next investigation is
X-ray
US of upper abdomen
CT

982- a 48 hour old newborn infant in critical care unit with respiratory
distress and jaundice.HB 9g/dl, retic 4%,. Maternal Hx of previous
normal term pregnancy without transfusion,blood typing shows
hetero specifity between mother and child.Indirect Coombs test
+ve.the most probable Dx is
a- Sickle cell disease
b- Thalassemia
c- Maternal fetal blood mismatch
d- Hereditary genetic disease
e-septicemia.
983-a disease lasts 2-3 wk with fatality rate 30%:
a-incidence=prevalence.
b-incidence >prevalence.
175

c-incidence<prevalence.
d-incidence=1/2prevalence.
e-has no relation.
__________________________________________________________________________
984-what is the least effective AB of the following to staph. Aureus:
a-clindamycin.
b-erythromycin.
c-amoxicillin.(over 80% of staph aureus are resistant to penicillin)
d-vancomycin.
985-35 y/o presented with left iliac pain and dysuria, mangment include
all the following:
a-blood C+S
b-microscopy of urine.
c-IVP.
d-urine C+S
e-norfloxacin.
986-Colles fracture:
a-distal end of the radius.
b-scaphoid fracture.
c-around the elbow.
d-head of the radius.
987-a child fell on an out-stretched hand and flexed elbow,exam showed
swelling around the elbow with no radial pulse, best management :
a-closed reduction.
b-closed reduction then check radial pulse.
c-open reduction.(because of the vessel involvement best way by open
repair)
d-cuff and collar for 3wks.
988-most common association with acanthosis nigricans:
a-hodgkin lymphoma
b-non-hodgkin lymphoma.
c-Internal malignancy.
d-DM
e-insulin resistance.
989-xanthoma:
a-on lateral aspect of the upper eyelid.
b-hard plaque.
c-around arterioles.
d-is not related to hyperlipidemia.
e-deposited in dermis.
990-patient suspected to have brain abscess,the most important q. in the
history is :
a-frontal sinusitis. (contiguous suppurative focus (45-50%)
176

b-ear discharge.
c-head injury.
d-bronchioctasis.
e-Hx. of vomiting.
991-the following are true about H. pylori except:
a-related to gastric outlet incontinence.
b-can cause gastritis but not related to duodenal ulcer.
c-can be eradicated by ampicillin and metronidazole.
d-there will be histological improvement after eradication.
e-it can split urea.
992-60 Y/O lady on OCP 21 days a month having recurrent vaginal
bleeding(spotting) after the stop of estrogen, best Tx:
a-endometrial Bx.
b-papsmear of the cervix.
c-add progestone .
d-stop estrogen.
e-abdominal US or laproscope.
993-most common site of gonococcus infection in females in:
a-cervix.
b-posterior fornix.
c-urethra.
994-post D&C the most common site of perforation is the:
a-fundus
b-ant.wall of the corpus
c-post. Wall of the corpus.
d-lat.wall of the corpus.
e-cervix.
995-regarding typhoid fever, all are true except:
a-fever and red spots appear on the same time .(fever first then rash)
b-can be completely eradicated even in the prescence of gall stones.
c-transmitted by food ,milk and water.
d-can be treated by quinolones.
996-high output HF causes include all except:
a-anemia.
b-MR.
c-AV fistula.
d-pagets disease.
997-a middle aged man having black spots on his thigh for years, it is
starting to become more black with bloody discharge,the best
management is to:
a-wide excision.(malignant melanoma ttt)
b-incisional Bx.
c-cryotherapy.
d-radiotherapy.
177

e-immunotherapy.
998- Old patient .. stopped smoking since 10 years suffering
from shortness of breath after exercise but no cough and
there was a table
Fev1=71%
Fvc=61%
FEV1/fvc=95%
Tlc=58%
What's the dx?
a- Restrictive lung disease
b- Asthma
c- Bronchitis
d- Emphysema
e- Obstructive with restrictive
999-its c/I to stop preterm delivery in the following condition:
a-aminochoronitis.
b-placental abruption.
c-preeclampsia.
d-A&B.
1000-PPH happens more commonly with:
a-multiple pregnancies.(due to increased risk of uterine atony)
b-anemia.
c-preterm delivery.
d-antithrombin iii deficiency.
1001-Before you start instrumental delivery it is important to check if
there is :
a-face presentation.
b-CPD
c-breech presentation.
d-cord prolapse.
1002-in occipitoposterior malpositioning of the fetal head ,all of the
following are true except:
a-10% of all vertex deliveries.
b-it causes significant delay of labor duration compared to the anterior
presentation.
c-andriod pelvis is a predisposing factor.
d-flexion of the head helps the rotation to the ant. Position.
1003-important tools for listening to a patient include:
a-using tools for asking.
b-imagination.
c-using similar words and expressions as the patient.
d-a sense of humor.
178

e-all of the above.


__________________________________________________________________________
1004-the mechanism of action of heparin:
a-activation of antithrombin iii
__________________________________________________________________________
1005-Cover one eye onother go laterally?
Strabismus
Ambylobia
3rd nerve palsy
1006-a 6 year old girl presented with low grade fever and arthralgia for 5 days. She
had difficulty in swallowing associated with fever 3 weeks prior to
presentation.physical examination revealed a heart rate of 150/min and pansystolic
murmur at the apex. There was no gallop and liver was 1 cm below costal margin.
The most likely diagnosis is:
a-bacterial endocarditis.
b-viral myocarditis.
c-acute rheumatic fever.
d-pericarditis.
e-congenital heart failure.
___________________________________________________________________
_______
1007 What is the most common congenital hear disease associated with
rheumatic heart disease ?
Asd
Vsd
Coacotation of aorta
1008- Child with URTI then complained from ear pain on examination there is
hyperemia of TM &+ve insufflations test he tri 2 drug no benefit what is the best
TTT>>>
ugmentine
azithromycin
ciprofloxacin
steroid??
1009-Our target hb A1C >> in DM
6
8
9
10
1010-Check pt after RTA;
LOC
179

1011-3m baby with hx of broncholities >>


cause RSV
1012- - Coronary artery disease atherosclerosis inside
artery or lumen ?!!
___________________________________________________________________
_______
1013- Heart receive blood through >>
aorta constrict
IVC dilate
Increase pul resistance
___________________________________________________________________
_______
1013- Pt fall from stairs >> no air entry
Mange by ;
ENdotracheal entbation
, oxygen ,
1014-Most common cause of otitis media all age >>
staph
strept . pneumonia
1015-child with proptosis , red eye , restrict eye movement , normal examination:
Orbital cellulitis
1016- Psychosis postpartum ?!!
insidious onset
common
usually suicide
1017- HTN with hyperaldosternism ?!!
spironlCtone
1018- Patient admitted as a case of emphysema, according to the vaccine what
you will do
a)give pneumococcal vaccine now
b)give flu vaccine now
c)give all vaccine 2week after discharge
d)give flu vaccine now and pneumococcal vaccine 4week after discharge
1019- what is most sensitive for DX of duodenal ulcer :
a- Epigasteric pain starting 30-60 min after the meal
b- Epigasteric pain staring immediately after a meal
180

c- Increasing of pain when lying supine


d- Pain radiating to the back
1020- The greatest method to prevent the diseases :
a- Immunization
b- Genetic counsling
c- Enviroment modefication
d- Try to change behavior of people toward health??
e- Screenig
1021-/least effect on tardia dyskinesia :
a. halipridol
b.respridol
clozepine
__________________________________________________________________________
__________________________
1022-pt received varicella vaccine after 30 min he developed itching ..
treatment is :
Subcutenous epinephrine
1023-case scenario pt with HZV ttt :
a. acyclovir for 3-5 days
b. acyclovir and refer for ophthalmology
1024- pt with hirshitism , obese , x ray showed ovary cyst she wants to

convieve best treatment :


Clomophine citrate.
1025- adolescent with asymptomatic hernia :
a.surgical is better than medical ttt
b.contraindication to do surgery in reducible hernia
c.can cause hypoinfertilty
1026- pt use antacid ,complain of vomiting and pain due to :
a. GERD
1027- scenario about ectopic pregnancy B-HCG 5000 hemodynamically
stable ttt:
a.observation
b.medical.
c.laproscopy
d.laprotomy.
__________________________________________________________________________
__________________________
1028-effect of niacin is :
a.decrease uric acid .
b.hypoglycemia
c.increase LDL
d.increase HDL
e.increaase triglyceride
181

__________________________________________________________________________
__________________________
1030-pt with DM and obese ,plane to reduce his wt is :
a.decrease calori intake in day time
b.decrease calori and increase fat
c.decrease by 500 kcal/kg per week
d.decrease 800 per day
_______________________________________________________________________________________
______________________________
1031-Romberg sign lesion in :
a. dorsal column
b. cerebellum
c. visual cortex
1032-ttt to increase fetal Hb in sickle cell disease :
Hydroxurea
1033-angioedema due to use of :
B blocker
ACEI
_______________________________________________________________________________________
______________________________
1034-fetal distrees in :
a.early deceleration
b.late deceleration

1035- Pregnant Teacher in her 20th week reported 2 of her student developed
meningitis. Prophylactic Treatment:
a) Observe for the sign of meningitis
b) Meningitis Polysaccharide vaccine
c) Ceftriaxone 500mg PO once
d) Cefuroxime 250 mg IM or IV once
e)Rifampicine 600 mg BD for 2 days
1036- treatment of erosive gastritis ?
a-Antibiotics
b- H2 blocker
c- depend on the pt situation
d- total gastroectomy
e- sucralfate
________________________________________________________________________________________________________

1037-Patient 2 h after delivery have sever vaginal bleeding initial


management ?
Ergometrin
Ringerlactate solution
Blood transfusion
1038- Patient with digoxin toxcicty the most important to give ?
Immune fab
k
182

1039- female pt , with RTA ,she has bilateral femur fracture >>>like this scenarion ,
systolic blood pressure 70 >>>what will you do:
a-Iv fluid
b- blood tranfusion
c1040- Long hx of pt with recurrent vomiting for 2 days
Heamocrit 65 the doctor can report this result caused by
Cytokine
Glucagon
C r protin
Apoprotein
1041-secondary amenorrhea
a-due to gonadal agenesis
b-sheehan's syndrome
c-It is always pathological
1042- pt with DM and obese ,plane to reduce his wt is :
a.decrease calori intake in day time
b.decrease calori and increase fat
c.decrease by 500 kcal/kg per week
d.decrease 800 per day
1043-chickpeas.kidney beans and lentils contain which element of following
bromide
chromium
iron
selenium
1044- what food causes bleeding in a patient on anticoagulates
garlic
spinach
avacados
ginko
1045- Patient on asprin, phenyton for sizuers came to clinic for rotein follow up, on
examination she has bilateral painless lymph nodes, no other symptoms or signs,
lymph node biopsy showed hyperplasia. DDx:
A- chronic lymphocytic leukemia.
B_hodgkin lymphoma
C- TB
D-???
most likely it is side effect of phenytoin
183

1046- Pt complaining of hypotension & bradycardia. Electrolytes show: dec.Na, &


inc. in the following K,Cl,Urea.
So the cause of this is:
a. hyponatremia
b. hyperkalemia
c. hyperchloremia
d. uremia
1047-old pt ,she have MI and complicated with ventricula tachycardia ,
, then from that time recive Buspirone
he came with fatige >>>>normotinsive , pulse was 65
what INX must to be done
a- thyroid function
b- liver and thyroid
1048 patient with hypopigmented macules.loss of sensation.thickend
nerves.diagnosis was leprosy.which type
tuberculoid
lepromatous
borderline
Loss of sensation is a feature of tuberculoid leprosy, unlike lepromatous leprosy, in
which sensation is preserved.)
Emedicine
1049- An active 64-year-old male complains of dysphagia. Endoscopic biopsy
confirms esophageal squamous cell carcinoma. A predisposing factor to this
condition is:
a) Hiatus hernia
b) Achalasia
c) esophageal varices
d) Diffuse esophageal spsm
e) Mallory-Weiss syndrome
1050- ulcer on the nose with averted edges
basal cell ca
herper simplex
1051- Boy with nocturnal enurisis psychotherapy fail to show result yo will sart him
on :
Imipramin vassopressin
Imipramin guanfacin
Clonidin vasopressin
Clonidin guanfacin
184

1052- pt 2 para 2 gravida had pco took cyclic progestrone,she is ona increased risk
of
endometrial cancer
cervical dysplasia
hip fracture
___________________________________________________________________
_______1053- smelling sputum increase with lying down + clubbing
1-bronchectasis
2- ba
3- pneumonia
1054-2.female presented to er with HCL burn on her face there was partial thickness
burn.management
irrigation with water
irrigation with soda bi carb
immidiate debridement
1055-young female with left sided abdominal pain.no dysuria or change in bowel
habit.history of hysterectomy 4yrs back but ovaries and tubes were preserved.on ex
abd tender but no guarding.inv show leucocytosis and few pus cells in urine.there
was also history of unprotected coitus with multiple partners.
i didnot get the scenario well but i think it was salpingitis.
management
consult surgeon
oral antibiotics
diagnose as ulcerative colitis
___________________________________________________________________
_______
1056-a picture of JVP graph to diagnose.patient had low vol pulse,low resting bp.no
murmr.pedal edema.
constrictive pericarditis
tricuspid regurg
tricuspid stenosis
pulmonary hypertension
___________________________________________________________________
_______
1057-treatment of psoriasis:
topical steroid
___________________________________________________________________
_______
1058-picture of pelvic x ray what is diagnosis
normal
paget disease
spondylitis
osteoporosis
185

1059- pt with vomiting, constipation pain and distention past history of


appendectomy 7 month ago
dX
1- mechanical IO
2- ileus
1060- In Pedia ,,IV fluid (LR) can be given at age:
A 3 months
B 8 months
C 12 months
D 24 months
1061- old aged female with atypical squamous cells of undetermined significance
(ASCUS) on pap smear, started 30 day ttt with estrogen & told her 2come back after
1 weak, & still +ve again on pap smear, what's next:
- vaginal biopsy
- endometrial biopsy
- syphilis serology
coloposcopy and cervical biopsy cone
1062-advise to pt. to avoid food high in cholestrol
- liver
chicken
tuna
egg white
1063- pt with meniere disease advised to take
low salt no caffiene
low salt high caffiene
high salt no caffiene
high salt low caffeine
1064- most common psychiatric condition come with mania ?
paranoid
grandiosity
1065- bad breath smell with seek like structure, no dental caries & Ix are normal,
what's the likely cause:
- cryptic tonsillitis
- Sojreen's synd.
1066- 24 y. Female with new Dx of DM2, she weared glasses for 10 years, you will
advice her to follow ophthalmic clinic every:
- 6 months
- 12 months
186

- 5 years
- 10 years
1067- - normal child ,he want to walking , he have brother dead after walking ,
what of the following must be excluded before walking ???
a-PDA
b-VSD
C-hypertrophic cardiomyopathy
D-!!!
1068- The best ttt for binge eating disorder:
- cognitive - behavioral therapy
- problem - solving therapy
- interpersonal therapy
_
1069- female pt ,KCO rheumatic heart , diastolic murmur ,complain of aphasia and
hemiplegia ,
what will you do to find the >>>etiology<<< of this stroke:
a-MR angiography
b-Non-contrast CT
c-ECHO
D-ECG
E-carotid doppler
1070- A case of a patient with polycythemia and develop itching after taking a bath ..
a. increase histamine sensitivity
b. abnormal histamine release
1071- pt with DM and obese ,plane to reduce his wt is :
a.decrease calori intake in day time
b.decrease calori and increase fat
c.decrease by 500 kcal/kg per week
d.decrease 800 per day
1072- the most common complication following hemorrhoidectomy is :
a. fecal impaction
b. bleeding
c. urinary retention
d. infection
1073- Pediatric came to you in ER with wheezing, dyspnea, muscle contraction
(most probably asthma), best to give initially is:
1. theophylline
2. Albuterol nebulizers
187

3. oral steroids
4. oxygen
1074- ld man presented to u complaining of rectal pain mostly at night with itching ..
what is the Dx:
a. Hemorrhoids
b. Gay bowel syndrome
c. Proctalgia fugax
1075- 28 yrs old known case of sickle cell anemia hospitalized two times in the last
two months because of abdominal pain, this time he present with abdominal pain,
back pain, and chest pain.. what will you do:
a. Hospitalize the patient and give him analgesics and observe him
b. give him IVF and treat him as an outpaient
c. Referred the patient to Tertiary center specialized in his problem
d. Give analgesics
e. blood transfusion
__________________________________________________________________________
1076- diabetic pstient with ulcer in foot , not healing , not infected , high
? blood glucose
a- high blood glucose stimulate bacteria to grow
b- decrease phagocytosis
c- dec. Immune system
1077- on flow cytometric analysis of a sample of fetal thymus a certain population
of cells is identified that is positive for both cd4 and cd8 cell surface Antigens .
These cells are best characterised as which of the following cells ?!
A- immature cortical T lymphocyte .
B- mature cytotoxic T lymphocyte.
C- mature helper T lymphocyte.
D- antigen presenting cells.
E- Natural killer (NK) cells .
1078- A case of a man who ride a motorcycle and make an accident then had a
basal skull fracture .. he developed a loss of taste, and loss of sensation in the
Anterior 2/3 of the tongue, and deviation of the angle of mouth .. if u will choose one
nerve injury .. which nerve u will choose:
a. CN I (Olfactory)
b. CN III (Oculomotor)
c. CN V (Trigemenal)
d. CN VI (Abducens)
e. CN VII (Facial)

188

1079- dysfunctional uterine bleeding :


a. most common in postmeneposal women
b.adolesent
1080- celiac disease severe form involve
1. proximal part of small intestine
2. distal part of small intestine
3. proximal part of large intestine
4. distal part of large intestine
1081- pt with moderatly sever acne valgarus best ttt
oral isotretinoin
topical retinoid
topical clindamycine
oral antibiotic
1082- Contraindicated in acute glaucoma management:
a. Pilocarpine
b. Timolol not sure
c. B-blockers, CA inhibitors, NSAID, Mannitol
d. Diprovin ??
e.
contraindicated drugs(topiramate antiepliptic ,steroid eye, tricyclic
antidepressant,Parkinson drugs,atropine,antispasmolytic
,antipsychotic
D.... it is DIPIVEFRIN (alpha agonist). Used in open angle glucoma and contraindicated in acute
angle glaucoma
The same question came to my friend and the choices were: Pilocarpine - Timolol - Dorzolamide Flvin...(and this was the answer)

1083- a colorectal carcinoma that invades the submucosa and has two positive
lymph nodes and no metastasis is :
a. stage 1
b. stage 2
c. stage 3
d. stage 4
1084- long case Pt.obese and newly Dx by FBS> 126 with long list of lab come to
me in the exam screen all normal including liver function test
On examination: pt had palpaple midly enlarge liver what you will give him:
a) Biguanieds
b) Sulphanylurea
1085 what is the definition of insomnia?
inability to have immediate sleep when you are very tired
189

disturbance of sleep cycle??


inability to get sleep even if you take medication
disturbance of sleep rhythm that person sleepy at daytime and
insomnic at night
1086 which of the following is the first test that should be performed in a
patient with lower GIT bleeding?
a. nasogastric aspiration
b. anoscopy
c. proctoscopy
d. colonoscopy
1087-pt with constipation . He had previous surgery in the past ( there is Xray )
a- surgery for obstruction
b-rectal decompression
c- treatment of Ileus
1088- yr old in his normal state of health presented with decrease visual acuity
bilaterally without any defect in visual field his VA Rt eye= 20/100 VA Lt eye=20/160
fundoscopic exam showed early signs of cataract and drusen with irregular
pigmentations. No macular edema or neovasculirization. The appropriate action
beside antioxidants and Zn is:
a. Refer the pt for emergency laser therapy
b. Refere the pt for cataract surgery
c. See the patient next month
d. No need to do anything
1089- Male patient present with exercise intolerance, HG is 9 and MCV is 78 and
positive fecal occult test. Upper GI scope show chronic gastritis. How u treat him:
Oral iron?
IV iron
Blood transfusion
1090- Newborn, 28 w, 900 g, successfully resuscitated in delivery room , then in
NICU he had PH 7.35, Pco2 42, Po2 63, what to do first:
IV bicarbonate infusion
Phototherapy
IV vancomycin
Start entral feeding
glucose infusion
1091- female pt ,KCO rheumatic heart , diastolic murmur ,complain of aphasia and
hemiplegia ,
what will you do to find the >>>etiology<<< of this stroke:
190

a-MR angiography
b-Non-contrast CT
c-ECHO
D-ECG
1092- female with positive urine pregnancy test at home what next to do:
Sreum beta HCG
CBC
1093- Infant in respiratory distress ,hypercapnia , acidosis & have rhinitis ,
persistent cough +ve aglutenation test & the doctor treat him by ribavirin DX
pertusus
RSV
1094- ptn with pharyngo tonsilities he took antibiotic and improved in 2 days <the full
course of antibiotic should be for:
1- 5 days
2-7 days
3- 14 days
4- 10days

1095- 43y old female with irregular menses 3m back & 1-2d spotting
what is
next to do:
US
Human chorionic gonadotropin
Placental ,,,,,,,,, ,,,,,,,,,,,,,,,,,,,
FSH
LH
1096- Drug that will delay need of surgery in AR:
a. digoxin
b. verapamil
c. nefidipine
d. enalpril
1097-Notching on the lower edges of the fourth to the ninth ribs indicate enlarged
intercostal arteries eroding the lower border of the ribs in cases of
coarctation of the aorta
1098-- An outbreak of TB as a prophylaxis you should give :
a) Give BCG vaccine
b) Rifampicine
c) Tetracycline
d) H. influenza vaccine
1099-n old man who had stable angina the following is correct except:
a) angina will last less than 10 min
b) occur on exertion
c) no enzymes will be elevated
d) will be associated with loss of consciousness

__________________________________________________________________________
191

1100-5 yr old adopted child their recently parents brought him to you with white nasal
discharge. He is known case of SCA. What you will do to him:
a) Give prophylactic penicillin

__________________________________________________________________________
1101-a

patient with acne of several appearances open .. closed .. red .. it is most likely:
a) obstructive
b) inflammatory
1102-70 yr old presented with wt loss, fatigue, anemia , upper quadtrant pain without
any previous history, the stool sowed high fat he is a known somker:
a) Acute pancreatitis
b) Chronic pancreatitis
c) Pancreatic carcinoma

1103- ATRIBUTED risk:


Difference btw incidence in exposed and incidence in non exposed
1104-tricky case:: young female with pain during menstruation and mood
PMS
A) 1 ry dysmenorrheal
1105-gush of bleeding in 9th month:
a) Placenta privia
1106-best way in advance age to increase bone density in OA:
a) Walking ................
b) High repletion and low resistance

1107-pt ttt from endocarditis likely of recurrence :


12%
1108- psycatric pt see alien talke to her and insertion of idea:
Start antipsychotic ttt
1109- best way to kill dust mite
Wash sheet and clothes in hot water
_______________________________________________________________________________________
______________________________
1110- which one make you relief when you aspirate a Brest mass:
Clear serous fluid in the needle
_______________________________________________________________________________________
______________________________
1111- mastitis with lactation:
Cont. brest feeding
1112-pt. with celiac spore best diet:
Gluten free
_______________________________________________________________________________________
______________________________
1113-Farmer with allergic conjunctivitis in spring and he can't avoid working what to
advice to do at night
a) Cold eye compression sure
b) Other not include antihistaminic
1114-To measure the cognition in old pt:
a) Clock test
b) Memory test my answer but I think A is the right answer
192

c) Other I did't remember


1115-0-Blue swelling below tung or inside lower lip:
A. Ranula
_______________________________________________________________________________________
______________________________
1116-pt with elevated TSH and elevated T3,T4
2ry hyperthyroidism
_______________________________________________________________________________________
______________________________
1117- Reflux esophegitis:
Mimic heart dis.
_______________________________________________________________________________________
______________________________
1118- Barrit esophagus how to precced:
f/u endoscopy for ............... to look for change in metaplasia


1119-- About lyme to educate parents
Wear long clothes to avoid bite my answer
_______________________________________________________________________________________
______________________________
1120- Child with elbow and knee pain with decrease ROM ..in the back < 1cm red
macule
Lyme dis.

1121-- 19 y.o yong male with good body and well muscular with bad mouth brith
c/o
Acne:
He use anabolic steroid
1122- Child his mother let him to go to bathroom befor sleeping and avoid drinking
befor sleep this mangment of:
Enuresis
1123-Pt with meningitis what to give initially:
Penicillin
1124- pt with liver dis. Jaundice Bx showed fibrosis which diet is good for him:
Low protein diet not sure
1125-2months infant with white plaque on tongue and greasy ,past h/o
clamydia conjunctivitis after birth treated by clinamycin what is ttt:
a. Oral nystatin
b. Topical steroids
c. Topical acyclovair
d. Oral tetracycline
1126-psycatric pt . with liver imparment best to give ?:
a) lithum
1127- man walking in street and saying bad words to stranger , he is
not aware of his conditiond he kept doing that as if he asked to , what
is the description :
a. flight of idea
193

b. insertion of idea
c. loosening of association

1128-a mother came with her son who is 7 years old with poor
concentration. Lack of intelligence and play and repeat some of his
action .....................forget the rest
a) Autism
b) Hyper active disorder

1129-pt with vesicle in mouth with gingivits and also vesicle in arm and leg
most likely cause
a.HSV type 1
b. HSV type 2
1130-early symptom of heart failure :
a) Orthopnia
b) PND
c) Palpitation
d) Chest pain

1131-young adult obese with snoring when sleeping and some time get up from
sleeping
Best mangment :
a) Reduce wt
b) tonsillectomy
_______________________________________________________________________________________
______________________________
1132- young adult in endemic area cripitation bilaterally with monopheseal sound in
auscultation what to give vaccination :
a) Hemophlous influenza
b) Meningococcal
_______________________________________________________________________________________
_____________________________
1133- pt with frothy hemoptysis, palpitation >>>>>>>>>>> forget the rest it's long
scenario
a) Mitral stenosis
b) Congestive heart failure
c) CAD
Mitral stenosis cause frothy hemoptysis on cardiac exam>>middiastolic murmur in
apex and malar rash on cheek
Congestive heart failure is an important cause of pink frothy hemoptysis on cardiac
exam>> S3+ basal crept.

1134-most benign vaginal bleeding:


a) Cervical polyp
b) Myomyoma
c) Endometrial hyperplasia

194

d) I forget the exact sentience but it's related to dryness in menopausal women
(atrophic vaginitis)

1135- non hormonal ttt of premenopausal flushes:
paroxitine
1136-mother after delivery have bad mood , depression , crying a lot for only 1
week , but she is o.k now Dx:
A. Maternal blues (transiet condition)
B. Post partum psychosis
1137- pt with pruritic foliclitis best ttt:
A. Local antibiotic xx
_______________________________________________________________________________________
______________________________
1138- pt. e hair loss in the rt. Temporal area. On examination there is elevated mass
with come spacious yellow crust:

A. Spacious cyst
B. Tricotelomania

1139-young male athletes with palpitation and............. maybe syncope and the fail
to do for
Him something.......i'm sorry :
A. Hypertrophic cardiomyopathy
_______________________________________________________________________________________
______________________________
1140- which one of the congenital heart dis. Have least complication with............... I
think endocarditis:
A. ASD
B. VSD
C. PDA

1141- child fall from stairs came with mild injury to the nose, no bleeding and
edema in the nasal sputum , ttt :
a- Nasal packing
b- Reassure
c- Analgesia
d- Refer to ENT
1142- eye exam. corneal ulceration. her symptoms freq. repeated.. which on of the
following is triggring for recurrence of her symptoms:
1- Dusts & pollen
2-HTN & hyperglycemia
3-dark and driving at night
4-ultraviolet light & stress ( this is the answer 100 % )
1143- What is special about placenta abruption:
a. Vaginal bleed
195

b. Fetal distress
c. Uterus pain and back pain
d. Abnormal uterine contraction
1144- 24 Y/o man presented with 4 month Hx of diarrhea with streaks of blood &
mucous. Ulcerative colitis was confirmed by colonoscopy. The initial therapy for this
patient:
a)oral corticostreiod
b)azathioprine
c)infleximabe
d)5-Aminosalicylic acid
e)Sulfasalazine
1145- I study done on 10,000 people for about 3 years in the beginning of the study
3,000 developed the disease and 1,000 on the end of the study what is the
incidence:
100
12.5
10.5
0.1
1146- 45 years old female came to ER with acutely swollen knee +
ballotment patella .. The most important to do is:
MRI of the knee
Aspiration
Complete blood count
Rhumatoid factor
1147- peritioial lavage when to say the amount is suffusion :
2 l blood
1000 wbs \ rbs
500 wbs
1148- 19- What best explain coronary artery disease:
a- Noatherosclerosis
b- Fatty deposition with widening of artery
c- Atherosclerosis with widening of artery
1150- Incidence is calculated by the number of:
A. Old cases during the study period.
B. New cases during the study period.
C. New cases at a point in time.
D. Old cases at a point in time.
E. Existing cases at a study period.
1151-9- secondary prevention one true:
a- physician screening quetionaire about the use of tobacco is sufficient
b- the screening of colon cancer is insufficient
c- the screening of breast cancer is decreasing
d1152-Which drug contra indication in cluster headache ?
a. Buperbion
196

b. Lithum
c. valium
1153-Most common cause of recurrent tonsillitis is :
- Group a beta hemolytic streptococcus
- The other choises are virusis..
parainfelunsa
rhinovirus..
http://en.wikipedia.org/wiki/Rhinovirus

1154Child with leukemia he has septicemia from the venous line the organism
is:
a) E coli
b) GBS
c) Pseudomonas
1155-Baby born & discharge with his mother , 3 weeks later he started to
develop difficulty in breathing & become cyanotic what is most likely
DX :
a- VSD
b- Hypoplastic left ventricle
c- Coarctaion of aorta
d- Subaortic hypertrophy
1156-Man who is having a severe pain on his big toe with knee pain
and examination revealed negative perferingent crystals:
a) uric acid deposit secondary to synovial fluid over saturation
b) Ca pyrophosphate secondary to synovial fluid over saturation
1157- 6 years old child presents with straddling gait and in ability to stand or walk
without support, he is irritable with vomiting 3 times, he has a history of chickenpox
3 weeks ago. O/E all are normal except resistance when trying to flex the neck, what
is the most likely diagnosis:
Fradrich's ataxia
Acute cerebellar ataxia
Meningioecephalitis
Gullian Barre syndrome
http://en.wikipedia.org/wiki/Acute_cerebellar_ataxia_of_childhood#Etiology

1158-a man fell down from the ladder, c/o SOB ( and i think cyanosis ), on
exam breath sounds are decreased even in the right side ( this is how
they wrote it !! ), u will do :
needle thoracotomy
insert endotracheal tube
other options i forgot
1159- Which of the following is true regarding antepartum (third trimester )
hemorrhage :
a- Can be caused by polyhydrominos
197

b- Rare to be associated with hypofibrogenemia


c- Cervical problems are a major cause
1160-case infant has genital rash ( the rash spares genital fold ) not
response to antibiotics , most likely Dx;
A-candida albicans
b-napkin dermitis (diaper dermatitis)
c-contact dermatitis
d- atobic dermatitis
e- sebborich dermatitis
1161- Male patient complain of excruciating headache, awaken him from sleep
every night with burning sensation behind left eye, lacrimation and nasal congestion.
What is effective in treating him :
Ergonavine
Sumatriptan SC
Methylprednisolone
NSAID
1162-34 yo female presented with cough , dyspnea for months , exam show
cervical adenopathy and hepatomegally . To confirm the likely
diagnosis you will do ..
1- liver biopsy
2- bronchoscopic lung biopsy
3-scalene nodal biopsy
4- ACEI level.
yes, lung biopsy to confirm the diagnosis of sarcoidosis.
1163Fresh frozen plasma in what case ?
a- Hemophilia a
b- Hemophilia b
c- Von willbrand
d- Dic
e- Coagulopathy form liver disease
1164which of the following cause hirsutism ?
a- Anorexia
b. Digitalis
c. clomiphine citrate
d- OCP
_______________________________________________________________________________________________________

1165- pt with epilepsy came with Lt shoulder pain , on examination flattened


contour of the shoulder, fixed adduction with internal rotation .. ur DX ?!!?
198

a- Inferior dislocation
b-subacromal post Dislocation
c-subglenoid ant dislocation
d- subclavicle ant dislocation
e-sub.. ant dislocatio
1166-pt with asthma on daily steroid inhaler and short acting B2
agonist what category:
a- Mildintermittent
b- Mildpersistent
c- Moderate
d- Sever
1167-Pt with active hepatitis what medication should not to give
:
a- Ranitidine
b- Heparin
c- Atrovastsin
1168-A man who is having severe vomiting and diarrhea and now developed leg
cramps after receiving 3 liters of dextrose .. he is having:
a) hypokalemia
b) hyponatremia
c) hyperkalemia
d) hypernatremia
1169-Pneumococcal vaccine :
A not recommended in healthy child
B cant be given with MMR
C cant be given to child less than 2 years
D if given to sickler and exposed to infection has to take pencilin
1170-Long scenario for pt smokes for 35 y with 2 packets daily, before 3 days
develop cough with yellow sputum, since 3 hours became blood tinged sputum, X
ray show opacification and filtration of rt hemithorax, DX:
Bronchogenic CA
acute bronchitis
lobar pneumonia
1171-Post partum female with recurrent attack of hearing loss , which diagnosed as
conductive hearing loss , on CT the is dehesion in the of semi circular canal
diagnosis >>>>
otosclerosis
miner's
Tuberus sclerosis
199

1172- True about dermatomyositis :


1-associated with inflammatory bowl dz
2-indicate underlying malignancy
3-present as distal muscle weakness
1173- newborn presented with conjunctivitis and O.M , whats the treatment
i guess this is a case of infection with chlamyedia intrauterine , they asked about
several AB
there is no dyoxycyclin nor erythromycin
1174- Differences Btw dementia and delirium (read about it )
i picked Amnesia !!
1175- about shoulder that is Adducted and internally rotated (what is the
mechanism of dislocation)
1-Anterior subclavicular
2- Ant
3-Post
4-POst
1176- anteriolateral placenta , term pregnancy , can't be felt when examiner admit
his finger through the Cervix :
1-Low set placenta ??
2-Marginal
3-normal
4-complete placenta previa
1177- Tinea capitis RX.
1-start Nystatin
2-wood's lamp (true)
1178- Rosacea case (redness patch on face with talangectasia ) what is the ttt
Dyoxycyclin
1179- child smile at
1- at birth
2- 1month
3-2 months
4- 6 months
___________________________________________________________________
_______
1180- pt has GERD for 5 years , now EGD reveals >> columnar cell surrounded by
Sq cell
1- Sq .c.c
2-Adenocarcnoma
3-barret esophgous
1181- old pt , has loin pain , U/S reveals bilateral hydroneprosis , whats the cause :
1-prostate cancer
2- bladder cancer
3- urethral stricture
200

1182- infertile women for 3 years with dysparunia


1- endometritis
2- Salpengitis
3-endometriosis (True)
1183-difficult consultation :
1- use medical term
2- open ended Q
3-close Ended Q = True
41184- hematology case prophral blood smear reveals target cell =SCD
1185-old pt with progressive weakness of hand grip , dysphagia , .
1-MG dz
2- Mysthenia gravis sx .
1186- male singer with colon cancer stage B2 ; which of the
following correct ?
a- no lymph node metastases
b-one lymph node metastasis
c-2 ===
d-lymph node metastasis + distant metastasis
1187- Young boy presented to the ER with inguinal mass, pain and vomiting. O/E
the mass is tender to touch, erythemetous skin over scrotum, (blue dotes) in the
pole of testis, intact cremasteric reflex , Dx is :
a- Testicular torsion
b- Testicular hematoma
c- Incarcerated herniad-torsion appendix of testis
1188- A middle age man presented with severe headache after heavy lifting
objects. His BP was high. He was fully conscious. Examination was otherwise
normal. the most likely diagnosis is:
a) Subarachnoid hemorrhage
b) Central HTN
c) Tension headache
d) Migraine??
e) Intracerebral hemorrhage
1189- PT WITH UTI ALLERGIC TO SULFA AND PENICILLIN ?
NITROFUNTON
CEPHLAXINSMT
1190 bad breath smell with seek like structure, no dental caries & Ix are normal,
what's the likely cause:
- cryptic tonsillitis
- Sojreen's synd.
1191- use of antibiotic in acne :
A - to prevent spread
B- to decrease scaring
201

1192- a 38 yrs old female came to you at your office and her pap smear report
was unsatisfactory for evaluation .. the best action is
a- consider it normal & D/C the pt.
b- Repeat it immediately
c- Repeat it as soon as possible
d- Repeat it after 6 months if considered low risk
e- Repeat it after 1 year if no risk
1193- Mass in the upper back .. with punctum and releasing
white frothy material
a- It's likely to be infected and Antibiotic must be given before
anything
b- Steroid will decrease its size
c- It can be treated with cryotherapy
d- It must be removed as a whole to keep the dermis intact??
1195- A pregnant lady full term presented with agitation and change level of
consciousness she is having difficulty breathing:
a. Pulmonary embolism??
b. Pulmonary edema
c. Amniotic fluid embolism
1196-Pt presented with nausea and vomiting and nystagmus with tinnitus and
inability
to walk unless he concentrate well on a target object. His Cerebellar function is
intact:
a. Benign positional vertigo
b. meniere's disease
c. vestibular neuritis
1197-tympanic mem. perforation in cases of cholesteotoma are commonly
situated in :
a- anterior part of mem. tensa
b- centre of mem. tensa
c- posterior superior segment
d- posterior inferior segment of the tympanic membrane
1198-Peripheral neuropathy can occurin all except:
a) Lead poisoning
b) DM
c) Gentamycin
d) INH
1199-child with 2 * 2 cm hair loss at the temporal area , normal examination ,
microscopic examination of hairs arround the area show clubbed and attenuated
hairs , the diagnosis is :
a- tinea capitus
b- alopecia areata
c-Trichotillomania
202

d- Telogen Effluvium
e-?
1200- What is the most important in councling
a. Exclude physical illness
b. Establishing rabbot
c. Family
d. Schedule appointement
1201-year old had an episode of rheumatic fever without any defect to the
heart. The
patient need to take the antibiotic prophylaxis for how long:
a. 5 months
b. 6 years
c. 15 years
IF :Rheumatic fever with carditis but no residual heart disease (no valvular disease)
10 years or until age 21 years (whichever is longer).......IF: Rheumatic fever without carditis
5 years or until age 21 years (whichever is longer)
http://www.aafp.org/afp/2010/0201/p346.html

1202- Pt daily asthma , nothing at night, using herbal for 2


months with no improvement :
inhalation salbutamol(the best answer)
, high
dose steroid inhaler
, ipratrubium
1203- when an a cyanotic middle age adult has roentgenographic evidence of
enlarged pulmonary arteries and increased lung markings, most likely diagnosis is:
a) Ventricular septal defect(according to sle studing)
b) Coarctation of aorta
c) Pulmonary valvular stenosis
d) Atrial septal defect
e) Truncus arteriosus
1204-all of the following will improve the patient compliance except:
a-making the appointment convenient?
b-simplify the regimen
c-writing the instructions clearly
d-tell about the danger of missing doses
e-involve the patient as active participant
1205- drug of choice of genaralized anexity dis. Is:
-acetalopram
-bubropione
-buspirone
-beat blocker
1206- Uric acid in body how the body removed by
a-increase co2 execreation of uric acid in urin
203

b-increase metabolism of uric acid in liver


c-execretion of uric acid by lung
In humans and higher primates, uric acid is the final oxidation (breakdown) product of purine
metabolism and is excreted in urine

1207- A man travelled to Indonesia and had rice and cold water and ice cream.
He's now having severe watery diarrhea and is severely dehydrated. Most likely he
has:
A. V. Cholera
B. C. Difficile
C. C. Perfringens
D. Dysentery?
E. Shigella
1208- Newborn has vomiting after every meal intake. The examination was normal
and the only abnormality was dehydration. No other clinical signs. No tests ordered
yet. What will you do?
a- Order abdominal CT
b- Reassure the parents
c- Refer to GS
d- Discharge on ORS
1209- child with SOB and runny nose came with fever (38 ) all the sign of
respiratory distress there .. There is diffuse wheezing on the chest with prolonged
expiration and inspiratory cracles ,, diagnosis:
-viral pneumonia
-bronchiolitis
-croup
-bacterial pneumonia
1210- pt came with fatigue , w.t loss and diarrhea .. He recived a blood transfusion
when he was in kenea .. He has low grad fever .. The vitals are stable .. Skin EX.
There is contagiosum mollosum in groin ( i guess it written like this ) .. There is
generalized lymphadenopathy and palpable liver ,, what is the diagnosis:
-secondary syphilis
-persistent chronic hepatitis B
-HIV
-acute lymphoma.
1211- the best description of the lesion in herpes :
-soft tender chancer
-firm non-tender chancer
-raised tender papule
-vesicle with fluid material
-................
204

1212- penis numbness after sitting for long time


Pudendal Nerve Entrapment
1213- What is the organism that will growth in the agar in a sample from a cat
bite ?!
salmonella
bacteroid species ??
streptococcus
1214- 5 y/o child had abdominal blunt trauma, doctor confirm presence of
intramural hematoma in 1st and 2nd parts of duodenum, high amylase, Mx ?
a- CT-guided hematoma drainage
b- duodenal resection
c- exploratory laparoscopy
d- don't remember.. I think conservative
1215- pt with recent Hx of URTI , develop sever conj. Injection
with redness, tearing , photophopia ,
So, what is TTT ?
a)Topical ABx
b) Topical acyclovire
c)Oral acyclovire
d) Topical steroid
photophopia is caused by adenovirus treated by topical steroid
1216- Pt came with mild abdominal pain, headache, oral ulcer, joint
pain,thrombophlebitis of leg
Ulcerative cholitis
SLE
Behchet
1217- Patient with echolalia, echopraxia, poor hygiene, insomnia, and weird
postures. Treatment? (catatonia)
A. Lithium
Benzodiazepines are the first line of treatment

1218- 43 yr old lady complaint about non itchy;white nonsmelling vaginal discharge
after intercourse;she z nt using any contraceptive or vag douche;what z diag??
A;prescibe azithromycine
b;local steroids
c;local antifungal
d;vaginal douche
e;do nothing
1219- If a patient of Magrine head ache has nt been treated;which condition do u
suspect the patient will develope??
205

A.hearing loss
b.depression
c.dysphagia
d.loss of vision
1220- a lady patient otherwise healthy complaint a hissing sound n her ears at
night during sleeping;her bedroom is sound proof and no noise coming from outside.
Diagnosis?
A;migrine
b;otosclerosis
c;otitis media with effusion
d;tinnitis
1221- acute diarrhea with epithelial infiltration
E- coli
Salmonella
Cholera
Rota virus
Shigella
1222- Boy presented with painless neck mass, 5-week history of fatigue,
generalized pruritis, and mild cough. Dx?
A. Hodgkin's lymphoma
B. Lyme's
C. Infectious mono
1223- what is the organism hat cause meningitis in college dormities :
1-h.influenza
2- nisseria gonorrhea
3- strep. pneumonia
4- staph. aureus
Neisseria meningitidis bacteria (meningococcal disease)
1224- a 19 yr sexually active lady came for her annual check up:she z otherwise
healthy using no contraceptive;her pap smear and all investigations are normal;what
will u suggest regarding her next check up???
A;afer 6 months
b;after 1 yr
c;after 3 yrs
d:after 5 yrs
1225- 6 yr old school going boy complaaint abt itchy scalp;n school his 10 friends
have the same problem:wt z ur diag??
A;lice (Pediculus humanus capitis)
b;tinea capitis
c:seborric dermatitis
d:scabies
__________________________________________________________________________
1226- Nonmedical treatment of premature ejaculation??
the use of acupuncture
206

1227- A 54 YO female with chronic pelvic pain is found to have a right sided
ovarian mass. After the initial evaluation, surgery is planned to remove the mass. To
avoid excessive bleeding during the surgery , the surgeon should ligate which of the
following structures?
A) Round ligament
B) Suspensory ligament
C) Ovarian ligament
D) Transverse Cervical ligament
E) Mesosalpinx
Suspensor ligament of ovary contains the ovarian artery, ovarian vein,[1] ovarian plexus,[4] and
lymphatic vessels.[3]

1228- leukemia with blast cells:


AML
ALL
CML
CLL
1229-cloboma when to do the operation>>
1stwk,
3month,
1 year
1230- Table with investigation
Na 112
Osmolality 311 low
What is the diagnosis?
a- Connssyndrome
b- Cushingsyndrome
c- SIADH(the only choice with low na)
d-Diabetes insipidus
1231- Female wants a temporary contraceptive method, which one is recommended
by research?
A) OCP
B) IUCD
1232- pt sustained major traum came to Er 1st thing to do
open air way give 2 breath
open airway remove foreign body
2 breath fowlled by chest compression
chest com after feeling pulse
1233- PT DYSMENORRIA DYSPARUNIA INVERTED UTERUS ???????
ADENOMYOSIS
-ENDOMETRIOSIS
-UTERINE LIOMATA207

UTERINE
CARCINOMA
1234-In moderate to severe asthmatic patient , u will find all the following EXCEPT :
A PO2 <60
B PCO2 > 60
C low HCO3
D IV hydrocortisone will relieve symptoms after few hours
E dehydration
1235-12 year old female , non pruritic annular eruption in the right foot for 8 months
, looks pale and not scaling , no response to 6 weeks of miconazole
A discoid lupus erythramotosis
B erythema nodosum
C tinea corporis
D granulomatous annulare
E choricum marginatum??
1236-A man who bought a cat and now developed watery
discharge from his eyes he is having:
a) Allergic conjuctivitis
b)Atopic dermatitis
c)cat scratch disease
1237-Senario about wilson (kayser-fleischer ring, low ceruloplasmin)Rx:
A)desferrioxamine
B)penicillamine
1238-effective ttt of mastalgia ?
a- Caffeine
b- OCP
c- tamoxifen
d-danazol
1239-Pt with abdominal pain and distension with vomiting and
constipation. He has mild symptoms of dehydration. There is
evidence of air in the rectum. The Rx:
a. Rectal decompression with IV antibiotics
b. Nasogastric tube with IV isotonic fluid
c. Systemic antibiotics
1240-5 months old baby , in ER with sudden abdominal pain , pain last 2-3 min with
intervals of 10-15 mins between each attack :
A- intussesption
B- infantile colic
C- appendicitis
1241-Patient with a scenario going with liver cirrhosis with acsites, diet instructions:
High carbs, low protein
Sodium restriction
208

1242- which of the following is used in minor burn:


a- apply cold water
b- apply room temperature water??
c- debridement
1243-Pt presented with severe hypothyroidism & serum sodium = 108. What do u
do?
a- Intubate, give 3% sodoium then treat hypothyroidism status
b- treat hypothyroidim & monitor S.NA level every 6 hours
c- Give 3% sodium, hydrocortisone & treat hypothyroidism status
1244-pt. with primary biliary chirhosis wich drug helps the histopathology of the
liver:
- Steroid
- Interferon
- Ursodiol
__________________________________________________________________________
1245-patient complaining of gradual loss of vision & now he can only identify light.
Which of the following is the LEAST cause of the problem ?
a retinal detachement
b central retinal artery embolism ??
c vitreous hemorrhage
d retinitis pigmentosa
e retrobulbar neuritis
1246- Patient with DM and HTN, gradually decreasing vision. Eye exam shows
maculopathy, Treatment:
A. Panretinal photocoagulation
B. Photocoagulation of macular area
1247- Child presented with gum and nose bleeding and bruising all over
the body after an episode of URTI. Dx:
1) Henoch Scholein Purpura
2) Idiopathic thrombocytopenic purpura
3) Vitamin K deficiency
4) Hemophilia
__________________________________________________________________________
__________________________
1248-Patient on isoniazid for TB prophylaxis, what test should be regularly
done:
1) Spirometry
2) LFT
3) RFT
__________________________________________________________________________
__________________________
1249-Child presented with jaundice, vomiting, hepatomegaly.. etc. What
hepatitis virus is more likely to be the cause:
209

1) A
2) B
3) C
4) D
1250- Regarding strep pharyngitis: same as q13 put choices are
complete
1) No treatment should be given until strep infection is proven.
2) Treatment has no effect on rapidity of solution of infection
3) Treatment prevents post-strep glomerulonephritis
4) Treatment can be postponed for 9 days
5) Clindamycin is the drug of choice
__________________________________________________________________________
__________________________
1251-. What could cause painful vision loss:
1) Acute close angle glaucoma
2) Retinal detachment
3) Retinal vein occlusion
4) Retinal artery occlusion
1252- Female patient with hirsutism, obesity, infertility. US shows
multiple ovarian follicles. Dx:
1) Klinefelters syndrome
2) Ashermans syndrome
3) Kallman syndrome
1) Stein-Leventhal syndrome other name of PSO
1253-. HbA1c is useful in:
1) Adjustment of insulin
2) Monitoring diabetic control on day-to-day basis
3) Longer term diabetic control monitoring
1254-. Child with iron toxicity, best way of management:
1) Gastric lavage
2) Ipecac syrup
3) Magnesium citra
Forgot the rest of choices, but there was no deferroxamine or charcoal.
__________________________________________________________________________
__________________________
1255- Patient presented with typical symptoms of hyperthyroidism.
Whats the most effective and rapid way to relieve symptoms:
1) Propranolol
2) PTU
3) Radioactive iodine
4) Surgery
1256- Young patient with unremarkable medical history presented with
SOB, wheeze, long expiratory phase. Initial management:
1) Short acting B agonist inhaler
210

2) Ipratropium
3) Steroids
4) Diuretic
1257-Patient with moderate persistent BA, on short acting B agonist and
low dose steroid inhaler. What will be the next step:
1) Add long acting B agonist to steroid
2) Increase dose of steroid
3) Theophylline
4) Ipratropium
1258-young male has a painless mass in the testis that is increasing with time what is
your advice:
US and consult surgeon
1259-drug binds to the bile and prevent its reabsorption:
cholystramine
1260-female patient on antiepliptic drugs wants to become pregnant what well you tell
her regarding eplipsy:
use of antiepliptic has risk of fetal malformation
epliptic attacks affects the fetus
1261-pt has solid dysphagia best for diagnosis:
endoscopy +biopsy
1262-pt has DM2 and HTN an CCB+metformin+glyburide+statin still having increase
BP what is your advice:
add ARBs
increase CCB dose
start thiazide
1263-In cervical LNs there are well differentiated thyroid cells, during operation you
find no lesion on thyroid what will you do next
A. Total thyroidectomy
B. Total thyoidectomy + radical cervical LNs dissection
C. Total thyroidectomy + specific LNs dissection
D. Thyoid lobectomy with ----e-thyroid lobectomy and isthmusthectomy and removal of all local enlarged lymph
nodes (new choice not in alqaseem q)
1264-a young healthy male complain of sleep apnea on examination there is only
enlarged tonsils mangment:
adenoidectomy
reduce weight
1265-long scenario about young male with spoon shaped nails:
iron deficiency anemia
211

1266-Which of the following radiological feature is characteristic of military


TB?
Sparing of lung apices
Pleural effusion
Septal lines
Absence of glandular enlargement
Presence of small cavity
1267-Pt with sudden cardiac arrest the ECG showed no electrical activities with
oscillation of QRS with different shapes. The underlying process is:
a. Atrial dysfunction
b. Ventricular dysfunction
c. Toxic ingestion
d. Metabolic cause
1268-An old patient presents with historyof uncontrolled HTN and DM . his hand is
big and tall in examination bulsitale abdominal mass. What is the diagnosis?
A. Peptic ulter
B. GERD
C. aortic aneurysm
D. marfan syndrome??
1269-in Window period of hepatits B
1) HBc
2) HBs ag
3) HBs antibody
HBc Ab ( IgM )
1270-Children while he was playing a football , the ball hit his
hand from lateral fingers, after a while the children
complaing pain and swelling on those fingers and painful
middle finger with hyperextension of interphalengial joint,
swelling was more in the DIP and IP Joints , also , there was
pain on his palm, what is the most likely cause:
Rupture of profound ms in hand
Rupture of superfacial ms
Intra articular interphlengial joint fracture
Extra articular
rupture of flexor digitorum profundus
1271-

212

Vous aimerez peut-être aussi